You are on page 1of 126

Table of Contents

Table of Contents – Geography


CHAPTER 07: MINERALS IN INDIA ........................................................................ 1 
MINERALS IN INDIA: ..................................................................................................................................... 1 
Classification: ............................................................................................................................... 1 
Distribution of Minerals (India): .................................................................................................... 2 
Review Questions and Answers: ............................................................................................. 3 
Miscelleous questions / home work ......................................................................................... 8 
Previous Years Board Questions: ........................................................................................... 8 

CHAPTER 08: AGRICULTURE IN INDIA ............................................................... 11 


CHAPTER MAP: ......................................................................................................................................... 11 
AGRICULTURE IN INDIA: ............................................................................................................................. 11 
Types of Agriculture in India: ........................................................................................................... 11 
Problems of Agriculture in India: ..................................................................................................... 12 
Solution: ..................................................................................................................................... 12 

CHAPTER 09: FOOD CROPS ................................................................................ 13 


Main crop seasons: ......................................................................................................................... 13 
Rice: ........................................................................................................................................... 13 
Wheat: ........................................................................................................................................ 14 
Millets: ............................................................................................................................................. 14 
Jowar, Bajra, Ragi: ..................................................................................................................... 14 
Pulses:........................................................................................................................................ 15 
Food Crops of India: ................................................................................................................... 16 
Review Questions and Answers ............................................................................................ 17 

CHAPTER 10: COMMERCIAL OR CASH CROPS & DISTRIBUTION OF CROPS


AND THEIR REQUIREMENT .................................................................................... 26 
Commercial or Cash Crop: .............................................................................................................. 26 
Definition: ................................................................................................................................... 26 
Types of Cash Crops: ................................................................................................................ 26 
Plantation crops:......................................................................................................................... 28 
Oil–Seeds of India: ..................................................................................................................... 33 
Beverages Crops:....................................................................................................................... 34 
Fibre Crops Of India: .................................................................................................................. 34 
Review Questions and Answers ............................................................................................ 34 
Miscelleous questions / home work ....................................................................................... 43 
Previous Board year questions:............................................................................................. 45 

CHAPTER 11: INDUSTRIES IN INDIA (MINERAL BASED & AGRO–BASED)..... 49 


Manufacturing Industries: ................................................................................................................ 49 
Need of Industrialization: ................................................................................................................. 49 
Factors affecting the infrastructure of Industries: ............................................................................ 49 
Geographical Factors: ................................................................................................................ 49 
Commercial Factors: .................................................................................................................. 50 
Classification of Industries:.............................................................................................................. 50 
Distribution of Industrial Regions: ................................................................................................... 50 

Volume 2 of 4 Universal Tutorials – X ICSE – Geography


Agro–based Industries: ................................................................................................................... 50 
Textile Industry: .......................................................................................................................... 51 
Sugar Industry: ........................................................................................................................... 51 
Review Questions and Answers ............................................................................................ 52 
Miscelleous questions / home work ....................................................................................... 59 
Previous Board year questions:............................................................................................. 59 

CHAPTER 12: MINERAL BASED INDUSTRIES .................................................... 60 


SYNOPSIS: ................................................................................................................................................ 60 
Iron and Steel Industry: ................................................................................................................... 60 
Heavy Engineering Industries: ........................................................................................................ 61 
Heavy Electrical Industry: ................................................................................................................ 62 
Petrochemical:............................................................................................................................ 62 
Electronic Industry: ..................................................................................................................... 62 
Review Questions and Answers: ........................................................................................... 63 
Miscelleous questions / home work ....................................................................................... 71 
Previous Years Board Questions: ......................................................................................... 71 

CHAPTER 13: TRANSPORT .................................................................................. 76 


INDIAN TRANSPORT: .................................................................................................................................. 76 
Road Transport: .............................................................................................................................. 76 
Rail Transport: ................................................................................................................................. 77 
Water Transport: ............................................................................................................................. 77 
Factors affect water transport: ................................................................................................... 77 
Air Transport: ................................................................................................................................... 78 
Major International Airports: ....................................................................................................... 78 
Review Questions and answers: ........................................................................................... 78 
Miscelleous Questions / home work ...................................................................................... 83 
Previous Years Board Questions: ......................................................................................... 84 

CHAPTER 14: SOURCES OF WASTE ................................................................... 85 


What is waste? ................................................................................................................................ 85 
Characteristics: ................................................................................................................................ 85 
Types of waste: .......................................................................................................................... 85 
Waste can also be classified into the following categories: ....................................................... 85 
Sources of Waste ............................................................................................................................ 86 
Domestic Waste: ........................................................................................................................ 86 
Industrial Waste:......................................................................................................................... 86 
Agricultural waste ....................................................................................................................... 87 
Municipal waste .......................................................................................................................... 88 
Bio–medical wastes ................................................................................................................... 88 
Nuclear waste............................................................................................................................. 89 

CHAPTER 15: NEED FOR MANAGEMENT OF WASTE ....................................... 90 


Introduction:................................................................................................................................ 90 
Transmission of Diseases ............................................................................................................... 90 
Waste on land : .......................................................................................................................... 90 
Common water borne diseases ................................................................................................. 90 
Greenhouse effect and global warming: .................................................................................... 91 
Greenhouse gases ..................................................................................................................... 91 

Universal Tutorials – X ICSE – Geography Volume 2 of 4


Table of Contents
Effects of Global Warming are: .................................................................................................. 91 
Depletion of Ozone Layer................................................................................................................ 91 
Acid Rain ......................................................................................................................................... 92 
Effects of Acid Rain .................................................................................................................... 92 
Soil Health ....................................................................................................................................... 92 
Waste management ........................................................................................................................ 92 

CHAPTER 16: IMPACT OF WASTE ACCUMULATION ......................................... 93 


Introduction: ..................................................................................................................................... 93 
Spoilage of Landscape: ................................................................................................................... 93 
Pollution ........................................................................................................................................... 93 
Eutrophication: ........................................................................................................................... 94 
Health hazards ................................................................................................................................ 94 
Spread of Disease Through Contamination: .............................................................................. 94 
Effects of Toxic Particulate Materials ......................................................................................... 94 
Effect on Terrestrial Life .................................................................................................................. 94 
Effects on Human Beings:.......................................................................................................... 95 
Effects on Plants : ...................................................................................................................... 95 
Effects on Animals and Birds : ................................................................................................... 95 
Effect on Aquatic Life: ..................................................................................................................... 95 
Biomagnification: ........................................................................................................................ 95 
Example: .................................................................................................................................... 96 
Case study: Minamata tragedy .................................................................................................. 96 

CHAPTER 17: SAFE DISPOSAL OF WASTE ........................................................ 97 


Segregation ..................................................................................................................................... 97 
Dumping .......................................................................................................................................... 97 
Sanitary Landfill .......................................................................................................................... 97 
Advantages ................................................................................................................................ 97 
Plantation at Landfill Site............................................................................................................ 98 
Municipal Waste Management: ....................................................................................................... 98 
Collection of Municipal Solid Wastes ......................................................................................... 98 
Storage of Municipal Solid Waste .............................................................................................. 98 
Transportation of Municipal Solid Wastes: ................................................................................. 98 
Segregation of Municipal Solid Wastes: .................................................................................... 98 
Composting: .................................................................................................................................... 99 
Mechanical Method: ................................................................................................................... 99 
Manual Method........................................................................................................................... 99 
Advantages of Composting ........................................................................................................ 99 
Drainage and Treatment of Effluents .............................................................................................. 99 
Primary Treatment:................................................................................................................... 100 
Secondary Treatment of Water ................................................................................................ 100 
Tertiary Treatment of Water: .................................................................................................... 101 
Incineration .................................................................................................................................... 101 
Advantages of Incineration....................................................................................................... 101 
Scrubber ................................................................................................................................... 101 
Electrostatic Precipitators (ESPs) ................................................................................................. 102 
Advantages of ESPs ................................................................................................................ 102 

CHAPTER 18: REDUCE-REUSE-RECYCLE........................................................ 103 


Introduction: ................................................................................................................................... 103 

Volume 2 of 4 Universal Tutorials – X ICSE – Geography


Reducing the waste ....................................................................................................................... 103 
Reusing the Waste: ....................................................................................................................... 103 
Recycle of Waste: ......................................................................................................................... 103 
Plastic: ...................................................................................................................................... 104 
Paper: ....................................................................................................................................... 104 
Government initiatives ................................................................................................................... 104 
Social Initiatives ............................................................................................................................. 105 
Individual Initiative: ........................................................................................................................ 106 
Review Question and Answer: ............................................................................................ 107 
Miscelleous questions / home work ..................................................................................... 115 
Previous Years Board Questions: ....................................................................................... 116 

Universal Tutorials – X ICSE – Geography Volume 2 of 4


Chapter 07: Minerals in India 1

Chapter 07: Minerals in India


→ Minerals in India (Coal, Petroleum, Iron Ore)
→ Classification
→ Distribution

Minerals in India:
Classification:
Metallic Minerals Non–Metallic Minerals Minerals Fuels
Iron ore, Manganese, bauxite Limestone Coal, Petroleum
 All minerals and mineral products are derived from rocks forming the earth’s crust. The various
mineral products of the rock system of India are the ores and minerals, which man has been
used in his development throughout the ages.
 As one looks around, the tractor farming, the wide range of heavy machinery for mines, mills
and factories, mineral fertilizers – all these bear ample testimony to the indispensability of
minerals in the’ life of man. The ever-changing techniques of construction, engineering, the raw
materials for road, rail and air transport etc. demand an ever-increasing use of metals and
minerals.
 An appraisal of the total mineral resources of India so far known to geologists brings home the
fact that the mineral wealth of India is not inconsiderable for a country of her size and
population.
 Nature has made a very unequal territorial distribution of minerals in Indian region. The vast
alluvial plain tract of northern India is devoid of mines of minerals. The terrain of Bihar,
Jharkhand and Orissa possesses the largest concentration of ore deposits such as iron,
manganese, copper, thorium, uranium, aluminium, mica, phosphates and coal.
 Madhya Pradesh and Chhattisgarh also carry good reserves of iron and manganese ores, coal,
limestone, and bauxite. Tamil Nadu has workable deposits of iron, manganese, mica,
limestone and lignite.
 Gujarat, Maharashtra and Assam have crude oil (petroleum). Assam also bears tertiary coal.
West Bengal’s minerals are confined to coal and iron ore.
 The most important source of power in Indian Subcontinent is coal. It is used in industries,
thermal power plants, railways, etc.
 Depending upon the carbon content and moisture, coal is divided into the following varieties:
z Anthracite coal: It has a carbon content of 90% and burns without smoke leaving a little
ash.
z Bituminous coal: It has a carbon content of 50% to 80%.
z Lignite: It is brownish in colour and has a carbon content of about 40%.
z Peat: It has least carbon content and inferior to the other varieties.
 India is the largest producer of coal in South Asian region. Bihar, Assam, Jharkhand, West
Bengal, Andhra Pradesh, Madhya Pradesh, Chhattisgarh and Maharashtra are the main coal
producing States of India.
 Petroleum is an important fuel. The prosperity of a country depends to some extent on this fuel.
Unfortunately, South Asian region generally lacks petroleum, however, there are some oil fields
Volume 2 of 4 Universal Tutorials – X ICSE – Geography 1
2
in India and Pakistan. India produces only one-third of her requirements. The remaining two-
third requirements are met by imported petroleum.
 In India, the main areas of mineral oil deposits are:
z Offshore in Maharashtra (Mumbai High is the largest producer of mineral oils in India.)
z Digboi and Naharkatiya in Assam
z Kalol and Ankleshwar in Gujarat
 The crude mineral oil is refined in refineries. There are 17 refineries in India.
z Reliance Petroleum Limited is the only refinery in the private sector.
z The Mangalore Refinery and Petrochemical Limited is a joint sector refinery.
z 15 refineries are in public sector.
 Iron is a metal of universal use. It’s uses have increased thousand fold since man discovered
steel. There is a wide variety of special steel to suit special purposes such as fatigue and acid-
resisting steels, hard steel for machinery tools, stainless steel and other special steels for
defence weapons.
 Iron is extracted from its ores – haematite and magnetite, which are oxides of iron.
 When these are smelted with coke, metallic iron is obtained.
 Iron-ore occurs in a large scale in India. Orissa, Jharkhand, Chhattisgarh, Karnataka, Goa and
Andhra Pradesh are the main producers of iron in India.
 Orissa accounts about one-third of total output of iron in India. The most important deposits
occur in Sundargarh, Mayurbhanj, Cuttack, Shambalpur and Koraput.
 Jharkhand ranks next to Orissa in the production of iron ore. The iron ore of Singhbum is of the
highest quality.

Distribution of Minerals (India):


a) Metallic:
Minerals States and important Producing Centres Uses
/Other Countries
(Imp.) Iron ore: NE Peninsular India(mainly the Used in the manufacture of iron and steel.
– India is very rich: ¼ of Chhotanagpur region)
the world’s reserves Bihar : Singhbhum,
– Annual Production: over Orissa : Mayurbhanj, Keonjhar
40 Million Tonnes M.P. : Bailadila
Karnataka : Kudremukh
Tamil Nadu : Salem
Goa
Manganese: M.P. : Balaghat, Chindwara Used as raw material for iron and steel
– Very rich Maharashtra : Bhandara, Nagpur industry.
– Annual Production: 1.6 Gujarat : Panch Mahal To make steel tough and rust proof.
Million Tonnes Orissa : Keonjher Removes impurities e.g. sulphur.
Karnataka : Chitraldurg For manufacture of heavy chemicals,
bleaching powder, glass and electrical
industries.
Bauxite (ore of aluminium): Bihar : Palamau, Ranchi It is the main ore for aluminium, which is
– Sufficient M.P. : Jabalpur, Katni used in aircrafts, automobiles, rail wagons,
– Annual Production: 1.7 Maharashtra : Kolhapur coaches shipping industry and household
appliances, reflectors and mirrors.
Million Tonnes Tamil Nadu : Salem
Karnataka : Belgaum
Gujarat : Kheda

2 Universal Tutorials – X ICSE – Geography Volume 2 of 4


Chapter 07: Minerals in India 3

b) Non–Metallic:
States and important Producing Centres
Minerals Uses
/Other Countries
Limestone: AP : Guntur, Kuddapah Used mainly in the manufacture of cement.
– Enough Gujarat : Junagarh, Kachchh Used as flux in iron and steel industry;
– 30.2 million tones Rajasthan : Ajmer, Jaipur, Jodhpur production of chemicals, paper glass and
Karnataka : Bijapur, Shimoga fertilizers.
M.P. : Durg, Raipur

c) Minerals: Fuels:
(Imp.) Coal: Bihar : Jharia, Bokaro, Gridih, Karanpura Coal is the most widely used source of
– Just sufficient West Bengal : Raniganj energy by industries and power generation.
– 102.0 million tonnes M.P. : Singrauli, Korba Largest consumers of coal are Indian
Orissa : Talcher Railways, Iron and steel industry.
– Lignite Coal 3.6 million
tonnes Also in AP, Jammu & Kashmir, Assam
th India’s largest area is at Neyveli in Tamil
– 7 largest production in
Nadu.
the world 234.11 million
tonnes of reserves.
(Imp) Petroleum: Assam : Digboi, Moran, Naharkatiya Petroleum is the source of primary
– India is poor Dibrugarh, Sibsagar commercial energy.
– 12.0 million tonnes Gujarat : Ankaleshwar, Kalol Used for electricity generation.
Khambat, Mehsana Used for automobiles and aeroplanes.
Mumbai high, Kaveri and Godavari basin, Products obtained are Petrol, diesel,
offshore areas of Andaman and Nichobar kerosene, benzene, vaseline, paraffin wax,
Islands. tar etc.

REVIEW QUESTIONS AND ANSWERS:

Section I: [2 Marks]
1) What is the difference between metallic and non-metallic minerals? Name one each of the
most abundant of these minerals found in South Asia.
Ans:
Metallic Minerals Non–Metallic Minerals
These minerals are those which possess the Some minerals that have no metals in them
physical properties of lustre, hardness and are used for extraction of non metals like
heaviness. The metals can be melted, and sulphur, phosphorus, carbonate, etc. They
exist in chemical compounds. Only few occur are also mineral fuels.
in a pure state.
In South Asia the most abundant ores are Minerals like limestone, gypsum, mica,
iron, copper, tin, zinc, lead, silver and gold. antimony, coal and petroleum are non-
metallic minerals.
2) Why does India export iron ore on a large scale? Give two reasons.
Ans: i) Iron–ore is one of the most important mineral products of the Indian subcontinent. India
occupies ninth position in world production. A lot is being consumed on the national front, as
most of the iron and steel industries are located in mining areas. But it is also being exported
on a large scale as the demand for steel has increased globally.
ii) The quality of iron-ore found is haematite. It is a good variety; India exports this to countries
like Japan.
3) a) Name important ores of iron.
b) Which of these ores are found in India?
c) Give two features of these iron deposits.
Ans: a) Haematite, magnetite and limonite are iron ore deposits.

Volume 2 of 4 Universal Tutorials – X ICSE – Geography 3


4
b) Haematite is widely found in India.
c) i) Iron ore deposits in India are not very far from the areas producing them,
ii) They have a low sulphur content.
4) i) Which Indian state leads in the production of iron ore?
ii) List the important iron-ore fields in the state of Orissa.
Ans: i) Orissa. It accounts for nearly one-third of the iron output in India.
ii) Sundargarh, Mayurbhanj, Cuttack, Koraput, Keonjhar, Badampahar, Kiriburu, Bohnai etc.
5) Name the chief iron-ore producing areas in:
(a) Chhattisgarh, (b) Maharashtra and (c) Jharkhand
Ans: a) Chhattisgarh – Bastar and Durg districts.
b) Maharashtra – Ratnagiri, Raigarh and Satara districts.
c) Jharkhand – Singhbhum and Palamau
6) Name a few areas where iron ore is found in: (a) Andhra Pradesh, (b) Tamil Nadu and
(c) Karnataka.
Ans: (a) Andhra Pradesh – Cuddapak and Chabali. (b) Tamil Nadu – Salem and Tiruchirapalli and
(c) Karnataka – Kemmangundi, Baba Budan Hills.
7) Name the important oil fields in India.
Ans: i) Important oilfields in India are Mumbai High–it is an off shore oil reserve.
ii) Oilfields of the Eastern region: Digboi, Naharkatia, Rudra Sagar, Nunmati Bappapunga,
Hansapunga, Naharkatiya and Moran in Assam.
iii) Oilfields of Western region: Ankleshwar, Koyali, Kolar, Kosamba, Sanand, Kathana,
Navgaon in Gujarat, Cambay in Gujarat.
8) i) Name the oldest oil field in India.
ii) Name three other oil fields in the same region.
Ans: i) The first oil well was drilled at Digboi in Lakhimpur district of upper Assam in 1867.
ii) Other oil fields of the Eastern region are Digboi, Rudra Sagar, Nunmati Bappapunga,
Hansapunga, Naharkatiya and Moran in Assam.
9) Name the source of crude oil for Mathura and Panipat oil refineries.
Ans: Mathura (U.P.) and Panipat (Haryana) oil refineries import crude oil, which is transported through
pipelines for more than 1200 km from the Gulf of Kachchh. Some crude oil is also obtained from
Mumbai High.
10) Name the largest deposits of oil and natural gas in India. Name the refinery associated
with it.
Ans: The largest reserve of oil and natural gas is in Mumbai High. The region produces more than
one third crude oil output of India. It is refined in the refineries of Trombay.
11) Name two inland centers of oil refineries in India.
Ans: The important inland centers of oil refineries are:
i) Nunmati, Numaligarh, Bongaigaon near Guwahati and Digboi in Assam.
ii) Barauni in Bihar.
iii) Mathura in U.P.
iv) Panipat in Haryana.
12) Name two coastal oil–refineries, one on the east coast and the other on the west coast of
India.
Ans: Oil–Refineries:
East Coast West Coast
Vishakhapatnam in Andhra Pradesh, Trombay (HPCL and BPCL) in Mumbai,
Chennai in Tamil Nadu Kochi in Kerala
Haldia near Kolkata Mangalore in Karnataka

4 Universal Tutorials – X ICSE – Geography Volume 2 of 4


Chapter 07: Minerals in India 5

13) Name the types of rocks where mineral oil is usually found.
Ans: Mineral oil is a product of decomposition of organic remains embedded in sedimentary rocks of
the Tertiary period. Petroleum is usually found in sedimentary rock formations like sand–stone,
shale and limestone.
14) Explain why coal is often used near the source of production and mineral oil is often
transported to great distances
Ans: Coal is a cheap and heavy mineral. Since coal is a bulky mineral, the transportation by road or
rail is costly and time–consuming. It is therefore, used near the source of production.
Mineral oil, on the other hand, is a fluid substance. It can be transported through pipelines with
minimum transportation cost. The laying of the pipelines is the only initial cost.
15) State two main drawbacks of the coal found in India.
Ans: The two main drawbacks of coal found in India are:
i) India does not have extensive deposits of the high grade coal i.e. Anthracite coal.
ii) Most of the coal produced is bituminous coal containing 50% to 80% of carbon content.
About 60% of India’s coal reserves are of inferior quality.
16) Distinguish between the conventional and non–conventional sources of energy.
Ans: Conventional sources of energy: (i) These are non–renewable sources of energy eg. coal and
petroleum. (ii) These are going to last just for 100–200 years. (iii) These cause air and water
pollution.
Non–conventional sources of energy: (i) These are renewable sources of energy eg. solar
energy, wind energy. (ii) These are going to last forever. (iii) These do not cause any pollution.
17) Name two main areas of iron ore in Goa and in Karnataka.
Ans: i) Most of the iron ore of South Asia is produced in India. Goa is the largest producer of about
27% iron ore, through its reserves are small. Most of it is exported. The good quality
haematite ore is mined at Sahqualim, Sanguem, Satari Ponda.
ii) Karnataka is the fifth major producer of iron ore. Reserves of magnetite and haematite are
found here. Important deposits are at Bellary. Hospet, and the Bababudun Hills.
18) What is meant by the term Gondwana deposits?
Ans: i) The coalfields in India are divided into two classes. a) The Gondwana system, b) Tertiary
beds. Gondwana deposits of coal are found in strata extending from Bengal, Jharkhand,
Bihar and Orissa including Madhya Pradesh.
ii) 98% of Indian coal comes from this belt. The Gondwana region has high grades of coal. It is
free from moisture but has sulphur and phosphorus in small quantities. They are located in
rock strata and consist of coking and non–coking coal.
19) Under the present day system of consumption, minerals are on the verge of extinction.
Do you agree? Give three reasons.
Ans: i) Minerals are important for the growth and development of the nation. There are two types of
minerals-metallic and non-metallic. The Indian subcontinent is rich in mineral resources. But
they are on the verge of extinction as they are not evenly distributed in the subcontinent. The
locations found may not be ideal for exploitation, but they are still being exploited. Thus this is
destroying valuable resources.
ii) The quantity and quality also vary. The regions with good quality may not be very well
developed. The infrastructure may not be good. Over mining causes the quality of the mineral
to degenerate.
20) Distinguish between Anthracte and lignite coal
Ans: Anthracite: (i) This is the best quality coal containing over 80% carbon. (ii) It has the highest
heating value.
Lignite: (i) It is a lower grade coal containing about 60% carbon. (ii) It has the low heating value

Volume 2 of 4 Universal Tutorials – X ICSE – Geography 5


6

Section II: [3 Marks]


1) With reference to iron ore, what type of iron ore does India produce and where? How has
it contributed to its industrial development?
Ans: i) Iron ore produced in India are mainly the haematite type (the other two types are magnetite,
and limonite). India has about 6.6% iron-ore reserves of the world.
ii) Iron ore deposits are found in Goa and Orissa which accounts for nearly one-third of the ore
production. It is mined from the Keonjhar district, Jharkhand in Singhbhum and Palamau
district, Chhattisgarh in Bastar and Durg district, Karnataka in Kemmangundi and Bababudin
Hills, Andhra Pradesh in Guntur, Tamil Nadu in Salem and Maharashtra in the Lohara hills.
iii) Industrially, iron ore is very useful. As the ore is found near the coal, dolomite, limestone and
manganese producing areas, it is well utilised in the iron and steel industry. This industry is
regarded as an index of a nation’s business activity and the standard of living. Thus it is a key
to other industries
2) Why are most of India’s oil refineries located along the coasts of India? Give two reasons.
Ans: i) Large deposits of mineral oil is obtained from the continental shelf area, off the coast of
Mumbai, Kachchh, Khambhat, Konkan, Malabar coast, Krishna and Kaveri deltas and
Sunderbans, Oil has been found in Mumbai High. Bassein and Aliabet off the coast of
Maharashtra and Gulf of Khambat. Oil refineries are set up along the coast to avoid
transportation cost.
ii) Two–thirds of the crude oil is imported, hence the refineries are situated near major sea
ports, e.g. Mumbai, Vishakhapatnam, Haldia near Kolkata, Kochi and Chennai. These cities
are also large consumers of petroleum and petroleum products as they are important
industrial centres.
3) What is “Off–shore Drilling”? What do you understand by Mumbai High?
Ans: Crude oil reserves are found in the coastal region and the off–shore continental shelves, e.g. off
the coast of Mumbai. Drilling of oil done in the sea with the help of self–propelled jack–up type of
drilling platform is known as “Off–Shore Drilling.”
Mumbai High: The Oil and Natural Gas Corporation Ltd. (ONGC) of India, discovered oil on the
continental shelf area, off the coast of Maharashtra about 176 km northwest of Mumbai.
It is known as ‘Mumbai High’ because the syncline of the rock structure in which the oil has
been found is higher than the normal height. Drilling of oil is done here with the help of Sagar
Samrat, a self propelled jack–up type of drilling platform.
4) What are the uses of mineral oil?
Ans: i) Petrol and Diesel obtained from mineral oil are used as fuel for automobiles, aeroplanes,
ships and locomotives.
ii) Kerosene oil, which is a by product of petroleum, is used as a domestic fuel and also used for
the generation of electricity.
iii) By products such as lubricants, teryline, paraffin wax, tar and vaseline are used in industry.
5) Describe in detail the factors regarding coal production, under the following heads:
i) Distribution of coal in India. (ii) Demerits of Indian coal. (iii) Need for conservation of
coal resources.
Ans: i) It is found in the Damodar region (West Bengal) i.e. Raniganj which has the oldest coal field.
Coal is also mined in the Mahanadi region–Madhya Pradesh, Orissa, Jharia, Bokaro,
Karanpura, Giridih, Talcher–Son river region in Madhya Pradesh in the Kanan Valley. In
Maharashtra coal is mined in the Wardha valley, in Andhra Pradesh in Singereni, in Tamil
Nadu in Neyveli, good quality coal is found in Goa.
ii) Indian coals are huge reserves mostly found in the Gondwana series in Peninsular India.
About 2% of Indian coal is new (tertiary type). Bituminous coal which is good quality coal is
very less in India. Lignite coal the second variety is mined in Neyveli, Palna in Rajasthan.
Indian coal is mostly peat i.e. inferior quality found in Bihar, Jharkhand, Madhya Pradesh and
Orissa. This variety has a high ash content and low carbon content.
6 Universal Tutorials – X ICSE – Geography Volume 2 of 4
Chapter 07: Minerals in India 7
iii) Coal conservation means progress in industry. Hence new reserves of coal should be
explored. Low grade should be blended with superior quality. Public and Private sector
participation should be encouraged. New techniques in mining for production influx should be
introduced. Steps should be taken to prevent wastage of coal.
6) Mention three environmental factors that must be kept in mind during mining activities
Ans: i) The mines in India are lack of infrastructure. They are very small and labour-intensive. Thus
environmental pollution is seen.
ii) The mines incur heavy losses due to fires in the mines. The smoke causes heavy pollution to
the surrounding areas, loss of life and property.
iii) The mines should have facility for disposal of waste, specially coal, as this waste causes
pollution which is hazardous to health. New modern techniques are necessary as obsolete
methods cause a lot of damage.
7) Name the different types of coal produced in India. Mention the characteristics of each
type. Name the three types of coal found in India and state the uses of each.
Ans: a) The different types of coal are:
i) Anthracite coal: It has 90% carbon content. It is jet black in colour and burns slowly without
smoke or soot. It is clean to handle and has high heating value.
ii) Bituminous coal: It has 50–80% carbon content. It is most widely found and used in India.
Used for coke required for smelting of iron ore.
iii) Lignite coal: It contains 40% carbon. It is brown or brownish in color. It has large quantities
of ash and moisture.
b) The three types of coal found in India and their uses are as follows:
i) Anthracite coal: It is used in manufacturing of iron and steel and steam generation.
ii) Bituminous coal: It is used in the smelting of iron ore and also in the generation of thermal
electricity.
iii) Lignite coal: It is used in the production of thermal electricity.
8) Name the main coal bearing areas of India.
Ans: The coalfields in India are classified into two groups.
i) Gondwana coal fields: found mainly in Peninsular India in the states of Jharkhand, Orissa,
Madhya Pradesh, West Bengal, Andhra Pradesh and Maharashtra.
ii) Tertiary coal fields: They include the coal–fields of Assam, Jammu and Kashmir, Rajasthan
and Punjab.
9) Name the following:
(i) The oldest coal field in India. (ii) The largest coalfields in India. (iii) Any two states in India
important for production of coal.
Ans: i) The oldest coal field in India is Talcher coalfield in Orissa.
ii) The largest coalfield in India is the Jharia coalfield.
iii) Jharkhand and West Bengal are two states important for the production of coal.
10) What are the uses of coal?
Ans: Uses of coal are as follows:
i) Coal is extensively used as domestic and industrial fuel.
ii) Coke obtained from Bituminous coal is used in the smelting of iron ore.
iii) It can be converted into other forms of energy like gas and oil. Coal is used in the generation
of thermal electricity.
 By–products such as tar, ammonia benzol, sulphur, and phenol are obtained during conversion
of coal to coke.
 Coal is an important raw material for various industries like chemicals, explosives, dyes,
fertilizers, perfumes, plastics and paints.
 High grade coals are used in iron steel industry and steam generation.
 In India the main consumers of coal are the Indian railways, and the iron and steel industry.

Volume 2 of 4 Universal Tutorials – X ICSE – Geography 7


8

11) Write a brief note on Gondwana coal.


Ans: Gondwana coal is mainly Bituminous coal. It is mostly moisture free, but it contains variable
quantities of sulphur and phosphorous.
The Gondwana coalfields are largely located in:
i) The valley of the river Damodar, which include Jharia, Raniganj, Giridih, Bokaro & Karnapura
ii) The coal–field in the valley of river Mahanadi.
iii) The coal–fields in the valley of river Godavari.
12) What do you understand by tertiary coal fields?
Name the regions of such coalfields.
Ans: The Tertiary group of coalfields account for about 2–3% of the total coal production.
 The coal found here is lignite coal, which has low carbon percentage and high percentage of
moisture and sulphur.
 The tertiary coalfields are located mainly in Assam and Rajasthan. It also includes coal fields of
Jammu and Kashmir and Punjab.
 The distribution of coal fields is as follows:
z Assam; Makum coalfields (extending from Naga Patkai ranges (Nazira) to Lakhimpur
district (Jeypore).
z The coal is of good quality, largely used by the railways, steamer companies and the tea–
factories.
z Rajasthan: Pallan in southern Rajasthan and Umarsar in Gujarat have deposits of lignite
coal.
z Tamil Nadu: Neyveli area of south Arcot district account for 90% of the lignite of reserves.
Jayamkondam of Trichy district, Mannargudi and East Veeranam account for a large part of
the lignite coal out put.

MISCELLEOUS QUESTIONS / HOME WORK


1) What is a mineral?
2) Name the quality of Gondwana Coal reserves.
3) Name on eimportant oilfield in India.
4) Name the Coalfield in West Bengal.
5) Mention two uses of coal.
6) Name the two main belts of India whre oil is found.
7) Where is the oldest oilfield located in India?
8) Name two main coal bearing areas of India.
9) Name two States that have huge deposits of the Gondwana coal.
10) Name two States that have petroleum deposits.
11) What are the advantages of the anthracite coal?
12) Why petroleum is called ‘liquid gold’?
13) Name two uses of iron ore.
14) Where is the largest aluminium plant in India located?
15) Give two uses of limestone.

PREVIOUS YEARS BOARD QUESTIONS:


1) Describe in detail coal under following heads:
a) Distribution of coal in India?
b) Demerits of Indian coal (any two).
c) Need of conservation of coal (two points). [2000]
2) Name two States in India where iron-ore is found on a large scale. [2005]
8 Universal Tutorials – X ICSE – Geography Volume 2 of 4
Chapter 07: Minerals in India 9
3) a) Name two important oil fields in India. [2005]
b) State two main drawbacks of the coal found in India.
c) Name the two main belts of India where oil is found.
4) a) Name two states in India where manganese is found. State one use of manganese.
b) Name the four types of coal. Which of these is the best for industrial purposes? Justify your
answer.
c) Which country in South Asia is the large the producer of bauxite? Why bauxite is considered
an important mineral? [2006]
5) a) Name the different types of iron ore found in India.
b) Name an important coal producing State in India and a coal-mine located in that State. [2007]
6) a) Name two states with large deposits of coal. Name the coal fields in the states that you have
named.
b) What is lignite? Name one place in India where it is mined.
c) i) Mention two uses of mineral oil.
ii) Name an old and a new mineral oil producing area.
d) i) Mention two reasons why minerals are important?
ii) Name one area in Orissa and one area in Chhattisgarh where iron ore is mined. [2008]
7) a) Which state is the largest producer of mineral oil? Name any two oil refineries of India?
b) Name any two off–share oil fields of India.
c) i) Name the oldest and the largest coal field in India.
ii) Name any two raw materials derived from coal.
d) Name the different types of iron ore found in India. Which is the best quality iron ore? [2009]
8) a) Name one centre in each of the following states where iron is mined:
i) Orissa ii) Jharkhand.
b) Which variety of coal is popular for domestic use? Give a reason for your answer.
c) Name a region which has natural gas deposits. Mention two uses of natural gas.
d) Why is an oil refinery located either close to an oilfield or in a coastal city? Name one oil
refinery in the private sector. [2010]
9) a) What grade of iron-ore is mostly mined in India? Name two leading iron-ore producing states.
b) What is lignite? Name the two areas where lignite is found in India.
c) Name the leading producer of manganese in India, Name two important industrial uses of
manganese.
d) Name the two states where limestone is found. Mention two important uses of limestone.
[2011]
10) a) i) Name two leading states producing Manganese.
ii) Name one use of the mineral.
b) i) Name two varieties of iron ore used in industry.
ii) How is the low grade iron ore utilized?
c) Give geographical reasons why:
i) Anthracite is used for domestic purposes.
ii) Oil refineries are located close to oil fields or near ports.
iii) the location of coal fields is an important factor in industial development.
d) Name the mineral:
i) which is converted to aluminium
ii) which is used in the manufacture of cement.
iii) the largest deposits of which are found in Balaghat in Madhya Pradesh. [2012]

Volume 2 of 4 Universal Tutorials – X ICSE – Geography 9


10
11) a) Name the ore of aluminium. Describe two main uses of aluminium.
b) Name any two industrial products for which limestone is used as a source of raw material.
c) i) Name two industries that use a high quantity of coal.
ii) Name one important area that has large coal deposits in the states of Jharkhand and West
Bengal.
d) i) Which state is the largest producer of mineral oil?
ii) Name two coastal and two inland oil-refineries in India? [2013]
12) a) Mention any two uses of manganese.
b) Which of the different varieties of coal is used for domestic purposes and why?
c) Name the mineral used in the manufacture of:
i) Cement ii) Aluminium iii) Synthetics
d) Which State is the leading producer of the following minerals?
i) Coal ii) Oil iii) Manganese [2014]

10 Universal Tutorials – X ICSE – Geography Volume 2 of 4


Chapter 08: Agriculture in India 11

Chapter 08: Agriculture in India

Chapter Map:
→ Agriculture
→ Types
→ Crop seasons
→ Problems of agriculture and solution
→ Classification of crops
→ Food crops (Rice, Wheat, Millets)
→ Cash crops or commercial crops (Oil seeds, Fibre crops, Beverage crops)

Agriculture in India:
The cultivation of the soil in order to grow crops and rear livestock is known as agriculture.

Types of Agriculture in India:


Subsistence Farming:
z It is the type of agriculture in which farmers work hard to grow enough food to survive only.
There remains no surplus to sell in the market. This type of farming is practiced in tribal
areas of Assam, other north–eastern hilly states and in the Himalayan region.
Mixed Farming:
z It is another type of farming in which cultivation of crops and rearing of animals are done
together on the same farm. Mixed farming keeps the farmers better–off and more secure,
because their income comes from various sources. Rotation of crops is practiced.
Shifting Cultivation:
z It is the oldest type of agriculture. This is also as ‘slash and burn’ method. In India, it is
known as Jhumming. This method of farming is carried on in jungles of north–eastern parts
of India. A patch of land is cleared for cultivation. As the yield decreases after two or three
years, the patch is abandoned and a fresh clearing is made. Shifting agriculture, in India, is
carried in the hill of Assam, Arunachal Pradesh, Mizoram and Nagaland.
Extensive Farming:
z It is practiced in regions where the population size is small and land is enough. Here, per
acre yield is low but overall production is in surplus due to less population. Agriculture is
done with the help of machines. In our country, extensive farming is practiced in the Terai
region of Sub–Himalayas and in parts of North–western India.

Volume 2 of 4 Universal Tutorials – X ICSE – Geography 11


12

Intensive Farming:
z In regions where the size of population is big but land is less, this type of farming is done.
Annually two or three crops are grown due to the demand of food for the large size of
population. Agriculture is done with the help of manual labour. Intensive farming is
widespread in the irrigated areas of the Northern Plains and coastal strips of South India.
Plantation Agriculture:
z It is also a type of agriculture in which trees or bushes are planted on huge estates. A single
crop like rubber, sugarcane, coffee, tea or banana is grown. These crops usually cater to
the export market and earn foreign exchange. Plantation farming is carried on in some parts
of India like the hills of south India and North–East states of India where tea, coffee and
rubber are cultivated.
Commercial Farming:
z This system of agriculture involves cultivation of crops for sale in the market. These crops
are called commercial or cash crops. They include sugarcane, tobacco, fibre crops and
oilseeds.
z It is usually practiced in areas where the population is sparse and plenty of spacious land is
available and market economy is well–developed.

Problems of Agriculture in India:


The problems faced by Indian agriculture are as follows:
 Soil erosion due to heavy monsoon rains, floods, gusty winds and insufficient vegetation cover
have increased the infertility of the soil.
 In equal distribution of rainfall is the cause of the failure of rain –fed crops.
 Due to the illiteracy, farmers cannot use modern scientific methods of cultivation.
 An unsound credit system and the poverty of the farmer they do not have capital to invest in
improvements.
 Majority of land holdings in India are very small. These small and fragmented holdings cannot
promote modern agriculture.

Solution:
 Improving the health of the farmers.
 Giving them letter credit facilities, improved hybrid seeds, chemical fertilizer and educating the
farmers in modern scientific methods of cultivation
 Reclaiming land and preventing fragmentation.
 Farming co–operatives, offering incentives and bonus to farmers.
 Introduced scientific farming programmes

12 Universal Tutorials – X ICSE – Geography Volume 2 of 4


Chapter 09: Food Crops 13

Chapter 09: Food Crops


Main crop seasons:
There are three main crop seasons in India.
 Kharif: The Kharif season begins with the onset of the monsoons in June–July. The crop
grows in the rainy season and harvesting takes place in the beginning of November. Rice,
maize, millets, groundnuts, cotton and jute are the principal crops grown in the Kharif season.
 Rabi: This season starts after the rainy season. Sowing begins in October–November and
harvesting takes place in the beginning of summer in March–April. Rabi season is cooler and
drier than the kharif season. Wheat, barley, pulses and some oil–seeds are grown in this
season.
 Zaid: This is the summer season for growing crops which remain till April, May and June. The
products grown in this season are mainly vegetables and fruits.
MAIN CROPS IN INDIA

Cereals Plantations Cash Crops


wheat, rice, jowar, coffee, rubber, tea. sugarcane, tobacco
gram, bajra, ragi,
pulses. Oilseeds Spices Fibre Crops
linseed, groundnut, Pepper, ginger, cotton, jute
seasame, sunflower, turmeric, chillies,
rapeseed, cloves, saffron
mustardseed

Rice:
 Rice is the staple food of millions of people in India. It is a kharif crop. After China, India is the
largest producer of rice.
 Geographical Requirements:
z Temperature: 22°C to 32°C average 24°C.
z Rainfall: 150 cm to 300 cm.
z Soil: Alluvial topsoil clayey impervious subsoil
Methods of Cultivation:
z Board-casting: The seeds are scattered all over the field after ploughing it. It is labour
saving also. This is done before the onset of monsoon.
z Drilling method: In this method, the seeds are dropping in a straight line at the regular
intervals through a bamboo shaft attached to the plough which makes furrows. The main
advantage of this method is that the seeds fall in the furrows in a systematic way. The
germination rate of the seeds is high and the wastage of seeds in minimal, as well as time
consuming.
z Dibbling method: Dibbling is the dropping of seed at regular intervals in the furrows made
by the plough.
z Transplanting Method: Transplantation is the sowing of seeds which is usually done in
well prepared seed beds call miseries, after 4 to 5 weeks when saplings attain 25 to 30 cm
of height they are transplanted in to prepared rice fields, in a regular distance. Then the

Volume 2 of 4 Universal Tutorials – X ICSE – Geography 13


14
surplus water is drained so that by the time the grains ripen, the field is dry. It is a
labourious tasks.
z Japanese Method: This method is very popular and best method, now a days because as
it yields three times the normal quantity. It involves:
„ The use of better quality seeds of the high yielding varieties (HYV).

„ The sowing of seed in raised nursery beds.


„ Transplantation of the seedlings in equal distant rows. It is easy for applying fertilizers.
„ Irrigation also done regularly during the period of growth
„ Heavy manuring done both in the nursery and the field.

Types of Rice:
z Upland type: grown on terraced fields on the hill slopes at higher elevations.
z Low land type: grown in flat, low–lying areas.
Distribution:
z Tamil Nadu, West Bengal, Orissa, Andhra Pradesh, Bihar, Assam, Punjab, Uttar Pradesh.

Wheat:
 Wheat is the world’s most important food crop for more then 1/3 of the world population winter
but in India it ranks after rice in importance.
Geographical Requirements:
z Temperature: 10°C to 15°C (When growing); 20°C to 25°C (When ripening)
z Rainfall: 50 to 100 cm
z Soil: Well drained clayey, loamy or black soil and alluvial soil also
Methods of Cultivation:
z Wheat sown in October–November after the monsoon rains, when temperatures are low,
the soil is moist and easily ploughed and fertilized.
z The grain appears at the end of January and harvesting is done just before the intense
summer heat sets in at the beginnings of March. Plenty of sunshine is beneficial to ripening
and early harvesting.
Distribution:
z Punjab and Haryana, Uttar Pradesh, Rajasthan, Madhya Pradesh.

Millets:
¾ The tern ‘millets’ refer to a member of inferior grains which serve as food grains for the poorer
sections of the society. Jowar, Bajra, and Ragi are kharif crops. Millets are called dry crops.
Geographical Requirements:
z Temperature: 27°C to 32°C
z Rainfall: 50 cm to 120 cm
z Soil: Inferior alluvial soil or sandy soil.
Jowar, Bajra, Ragi:
 Jowar, Bajra and Ragi is a kharif as well as a rabi crop.

14 Universal Tutorials – X ICSE – Geography Volume 2 of 4


Chapter 09: Food Crops 15

Geographical Requirements:
z Temperature: 27°C to 32°C
z Rainfall: 50 cm 100 cm (Ragi)
z Soil: Dry Soil, clayey loams are the best.
Distribution:
z Uttar Pradesh, Punjab, Haryana, Maharashtra, Gujarat, Madhya Pradesh
Pulses:
z Pulses are enlivened as kharif and as Rabi crops.
Geographical Requirements:
z Temperature: 20°C to 25°C
z Rainfall: 50 cm to 75 cm
z Soil: Dry light soil
Distribution:
z Punjab, Haryana, Uttar Pradesh, Maharashtra, Madhya Pradesh

Volume 2 of 4 Universal Tutorials – X ICSE – Geography 15


16

16
Crop Temperature Rainfall Soil Methods of farming Area and States

Rice (Kharif) 22°C to 32°C 150 cm to 300 cm Alluvial topsoil, Rice can be grown by Northern Plains, Eastern
Most imp. staple Average 24°C clayey broadcasting, dibbling, Coastal plains especially the
food crop of impervious drilling, transplanting and Deltas of Krishna, Kaveri,
When ripening
India subsoil Japanese method of Mahanadi and Godavari. West
cultivation. Transplanting Bengal, Uttar Pradesh, Andhra
method is the best Pradesh, Punjab, Bihar,
Food Crops of India:

Jharkhand and Tamil Nadu are


the leading rice producing
states.

Wheat (Rabi) 10°C to 20°C 50 cm to 100 cm Clayey, loamy It is sown by broadcasting, Uttar Pradesh, Punjab,
Second most when growing (winter rain soils dibbing and drilling Haryana, Madhya Pradesh,
imp. food crop beneficial) methods. In India, Bihar, Chhattisgarh, Rajasthan,
ploughing, sowing Maharashtra, and Gujarat.
25°C to 25°C
harvesting, and threshing
when ripening is generally done by
hands.

Millets: 20 cm to 100 cm Grown on a Mostly by broadcasting Jowar and Bajra: Maharashtra


Jowar (K & R) 27°C to 32°C variety of soils method. Kamataka, Andhra Pradesh,
Bajra but clayey Madhya Pradesh, Jharkhand,
27°C to 32°C
50 cm to 100 cm loams are more Tamil Nadu, Uttar Pradesh,
Ragi (Kharif) 27°C to 32°C suitable. Gujarat, and Rajasthan.

Universal Tutorials – X ICSE – Geography


50 cm to 100 cm Any type of soil Ragi: Karnataka, Tamil Nadu,
is suitable and Andhra Pradesh.
Variety of soils

Pulses Gram 20°C to 25°C 50 cm to 75 cm Dry light soils Dibbling method. Gram: Punjab, Haryana, Uttar
(Rabi) Arhar, Pradesh, Bihar.
and Moong Moong, Arhar: Andhra Pradesh
(Kharif) Masur Maharashtra, and Bihar.
(Rabi and kharif Masur: Madhya Pradesh, Tamil
both) Nadu, and Uttar Pradesh.

Volume 2 of 4
Chapter 09: Food Crops 17

REVIEW QUESTIONS AND ANSWERS

Short Answers: [2 Marks]


1) What is meant by the term Agriculture?
Ans: Agriculture can be defined as the cultivation of the soil in order to grow crops and rear livestock.
2) What is meant by monoculture?
Ans: Monoculture or single crop plantation, only one crop is grown in the whole plantation.
i) It is more common in tropical countries because there is no fear of frost
ii) Tea and rubber plantations in India are its examples.
3) What is known as the zaid season?
Ans: i) Besides Rabi and Kharif crops, the farmers also grow an extra – crop, known as ‘zaid’.
ii) It is grown in the summer months of April May and June, just after the rabi crops are
harvested.
iii) Its main products are seasonal fruits and vegetables.
4) What are the advantages of commercial farming?
Ans: i) In this type of farming mechanised agriculture is practiced and farmers grow crops for the
market.
ii) Maximum yield is produced with the help of irrigation, fertilizers, and scientific methods of
farming and use of hybrid seeds.
iii) This kind of farming is generally practiced in areas of sparse population, where the spacious
land is available for large farms and market economy has taken its roots.
5) What are food crops? Why is it necessary to establish industrial processing units for food
crops?
Ans: i) Since India has to feed a large population, preponderance of food crops over other crops is
the most important feature of Indian agriculture. More than 2/3rd of the total cropped area is
taken up by the cultivation of food crops like rice, wheat, maize, millets.
ii) In order to establish a good foreign exchange from food crops that are surplus, specially after
the Green Revolution, food must be well processed in industrial units so that they can be
preserved and stored.
6) What are Rabi and kharif crops?
Ans: Rabi: This season starts after the rainy season. Sowing begins in October–November and
harvesting takes place in the beginning of summer in March–April. Rabi season is cooler and
drier than the kharif season. Wheat, barley, pulses and some oil–seeds are grown in this
season.
Kharif: The Kharif season begins with the onset of the monsoons in June–July. The crop grows
in the rainy season and harvesting takes place in the beginning of November. Rice, maize,
millets, groundnuts, cotton and jute are the principal crops grown in the Kharif season.
7) Why are the leguminous crops an important component of any cropping system?
Ans: The leguminous crops are important component because it is helpful to maintain fertility of soil
by nitrogen supply. Thus rotation of this crop reduces fertilizer needs.
8) Give Reasons: Drilling is the best method of sowing of seeds
Ans: i) In the drilling method, the seeds are dropped in a straight line at regular intervals through a
bamboo shaft attached to the plough which makes furrows.
ii) This method requires more labour and time but is economical in the use of seed.
Therefore, drilling is considered to be the best method of sowing of seeds.
9) Give Reasons: Jhumming is forbidden.
Ans: i) This method of farming is not an environment friendly method as it involves in deforestation
and burning increases the level of greenhouse gases.
ii) Moreover, the method is wasteful and yield per hectare is also low.
Hence, Jhumming is forbidden.
Volume 2 of 4 Universal Tutorials – X ICSE – Geography 17
18

10) i) Why are millets widely grown?


Ans: a) Millets are used widely used as animal fodder and bird seed.
b) It does not require any extra effort to grow and can grow well even in high temperatures like
in drought conditions. Moreover, it gives an enough yield.
Hence, millets are widely grown.
ii) What is the common feature of all millets?
Ans: The common feature of all millets is that they belong to the grass family.
11) Why is the area under the cultivation of wheat on the increase?
Ans: i) The use of package technology or the Green Revolution in India in late sixties has helped
high yielding varieties of wheat which are drought – resistant and also able to survive wide
fluctuations in rainfall.
ii) Use of chemical fertilizers, irrigation, machinery crop protection measures, and institutional
support also helped in the increase of wheat production.
12) In which type of soils, rice can be grown best?
Ans: Rice can be grown in a variety of soils but alluvial soil with subsoil of impervious clay is ideal of
its cultivation. The impervious subsoil does not allow water to drain away and retains the water
to stagnate in the fields.
13) What are the advantages of transplantation method of sowing rice?
Ans: i) Use of transplantation method increases the yield of rice crops compared to other methods.
ii) In this method, the problem of weeding is reduced because at the time of puddling, weeds
get buried.
14) What are the soil and climatic conditions suitable for the cultivation of grams?
Ans: Soil: Gram can be cultivated in comparatively less fertile alluvial soils. Deep medium black soil is
ideal for its growth.
Temperature and rainfall: Gram is a rabi crop which requires temperatures between 20°C to
25°C during its growth. It is generally grown as a dry crop in areas having moderate rainfall
between 50 cm and 75 cm.
15) How are pulses harvested?
Ans: The pulse crops mature in about 150 days or about five months. When leaves of plants become
dry and begin to shed, these plants are pulled out. They are dried for a few days. Then they are
thrashed by trampling under the feet of bullocks or with sticks to get the seeds.
16) Where is intensive commercial farming done in India?
Ans: Intensive farming is widespread in the irrigated areas of the Northern Plains and the coastal
strips of South India.

Short Answers: [3 Marks]


17) Why is agriculture said to be the backbone of the Indian economy?
Ans: i) Agriculture not only provides food to human beings but also fodder.
ii) It is also the source of raw material for many key industries e.g. sugar, textile and edible oil.
iii) It provides a large part of the market for industrial goods, especially the farm inputs like
fertilizers, pesticides, implements, machinery etc.
iv) It accounts for a substantial portion of India’s exports.
Hence, agriculture said to be the backbone of the Indian economy
18) State two problems faced by Indian agriculture.
Ans: The following problems are faced by the Indian agriculture:

18 Universal Tutorials – X ICSE – Geography Volume 2 of 4


Chapter 09: Food Crops 19
i) More than 60% of the cultivated land depends on the vagaries of the monsoon rainfall, Un
reliable, seasonal and unequal distribution of rainfall is the cause of the failure of rain fed
crops. There are inadequate irrigational facilities in such areas.
ii) Soil erosion due to heavy monsoon rains, floods, gusty winds and insufficient vegetation
cover have increased the infertility of the soil and reduced the yields per hectare.
iii) Due to the illiteracy the farmers are reluctant to use modern scientific method of cultivation.
Many of them do not use new and better quality seeds, fertilizers and pesticides to ensure
good quality crops.
iv) Due to small uneconomic holdings due to farm fragmentation, half of the farms in India are
less than 1 hectare in size and 4 percent are of 10 hectares. They cannot use tractors or
agricultural machines on these small holdings.
v) Farmers are disinterested in improving the land or taking the risk of trying new methods of
cultivation or new varieties of seeds.
vi) The system of land tenure is such that most of the farmers do not own the land cultivated by
them. They belong to absentee landlords who are indifferent to land improvements.
vii) An unsound credit system and the poverty of the farmers are serious problems. They often
suffer the burden of heavy inherited debts. They do not have capital to invest in
improvements.
viii) Indian agriculture is mainly of subsistence type. About 72% of the total area is devoted to
food crop to meet the requirements of the country. There is no surplus to export.
xi) Average productivity of crops is low due to outdated farming practices, less use of manures
and pesticides and low quality seeds.
19) In which region is Ragi grown in Indian? Why?
Ans: Ragi is mainly grown in Peninsular India, in the southern states of Karnataka, Tamil Nadu and
Andhra Pradesh.
z It is both a kharif and a rabi crop.
z Ragi grows in areas of less rainfall. It is grown either with dry–farming method or as an
irrigated crop.
z It grows on a variety of soils of Peninsular India.
20) With reference to jhumming, answer the following question:
i) How is jhumming carried out?
Ans: In this type of farming there is continuous shifting from one clearing in the forest to the other due
to loss of soil fertility.
ii) Where is it practiced in India?
Ans: It is practiced in the North east hill states, Madhya Pradesh, Chhattisgarh, Jharkhand.
iii) What are its disadvantages?
Ans: It is not an environment – friendly method as it involves deforestation and burning increases the
level of greenhouse gases. It is a health hazard and produces low crop yield.
21) With reference to the types of farming in India, answer the following question:
i) Give the meaning of the term plantation farming.
Ans: Plantations are large tracts of land used for cultivation of a single agricultural crop like tea,
coffee, rubber or spices on a large scale.
ii) Who introduced plantation farming to India?
Ans: The European colonists introduced plantation farming to India.
iii) What are the advantages of this type of farming?
Ans: i) The plantations usually cater to the export – market and earn foreign exchange
ii) It involves large number of labourers like managerial labour, daily wage labourers; hence it
provides employment opportunities to a large section of the population.
22) Differentiate between the following:

Volume 2 of 4 Universal Tutorials – X ICSE – Geography 19


20
Ans: Subsistence farming and Commercial farming.
Subsistence farming Commercial farming
Subsistence farming is practiced in the This kind of farming is practiced for the sale
densely populated regions of the world for the of the crops in the market. Generally
sole purpose of the farmer’s subsistence. practiced in areas of sparse population
There is no surplus left for sale. where holdings are large.
Farms are small and scattered and the yields Farms are large and yield is very high.
are not large.
This kind of farming may be intensive This kind of farming may be intensive or
primitive. There is little use of modern tools extensive. Latest knowledge and modern
and implements. methods of agriculture are used.
23) Intensive farming and Extensive farming.
Ans:
Intensive farming Extensive farming
In this type of farming, farmers raise more In this type of farming, the farmer
than one crop from the same field by making specializes in a couple of major commercial
use of irrigation facilities. crops with the help of machines.
The land is limited, so the farmer uses it The farms are huge, so no extra care is
intensively in order to have the maximum needed to maintain the soil–fertility.
output.
Intensive labour, rich manure and fertilizers, Mechanisation in farming is a unique feature
quality seeds and water supply through with the absence of human and animal
irrigation are used to maintain the high yield labour. There are bumper crops.
Intensive farming is widespread in the Extensive farming is practiced in the Terai
irrigated areas of the Northern Plains and the region of the Sub–Himalayas and in parts of
coastal strips of South India. Rice and wheat North Western India. Wheat, rice, sugarcane
are main crops. are the main crops.
24) i) What is meant by fallow land?
Ans: Fallow land: The much used land is often allowed to rest or lie fallow for period of time. This
piece of land is called fallow land.
ii) State one advantage and one disadvantage of leaving land follow, so that the natural
forces could act on the soil.
Ans: Advantage of fallow land: The decayed natural vegetative matters helps to increase the plant
nutrition in the soil.
Disadvantage: The marginal farmers can’t afford to get more production by leaving the land to
rest.
25) Why is agriculture called the mainstay of the Indian economy?
Ans: i) Agriculture is the main stay of Indian economy. Between 67% of our population depends
directly or indirectly on agriculture.
ii) It provides raw material to the industries.
iii) India earns foreign exchange by exporting agriculture products.
iv) It contributes 30% to the national income.
26) What is mixed farming? Mention advantages of this farming to the small farmers.
Ans: Mixed farming is a system of farming on a particular farm to sustain and satisfy the essential
needs of the farmers. It includes rearing of live stock and poultry, fish and bee keeping.
Advantages:
i) The small and marginal farmers in rainfed regions cannot take the risk of growing specialized
crop.
ii) In this farming equal importance is given animal rearing.
iii) Farmer remains busy in their work throughout the year. It supplements the farmer’s earning in
the lean season.
27) Differentiate between Rabi, Kharif and Zaid crops. Give an example of each.

20 Universal Tutorials – X ICSE – Geography Volume 2 of 4


Chapter 09: Food Crops 21
Ans:
Rabi crops Kharif crops Zaid crops
Winter crop Summer crop Extra crop
Rabi crops are sown in the They are sown at the It is grown in the summer
winter season (October– beginning of the rainy season months of April, May and
November) and harvested in in June and harvested in the June after the rabi season.
the beginning of summer autumn season in the
(March–April). beginning of November.
The main crops are wheat, The principal kharif crops are Its main products are
barley, gram, linseed, tobacco rice, maize, millets, cotton, seasonal fruits and
and mustard. jute, castor, groundnut and vegetables.
sesamum.
28) Why is wheat growing important in Punjab?
Ans: Although Punjab is not the biggest producer or wheat in India, yet wheat is the most important
crop of Punjab. The favourable factors are as follows:
i) It is region of alluvial soil deposited by the rivers of Punjab
ii) Punjab has ideal climatic conditions for growing wheat.
iii) Water requirements are supplemented by network of irrigation canals, which are fed by
Bhakra Nangal Project, Harike project and Beas project.
iv) Rain from Western Disturbances is a boon for wheat crop in Punjab.
v) The Punjab’s farmers are very laborious. Large holdings and improved techniques are
favourable for wheat growing.
29) Give two reasons why the growing of pulses is important in India.
Ans: Owing to following reasons the growing of pulses is important in India:
i) Pulses include a very important part of the Indian–diet particularly for the vegetarian
population because of protein content of the pulses.
ii) They are leguminous plants. They help in restoring the soil fertility, so they are used as
rotating crop.
iii) They are dry crops and required less water.
iv) They serve as excellent fodder for cattle.
a) The geographical conditions in Punjab are most suitable for growing wheat.
Ans: Punjab experiences following climatic conditions:
i) Punjab receives winter rain due to western disturbances which is beneficial to the wheat crop.
ii) The draining of perennial rivers makes the soil clayey, loamy, and well drained helping the
wheat crop to grow well.
iii) In winters, the state experiences 10° to 15° C of temperature which favours the wheat crop in
ripening. Even in the mid-March, April-mid the temperature ranges 20°C to 25°C which again
is ideal for wheat cultivation.
b) Millets and pulses are called ‘Dry Crops’.
Ans: i) These crops mature early and are drought resistant crops.
ii) They are and therefore cultivated in the drier parts of monsoon lands, particularly the Deccan
Plateau.
iii) Hence they are called as ‘Dry Crops’.
c) Which is a useful ‘rotation crop’? Why?
Ans: i) Pulses form an important part of the Indian diet and are grown in the Rabi season. Pulses
include a number of crops (dals) which provide protein. They are leguminous plants with root
nodules which have the capacity to fix and use atmospheric nitrogen in the soil. They utilize
nitrogen from air.

Volume 2 of 4 Universal Tutorials – X ICSE – Geography 21


22
ii) The crops are rotated with other crops to maintain or restore soil fertility. They act as fertilizer
to the soil.
iii) Pulses serve as an excellent forage (food for cattle) and grain concentration in the feed of
cattle. It also supplies protein in a vegetarian diet as there is an absence of animal protein in
a vegetarian diet.
f) Rice is not the main crop in the Deccan Plateau.
Ans: Rice is not the main crop in the Deccan Plateau due to the following reasons:
i) Rice grows best in heavy loam and alluvial soil with sub–soil of impervious clay which allows
water to stagnate in the fields. Deccan plateau has mainly black cotton soil or red soil, which
are not suitable for growing rice.
ii) Rice needs warm or hot humid climate, with temperature between 18°C to 35°C. Deccan
plateau has a hot dry climate with little moisture in the air and temperature above 30–35°C.
iii) Rice needs abundant rainfall ranging between 150 cm–300 cm. It requires 5–10 in of
standing water during the early part of the growing season.
The Deccan Plateau region receives an average of 50–100 of rainfall, which is inadequate for
growing rice. Irrigation facilities are also not widely available.
Due to the terrain, vast tracts of flat land are not available for water to stand in the fields.
Hence rice cannot be widely grown on the Deccan Plateau.
30) Give three main aspects of the Japanese method of rice cultivation.
Ans: The three aspects of the Japanese method of rice cultivation are:
i) Use of high yielding paddy hybrids called JAPONICA, which increases output tremendously.
ii) Sowing of seeds in raised nursery beds.
iii) Transplantation of seedlings in rows. This facilitates weeding and fertilizing.
iv) Irrigation is done to ensure the required supply of water during the period of growth.
v) Heavy manuring done both in the nursery and the field.
31) For wheat in Punjab:
i) Give two natural factors and one man–made factor that favour the cultivation of wheat.
Ans: The two factors favouring the cultivation of wheat are as follows:
i) The winter rain due to western disturbances received in Punjab helps the wheat crop for
speedy germination.
ii) The state has clayey, loamy and well drained soils owing to the flow of various river channels.
iii) The man–made factor, favouring the cultivation of wheat includes introduction of Green
Revolution in 1967–68. The high–yielding Mexican wheat and hybrids led to the maximum
yield in acreage.
32) Why millets are called “dry crops”? Why is bajra grown in Rajasthan and jowar in
Maharashtra?
Ans: i) Millets are called “dry crops”. This is common name for several species of the grass family.
They can be grown on lands which are not suitable for rice and wheat production because of
deficient rainfall and poor soil. They can be grown in areas of high temperature ranging from
27°C to 32°C. They can survive in high heat and drought conditions, in rainfall from 50 to 120
cms and in medium to deep black soils. They are grown in the drier parts.
ii) There are three types of millets in India – jowar, bajra and ragi.
Jowar is grown in Maharashtra as they can be grown in deep black soils. They take only
three months to mature, whereas other crops take five months. Sowing is done by the
broadcasting method. After harvest, the stalks are used as cattle fodder, and the grain is
used as food.
iii) Bajra can be grown in arid conditions with less rainfall, it grows on sandy soil as in Rajasthan.
The sowing is done in the middle of June and harvested in November. When the harvest is
ready, the heads with grain are cut by hand. The plant is given for cattle fodder and grain
from the heads is separated by threshing, on boards or floors.

22 Universal Tutorials – X ICSE – Geography Volume 2 of 4


Chapter 09: Food Crops 23

33) Compare and contrast the geographical conditions required for the growth of the staple
food crops of India.
Ans: Two staple food crops of India are wheat and rice.
i) Wheat is sown in winter (rabi), whereas rice is sown in summer (Kharif).
ii) Wheat requires cool and moderate climate, whereas rice requires hot and humid climate.
iii) Wheat requires less than 18°C temperature and medium rainfall of 50 cm to 100 cm, whereas
rice requires a temperature from 25°C to 30°C and rainfall more than 100 cm.
34) Mention the climatic conditions required for rice cultivation in India
Ans: Rice is a crop of hot and wet regions in the tropics.
Temperature: Rice plant requires temperatures ranging between 16°C and 20°C during the
growing season and 18°C to 32°C during the harvesting season. It requires plenty of water and
bright sunshine.
Rainfall: Rice plant requires good rainfall ranging between 100 cm and 200 cm. it requires
flooded fields at the time of early growth and during transplantation.
Frequent showers before ripening helps in increasing the size of the grain. Rice can be grown in
areas with less rainfall, if the facility of irrigation is available.
35) With reference to wheat: a) During which part of the year is it grown in India and why?
Ans: Wheat is grown in the winter in India because it requires cool and moist weather with a
temperature of less than 20°C during its growth. Warm and dry climate is good for its ripening. It
also requires 200 frost free days.
b) Why is it not grown in the southern and eastern parts of India?
Ans: Wheat is a Rabi crop in India. It grows in areas where the temperature does not rise beyond
20°C in winter during growing period.
Hence, wheat cannot be cultivated in the eastern and southern parts of India.
36) With reference to the type of soil only, state why
a) Bajra is grown in Rajasthan?
Ans: Bajra is grown in Rajasthan because it needs sandy soil and dry climate.
b) Jowar is grown in Maharashtra.
Ans: The Deccan plateau in Maharashtra has drier black soil hence Jowar is grown here.
c) Ragi is grown in Tamil Nadu.
Ans: It is grown in Tamil Nadu since it requires less rainfall and can be either grown with the dry
farming or as an irrigated crop. Ragi can grow on a variety of dry soils.
37) State the differences between cultivation of upland rice and low – land rice.
Ans:
Upland Rice Lowland Rice
This type of rice is grown in terraced field on This type of rice is grown in flat low lying
hill slopes. areas.
It is cultivated without irrigation. It needs a lot of irrigation.
It is grown in March – April and harvested in It is sown in June and harvested in October
September – October. and November
This crop is used for local consumption. Most of the rice grown in India is low land
variety.
38) Distinguish between transplantation method and Japanese method of rice farming.
Ans: Transplantation Method and the Japanese Method of Rice Farming:
Transplantation Method Japanese Method
In transplantation method of rice farming, first In Japanese method of rice farming, first the
the seeds are sown in small and well prepared seeds are sown in already enriched and
plots. well–drained nursery beds.
About 15 cm high saplings are transplanted at About 15 cm to 20 cm high saplings are
regular intervals in other fields. transplanted in rows in another field.
Harvesting is carried out when ears are nearly Top–dressing with nitrogenous fertilizers is
ripe. done before flowering of the plants.
Volume 2 of 4 Universal Tutorials – X ICSE – Geography 23
24

39) What improved methods of cultivation can raise rice production in India?
Ans: Japanese Method: A popular method known as the Japanese method of rice cultivation is being
popularized in India in order to increase the productivity of rice.
In this method
a) The good quality seeds are sown in well-drained nursery beds already enriched with manure.
The sapplings are then uprooted when they grow to 15 to 20 cm in height and replanted in
rows in another field.
b) Huge amount of compost manure is used in the field before transplanting. If the green
manure is used, the mixture of ammonium-sulphate and superphosphate fertilizers are used.
The mixture is used again after one month of transplanting.
c) Planting is done in rows at appropriate distance and then irrigation is provided as needed.
Before flowering of the rice plants, top-dressing is done with nitrogenous fertilizers.
40) State the advantages of growing rice in nurseries. Explain the method.
Ans: Transplanting Method:
a) In this method, first the seedlings are sown in small, carefully, prepared plots. When the
plants are about 15 cm high, they are uprooted by hand in small bunches. Then they are
transplanted at regular intervals in fields.
b) The fields, as the seedlings grow, are watered well. The water is supplied abundantly till the
plants become mature; but about two weeks before harvesting, the water in the field is
drained off.
c) Harvesting is done when ears are nearly ripe. This method, though labour intensive, gives
better yields.
41) In West Bengal which three crops of rice are grown in a year? Explain.
Ans: The three crops of rice grown in West Bengal are as follows:
i) Aus (autumn) crop is sown in May-June and harvested in September-October.
ii) Aman (winter) is sown in June-July and harvested in November-December. This crop
accounts for 85% of the rice produced in West Bengal.
iii) Boro (summer) is sown in November-December and harvested in March-April. This crop is
grown in low-lying marshy land of poor quality.
42) Why yield of rice is very low in India as compared to other countries of the world?
Ans: Currently, the average yield of rice per hectare is 1756 kg. It is the lowest in the world. The per
hectare yield of rice in Japan, China and Korea is about three times that of India. This is
because:
i) Rainfall in India is uncertain and is concentrated to four months.
ii) There is a lack of assured water supply through irrigation in rice growing areas.
iii) General fertility of Indian soils is less because these have been cultivated from the last 5000
years.
iv) Farmers do not use efficient methods of cultivation.
v) Holdings are very small.
vi) Widespread poverty in the rural areas prohibits the use of adequate amount of fertilizers,
pesticides, etc.
43) Discuss briefly different methods of rice cultivation.
Ans: Rice can be sown in the following five ways:
i) Broadcasting: It is simply scattering or throwing seeds over the soil by hand. This method is
practised in areas where labour is scarce and soil is not much fertile.
ii) Dibbling: In this method, seeds are dropped at regular intervals in ploughed furrows. This
method of rice sowing is used in Northern Plains of India.
iii) Drilling: In this method, seeds are dropped through shafts of bamboo which is attached to
the plough. In this way, seeds fall in straight line. Now-a- days, drilling machines are used in
place of bamboo.

24 Universal Tutorials – X ICSE – Geography Volume 2 of 4


Chapter 09: Food Crops 25
iv) Transplantation: In this method, first seeds are sown in the nurseries. When they become
15 cm to 20 cm tall after about four weeks of time, they are uprooted and planted again in the
flooded fields. This transplantation is done mainly by human labour.
v) Japanese method:
a) In India, the Japanese method of rice cultivation is becoming popular because this method
gives three times higher yield of rice than any other method.
b) In this method, better quality seeds are used.
c) Seeds are sown in raised nursery beds.
d) Transplanting is done in rows at regular intervals. This helps in weeding and fertilising.
e) In this method, abundant application of fertilizers is done at proper stage of the growth of
the crop.
44) What is the importance of agriculture in India?
Ans: i) Agriculture provides raw materials for agro–based industries such as raw cotton for cotton
textiles Jute, Sugar, tobacco are also important agro –industries. These provide employment
to many and help to develop the rural area as well.
ii) Commodities such as tea, cotton and jute textiles, tobacco, cashew nuts, oilseeds and spices
are exported to industrialized countries and consequently are great foreign exchange
earners.
iii) Agriculture products, being a source of income, prevent an unnecessary shift of population
from rural to urban areas.
iv) Indian agriculture has two outstanding features i.e. wide variety of tropical and temperate
crops are grown.
v) Intensive and extensive cultivation both are practiced but the former is more important.
In this way, the importance of agriculture to India can be assessed.

Volume 2 of 4 Universal Tutorials – X ICSE – Geography 25


26

Chapter 10: Commercial or Cash Crops &


Distribution of Crops and Their Requirement
→ Commercial or Cash Crops
→ Definition
→ Types of Cash Crops
→ Oil Seeds: (Groundnut, Sesame, Linseed, Castor Seed,
Soyabean or Rape seed or Mustard)
→ Plantation: (Rubber, Sugarcane) Beverages, (Tea, Coffee)
→ Fiber: (Cotton, Jute, Silk, Wool)

Commercial or Cash Crop:


Definition:
 Cash crops are those that are primarily grown for sale and not for use by the grower and his
family for example: Sugarcane

Types of Cash Crops:


 Oil Seeds: Vegetable oils are produced from a large member of crops. All the principal oil,
Seeds. Groundnut, sesame, linseed, castor, soyabean sunflower, rape seed or mustard etc.
are grown in India.
Ground Nut (Kharif):
Origin:
„ Brazil - 16C
Geographical conditions:
„ Temperature: 20°C to 25°C
„ Rainfall: 50 cm to 100 cm
„ Soil: Dry, Light and Sandy soil, loamy, well drained.
Method:
„ Broadcasting: drilling.
Uses:
„ Nuts are eaten raw or roasted
„ As a cooking medium, manufacturing of soap, hydrogenated oil.
„ As a fodder for animals.
Area:
„ Gujrat, Maharashtra, Andhra Pradesh, Rajasthan and Punjab.

26 Universal Tutorials – X ICSE – Geography Volume 2 of 4


Chapter 10: Commercial or Cash Crops … 27

Sesame (Karif / Rabi):


Position: 1st position.
Geographical conditions:
„ Temperature: 21°C
„ Rainfall: 40 cm to 60 cm
„ Soil: Light, Sandy soil, Black soil
Uses:
„ As a cooking oil, medicinal, perfume, foder
Area:
„ Gujrat, Andhra Pradesh, Tamil Nadu, Karnataka, Maharashtra
Linseed (Rabi):
Geographical conditions:
„ Temperature: 15°C to 20°C
„ Rainfall: 45 cm to 75 cm
„ Soil: Clayey loom mainly / Alluvial
Uses:
„ It is used for making paints varnishing, Printing, ink, seasoning of wood.
„ Oil cake for cattle, as a manure.
Area:
„ Madhya Pradesh, Chhattisgarh, Uttar Pradesh, Rajasthan and Maharashtra.
Sunflower (Karif / Rabi):
Polyunsaturated fatty acid.
„ Temperature: 26°C to 30°C
„ Rainfall: less than 50 cm
„ Soil: Alluvial and black soil.
„ Area: Bihar, Maharashtra, Andhra Pradesh, Karnataka
„ Use: Oil cake for Animals, cooking oil–(good for heart)
Castor Seed (Karif):
Geographical conditions:
„ Temperature: 20°C to 25°C
„ Rainfall: 50 cm to 75 cm
„ Soil: Deep loamy Soil, Alluvial soil, Red soil
Uses:
„ Caster seed is used in points, varnishing, ink and plastics and for medicinal purposes
„ Used as a manure, as well as poison.
Area:
„ Andhra Pradesh, Gujrat, Karnataka
Soyabean (Karif): Protain sources
Geographical conditions:
„ Temperature: 13°C to 24°C

Volume 2 of 4 Universal Tutorials – X ICSE – Geography 27


28
„ Rainfall: 40 cm to 60 cm
„ Soil: Any type of soil but moisture retentive, Acidic soil needs irrigation
Uses:
„As a vegetable oil, sauce, milk, margarine, soaps, lubricants, artificial meat.
Areas:
„ Maharashtra, Uttar Pradesh, Gujrat, Madhya Pradesh.

Rape Seed or Mustard (Rabi):


Geographical conditions:
„ Temperature: 10°C to 20°C
„ Rainfall: 25 cm to 40 cm
„ Soil: Alluvial (heavy and light)
Uses:
„Mustard is used for cooking oil, seasoning pickles. Oil cake is used for cattle feed and
fertilizers.
Areas:
„ Uttar Pradesh, West Bengal, Punjab, Assam, Bihar, Jharkhand, Orissa.

Plantation crops:
Rubber: 4th position
z Rubber is the coagulated sticky milk or latex of the rubber tree, a native of the Amazon
forest, and is known as Hevea brasiliensis.
z It holds air, keeps out moisture and does not readily conduct electricity. But its chief
importance is that it is elastic and therefore useful for water–proofing, insulation and where
a strong, elastic substance is required.
Geographical conditions:
„Temperature: Between 25°C and 35°C. It should never fall below 21°C.
„Rainfall: 150 cm to 200 cm, well–distributed throughout the year.
„ Soil: Alluvial, deep friable, rich, well–drained soils or laterite soils, red in parts.
Tapping of Rubber:
„ Tapping is normally done daily in the early hours of the morning before 6 a.m. as latex
flows freely then, as also to avoid likely rain and heat of the sun. It is stopped by
noon and suspended in January, February and during the rains. Trees are tapped
for about 200 to 300 days.
„ A tapping panel is made at V shape, by removing the outer bark.
„ Care is taken not to damage the cambian–the paper–like skin exposed after the removal
of the bark–when a cut, about 2 mm, is made for the latex to flow. The latex drips and
collects in the coconut shells attached to the tree.
Processing of Rubber:
„ At the factory, the latex is strained to remove the impurities, weighed, coagulated into
coagulum (soft spongy blocks) by the addition of acetic acid and formic acid. Sometimes
part of the fresh latex is put into machines called separators which remove part of the
water from the latex. Ammonia or some other preservative keeps the latex from
coagulating and prevents spoiling. The liquid latex is marketed in drums or tanks.
„ It is then passed through a series of rollers to squeeze the surplus water and if
necessary the sheets are hung up to dry. The final drying of the rubber sheets is done in

28 Universal Tutorials – X ICSE – Geography Volume 2 of 4


Chapter 10: Commercial or Cash Crops … 29

smoke houses where the temperatures vary from 45°C to 60°C. The rubber sheets are
then graded packed in bales and exported.
Area:
„ Kerala, Tamil Nadu, Karnataka, Andaman and Nicobar Islands
Sugarcane:
z Sugar cane is a tall, perennial, tropical and semitropical grass, the stem of which is used to
produce can sugar and sucrose.
Geographical conditions:
„ Temperature: 20°C to 30°C. Sugar cane is sown between January and April. Frost is
injurious to the growth of sugar cane plant.
„ Rainfall: 100 cm to 150 cm plenty of water during the growing period is required.
Irrigation may be necessary as the growing season is usually 9 months. However, water
logging has to be avoided as it reduces the sugar content.
Soil:
„ Soil should be alluvial or lava soil and well drained. Sugar cane is soil–exhausting so the
soil regularly needs nitrogen fertilizers, labour intensive crops.
Sowing Methods:
„ By seeding which are sown between January and April.
„ The sett method consists of cutting of two or three joints taken from the top of a healthy
cane, planted in horizontal, equidistant (1.5 m to 2 m apart) furrows, and piled with
fertilized soil. Within two weeks the buds on the nodes swell and burst and young stalks
emerge from the soil. The leaves appear soon after, followed by the appearance of
nodes and internodes.
„ Ratoon cropping is the process by which the cane is cut close to the ground where the
sugar content is concentrated. After the crop has been cut the stem begins to grow
again and produces a second crop called ratoon.
„ Process: Boil with line then it will form crystal brown sugar.
Products and By Products:
„ Sugar, Sugar–cane juice, Khandsari–powdered gur or jaggery and Icing sugar.
„ Molasses: It is a kind of dark brown syrup)
„ Bagasse: Rejected cane–after being crushed – is ploughed into the soil fed to cattle and
used as fuel for mills.
„ Pressmud: This is used to make wax, shoe polish and carbon paper, gur, khandsari
Area:
„ Uttar Pradesh, Maharashtra, Karnataka, Andhra Pradesh, Tamil Nadu.

Beverages (Tea):
z It is a beverage crop and a plantation crop. India is the largest producer, consumer and
exporter of Black tea.
Geographical conditions:
„Temperature: 24°C to 30°C but 25°C is most suitable
„ Rainfall: 150 cm 375 cm
„ Soil: The soil must be well drained and rich in nitrogen sandy loams are the best.
There are three main kinds of tea:
„ Black

„ Green

Volume 2 of 4 Universal Tutorials – X ICSE – Geography 29


30
„ Oolong tea
Processing Black of Tea:
„ Withering: The gathered leaves are first withered or dried in the sun for a day or two to
extract moisture.
„ Rolling: This twists the leaf to break its cells, exposing their juice to the air for
fermentation which helps to give tea its special flavor.
„ Fermentation: The leaves are allowed to ferment and this reduces the amount of tanmic
acid in the tea by half, but does not impair its flavour
„ Drying: Leaves are placed on conveyor belts in an over to dry them and give them black
colour.
„ Blending: Experts blenders and tea testers further blend the various grades of tea to
give it special aroma and make many proprietary brands.
Area:
„ Assam (Brahmaputra Valley and Surma Valley)
„ West Bengal (Duars and Darjeeling)
„ Tamil Nadu (Nilgiri)
„ Kerala

Coffee:
z Introduced by Muslim fakir (Bababudan hills in 17 C. from Arabia)
z Coffee is a leading tropical commodity in international trade and is the most abundantly
produced of the non–alcoholic beverages.
Geographical conditions:
„Temperature: 30° C and above
„ Rainfall: 150 cm to 200 cm well distributed throughout the year.
„ Soil: Must be black lava, well drained, with humus; volcanic soils, red and laterite are
ideal.
Types of Coffee:
„ Arabica: which is a superior quality
„ Robusta: which is the cheapest
„ Liberica: mainly used to make instant coffee.
Special processing Methods:
„ Wet Method or Parchment Method: This method is used for parchment of coffee which
consists of removal of the skin, pulping, fermenting, washing and drying. Nowadays,
there are mechanized processes on coffee estates.
„ Dry Method or Native Method: The covering is removed by drying in the sun. Seeds
are pounded to remove the outer covering. The colour, flavour and taste of coffee are
the result of roasting.
Areas:
„ Karnataka, Tamil Nadu and Kerala are three main commercial coffee producing states
(Nilgiri, Cardamom, Palni and Anaimalai Hills)
Fiber crops: (Cash) 4th rank
z Cotton: Cotton is the most important fibre used in making clothing. China is the world’s
leading producer of cotton (18.8 million bales in 199–99) followed by USA)
Geographical conditions:
„ Temperature: 21°C to 30°C and at least 200 frost free days.
30 Universal Tutorials – X ICSE – Geography Volume 2 of 4
Chapter 10: Commercial or Cash Crops … 31
„ Rainfall: 50 cm to 75 cm, well distributed during the period of growth. If Rabi, it requires
irrigation and has to be protected against ball weevils. Abundant sunshine is required
during ripening and picking. (200 frost free day to mature the crops)
Cotton grade:
„ Cotton is graded according to the length of the staple, which develops in the cotton ball.
The longer staples from sturdier threads and hence are considered superior.
Short Staple – less than 2.2 cm (24 mm)
Medium Staple – 2.2 to 2.8 cm (26 mm)
Long Staple – above 2.8 cm (27 mm)
Ginning:
„ Ginning is the separation of seeds and short fibres or linters from the raw material fibres
or linters from the raw material fibre called lint. After ginning, lint is packed in bales and
sent to factories to be made into thread.
Area:
„ Gujrat, Maharashtra, Punjab, Andhra Pradesh, Madhya Pradesh, Tamil Nadu, Uttar
Pradesh (Producing long staple)
Problems of Cotton Cultivation:
„ Frost destroys the plant, so at least 200 frost free days are necessary for the crop to
grow.
„ Rain should be experienced only during the earlier part of the growing period. Otherwise
the fibre becomes muddy and discoloured.
„ Cloudy weather is detrimental to the healthy growth of cotton plants.
„ Abundant cheap skilled labour is required for picking, pest control, etc. Most of these
processes in India are carried out manually.
„ A beetle called boll weevil lays eggs inside the boll and destroys the crop. A single pair
of boll weevils can produce between 10 to 13 million young ones in a single season and
they are extremely difficult to exterminate, so the infected plants have to be destroyed.
The other pests are pink boll worms and wilt.
„ Storms and windy weather are harmful to the cotton pods as they fall off.
„ Sunshine during picking time is essential to ripen the pods.

Jute:
z Jute is a long, soft, shiny fibre that can be spun into coarse strong threads. Both the fibre
and the plant from which it comes are commonly called jute. Jute is known as the golden
fibre because of its colour and value as a foreign exchange earner.
z Types of Jute: White (75%), Tossa (25%)
Geographical conditions:
„ Temperature: The temperature should be 24°C to 35°C o have an average of 27°C
during the period of growth.
„ Humidity: 90%
„ Rainfall: 150 cm to 200 cm
„ Soil: A rich, loamy, alluvial soil formed in delta regions with comparatively flat
topography is ideal
Area:
„ West Bengal, Bihar, Orissa, Assam, Tripura, Terai region, of Uttar Pradesh.
„ Processing: Retting is followed by rinsing, washing, cleaning and drying in the sun. It is
then graded and made into bundles which are pressed into bales.

Volume 2 of 4 Universal Tutorials – X ICSE – Geography 31


32
„ Method: Drilling and Broad Casting.
„ Uses: It is used for making cloth to wrap bales of cotton, gunny bags, sack cloth, rope,
string, carpets, fibres and twine. Now it is also used for making upholstery, furnishing
material, shopping bags and sail–cloth.
„ Mesta is a substitute for jute. It is a coarser fibre, inferior to jute in quality and strength
and is used for making gunny bags. It tolerates drier condition and is grown in more or
less the same areas

32 Universal Tutorials – X ICSE – Geography Volume 2 of 4


Chapter 10: Commercial or Cash Crops … 33

Synopsis: Important Cash Crops of India


Oil–Seeds of India:
Crop Temp. Rainfall Soil Area Uses
Groundnuts 20°C to 50 cm to Dry and light sandy loam Gujarat, Tamil Nadu, Andhra Groundnut can be eaten raw, roasted
28°C 75 cm Pradesh, Maharashtra, salted or sweetened. Groundnut oil is
Karnataka, Andhra Pradesh used for manufacturing soap, oil
Madhya Pradesh, Chhattisgarh, requisites, artificial leather and
Uttar Pradesh, Uttaranchal, medicinal emulsions.
Rajasthan and Punjab.
Sesamum (til 22°C to 40 cm to Light, sandy soils or black Gujarat, Andhra Pradesh, Tamil Oil is used for cooking purpose and
or gingelly) (K 28°C 70 cm cotton soils Nadu, Karnataka, Maharashtra, pickles, cosmetic purposes, for
&R) Madhya Pradesh, Chhattisgarh, manufacturing perfumes and medicines.
Uttar Pradesh, Uttaranchal and Oil–cakes are led to mulch cattle and
Rajasthan pigs
Linseed (not 10°C to 45 cm to Clayey loamy soils Madhya Pradesh, Chhattisgarh, Used in paints and varnishes, oil cloth
edible) (R) 30°C 75 cm Uttar Pradesh, Uttaranchal, and printing ink.
Bihar, Jharkhand, Rajasthan,
Maharashtra, Gujarat,
Karnataka, Tamil Nadu and
Punjab
Castor seed 20°C to No specific Deep loamy soil Andhra Pradesh, Gujarat, Castor oil is used in paints, varnishes
(Medicinal) (K 30°C amount is Maharashtra, Bihar, Jharkhand, oil–cloth, plastics, medicines and
& R) required Karnataka, Tamil Nadu, Orissa, lubricants.
West Bengal, Uttar Pradesh and
Uttaranchal
Soyabeans (K 20°C to 50 cm to Any type of soil Maharashtra, Uttar Pradesh, Being a high protein content, soyabean
& R) 30°C 100 Uttaranchal, Gujarat, Madhya, may be eaten or made into soya–
Pradesh, Chhattisgarh sauce.
It is used as a substitute for non
vegetarian foods.
Soyabean oil is used making oil and
other edible products.
Rape (Sarson) 10°C to 50 cm to Alluvial soils Uttar Pradesh, Uttaranchal, Used as edible oil, oil–cakes are used
Mustard (Rai) 32°C 100 cm Haryana, Rajashthan, Punjab, as an important cattle feed. Leaves are
(R) Assam, Bihar, Jharkhand, used as vegetable, especially sarson.
Orissa and Maharashtra
Sunflower 21°C to 50 cm to Regur soil preferable Maharashtra, Punjab, Haryana, Used for cooking oil, vanaspati, soaps
seeds (K) 32°C 100 cm Madhya Pradesh, Chhattisgarh, and cosmetics. Residue–cakes are
(Average Andhra Pradesh, Tamil Nadu used as cattle feed.
25°C)
Coconuts 20°C to 150 cm to Loose, prous, sandy soils Punjab, Kerala (75%), Andhra Oil is used as edible oil. Cosmetic hair
(Generally K) 27°C 300 cm along the coast or alluvial Pradesh, Tamil Nadu, oil. Fibre is used as coir leaves and
flat lands or seaside of Karnataka, Maharashtra, trunk for house construction.
tropical lands Gujarat, Orissa and West
Bengal, Andaman and Nicobar
Islands
Rubber 21°C to 175 cm to Alluvial soil, deep soil with Kerala, Tamil Nadu, Karnataka, For water–proofing, insulation
35°C 300 cm good drainage Andaman and Nicobar Islands
Average
25°C
Sugarcane 20°C to 100 cm to Well–drained rich alluvial Uttar Pradesh, Maharashtra, Of the total production, 50% is
30°C 170 cm lava soil it is a soil– Karnataka, Punjab, Andhra processed into jaggery (gur), 30% into
exhausting plant and Pradesh, Tamil Nadu, and white sugar and 20% into khandsari.
requires water during most Haryana. Concentration of sugar industries in the
of its growing season. southern states in increasing.

Volume 2 of 4 Universal Tutorials – X ICSE – Geography 33


34

Beverages Crops:
Crop: Tea
Temp. Rainfall Soil Methods of Farming
10°C to 35°C (Average 150 cm to 250 cm Light, friable loams with a little iron and porous sub–soils By sowing seeds in germinating beds.
of 25°C is ideal). Frost (distributed which allow water to percolate are ideal (stagnant water Another method is to use cuttings from
ins injurious throughout the year) near the roots is harmful, hence grown on hill slopes) high–yielding plants, clonal planting.
Area Processing Varieties and Uses
The Assam Hills region, West There are five stages of processing black tea: There are three varieties:
Bengal (Darjeeling), Himachal 1) Withering: Tea leaves are spread on racks for 15 to 20 hours. Black tea, Green tea and
Pradesh, Uttaranchal, Bihar, Oolong tea, India is the
2) Rolling: Tea leaves are pressed in rollers to break their cells.
Tripura, Manipur, Arunachal largest producer of black
Pradesh, the Nilgiris and 3) Fermentation: Tannin in tea is oxidized and colour changes and flavour develops. tea in the world.
Anaimalai Hills in the South 4) Firing or Drying: Fermented leaves are passed through ovens over a belt.
5) Cutting and packing: It is the last stage.
Crop: Coffee
Temp. Rainfall Soil Methods of Farming
18°C to 28°C. Sensitive 125 cm to 250 cm Well–drained subsoil, weathered volcanic, Seeds are sown in nurseries and then
to cold and frost well–distributed Loamy soil with humus content; well drained transplanted on coffee estates. Young plants are
throughout the year. hill slopes, between 450 m to 1800 m sheltered from the sun by shady trees. Regular
altitude are suitable. pruning is must.
Area Processing Varieties and Uses
Karnataka, Tamil Nadu, Coffee berries are processed in two ways: The three types of coffee are: Arabica
Kerala, Nilgiri Hills, i) Wet method: Berries are fermented, washed, dried and roasted. coffee, Robusta coffee and Liberica
Cardamom Hills are
ii) Dry method: Berries are dried in the sun and then pounded to removed coffee.
important coffee producers.
outer cover

Fibre Crops Of India:


Methods of
Crop Temperature Rainfall Soil Distribution Processing
farming
Cotton 23°C to 32°C, 80 cm to 120 Water retentive Broadcasting Gujarat, Tamil Nadu, After picking from fields,
(Kharif) and a major part cm (well– black or lava soil and drilling Maharashtra, Uttar Pradesh, cotton is sent for ginning
of it develops at distributed) and light well– methods. Punjab, Madhya Pradesh, in factories. Then it is
about 27C. drained alluvial Jharkhand, Andhra Pradesh, packed into bales and
soil. Haryana. The highest yield of sent to mills.
cotton comes from Punjab.
Jute 27°C to 34°C 170 cm to Alluvial soil found Sown by West Bengal, Assam, Orissa, After harvesting, retting
(Kharif) 200 cm in flood plains and broadcasting Bihar and Chhattisgarh. is done. After retting jute
deltas dibbling and is packed in bales and
drilling sent to mills.
methods.

REVIEW QUESTIONS AND ANSWERS

Short Answers: [2 Marks]


1) What are the commercially important by-products of sugarcane? What are their uses?
Ans: The commercially important by-products of sugarcane are molasses and bagasse.
i) Molasses is dark brown syrup that comes out during the manufacture of sugar. Molasses is
used to manufacture fertilizers, industrial alcohol, rum and yeast.
ii) Bagasse: This is the rejected or crushed sugarcane pulp, from which the juice has been
extracted. It is used as cattle-feed, raw material in the manufacture of paper, fibre board,
synthetic fibre and as an organic fertilizer and fuel for mills.
2) Explain the process of separating jute fibre.
Ans: The process of separating the jute fibre from the stem is known as retting. Harvested plants are
tied into bundles of 20 cm to 25 cm diameter and immersed in soft running water for two to three
weeks. Fibre is obtained from the stalk by beating small bundles of stalk with a wooden mallet,
which loosens the fibre.
34 Universal Tutorials – X ICSE – Geography Volume 2 of 4
Chapter 10: Commercial or Cash Crops … 35

3) Give the areas of rubber production in India.


Ans: Kerala, Tamil Nadu, Karnataka, and Andaman and Nicobar Islands are the rubber producing
areas of India. 91 per cent of the total area under rubber lies in Kerala alone.
4) Answer the following questions regarding tea cultivation in India:
a) What are the main varieties of tea?
Ans: There are three main varieties of tea: (i) Black Tea (ii) Green Tea (iii) Oolong Tea.
b) Why hill slopes are preferred for tea cultivation?
Ans: Tea plant cannot tolerate stagnant water around its roots. So, to prevent water-logging,
mountainous slopes are preferred for tea cultivation.
5) In which two ways coffee berries are processed?
Ans: Wet or Parchment method: In this method, coffee beans are fermented and washed in tanks
several times. Then they are dried and cured. Machines remove the thin skin of the berries
called parchment.
Dry or Native method: The outer covering of berries is removed in this method by drying them
under the sun. Seeds are pounded to remove their parchment.
6) Discuss some of the problems being faced by the coffee industry in India.
Ans: The total production of coffee in India is low as compared to other countries of the world. It is due
to the lack of high-yielding plants, poor and outdated management techniques and unimpressive
use of manures and pesticides. The quality of Indian coffee also fluctuates frequently depending
upon climatic and soil conditions in India.
7) Mention two features which reflect a shift from subsistence farming to commercial
farming.
Ans: Two features which reflect a shift from subsistence farming to commercial farming are:
i) Subsistence farming is meant to satisfy the bare needs of the farmers, whereas the
production in commercial farming is meant for markets.
ii) In subsistence farming, manual labour is applied to get enough production for the farmer and
his family, whereas machines, such as tractors, are used in commercial farming to achieve
higher yields.
8) Give two reasons why the yield of Indian agriculture is low as compared to world
standards.
Ans: The two reasons are:
i) Decreasing fertility due to overuse of land and primitive farming methods.
ii) Lack of knowledge of the use of chemical fertilizers, pesticides, soil characteristics, etc.
9) State two characteristics of plantation agriculture.
Ans: The two characteristics of plantation agriculture are:
i) The purpose of plantation agriculture is not only earning cash but also on a much larger scale
that resembles the factory production.
ii) The production in plantation agriculture, such as tea, coffee and rubber, requires huge
machines, scientific methods of cultivation, and special skill to produce and to market it.
10) Why is cotton grown widely in the Deccan Plateau?
Ans: The following geographical conditions are favourable in Deccan Plateau for growing cotton:
a) Availability of black/ regur soil suitable for growing cotton.
b) Frost free season.
c) Light rain during growing and clear sky with bright sun-shining during harvesting.
11) a) Which two types of groundnut plants are grown in India?
Ans: (i) Bunch type and (ii) Runner type.
b) Which are the main groundnut producing areas in India?
Ans: Gujarat is the leading producer of groundnut in India. It is also grown in Andhra Pradesh, Tamil
Nadu Karnataka and Maharashtra.
12) In which states of India mustard and rapeseed are grown extensively.
Ans: Uttar Pradesh, Haryana, Rajasthan, Punjab, West Bengal, Bihar, Orissa and Maharashtra,
Assam are the important states where mustard and rapeseed are grown extensively.
Volume 2 of 4 Universal Tutorials – X ICSE – Geography 35
36

13) Mention the geographical requirements of safflower plant and also mention its uses.
Ans: Safflower is grown in the Rabi season. It requires a temperature of 10 °C to 20 °C, and rainfall of
about 100 cm. Loamy alluvial and black soils are suitable for safflower plant.
Other than cooking purposes, the oil of safflower is used in the manufacture of paints and
varnishes as it has good drying properties. Its oil-cake is also used as animal feed.
14) Why Kerala leads in rubber production in India?
Ans: Rubber is an equatorial plant. The climate of Kerala similar to the climate of equatorial region. It
has high temperature and heavy rainfall throughout the year. Kerala gets rainfall from the south-
west monsoons as well as from north-east monsoons. It also has well-drained soil suitable for
rubber cultivation. Hence, Kerala leads in rubber production in India.

Short Answers: [3 Marks]


1) Name the most widespread fibre crop of India. What are its varieties?
Ans: i) Cotton is one of the most important fibre crops of India. They provide basic raw material
(cotton fibre) to the cotton textile industry. India ranks fourth in cotton production in the world.
It is a tropical and subtropical crop.
ii) On the basis of length, strength and structure there are three varieties of cotton:
„ Long staple cotton: longest fibre, used to make fine quality cotton found in Punjab,
Haryana.
„ Medium staple cotton: found in Rajasthan, Tamil Nadu
„ Short staple cotton: is less than 19 mm long, found in Uttar Pradesh, Andhra Pradesh,
etc.
2) What are the two main types of coffee grown in India? What is the purpose of the third
type of coffee?
Ans: i) Coffee thrives well in a hot and humid climate, with temperatures ranging from 18°C to 28°C.
The plant is sensitive to frost and cold. There are three varieties of coffee grown
commercially.
ii) a) Coffee Arabica: 75% of the coffee produced in India is of this variety. It is a superior
quality and most expensive. Roasting of coffee gives aroma and flavour to coffee.
b) The second variety of coffee is Robusta: The cost of production of this variety is less
and the yield per acre is higher; so it is cheaper.
c) The third variety of coffee is Liberica: It is cultivated in a small area and its main
purpose of produce is its use in making instant coffee.
3) What do you understand by the following technical terms:
(i) Inter-culture, (ii) Ginning, (iii) Tapping, (iv) Setts, (v) Topping, (vi) Retting, (vii) Curing
and (viii) Clonal planting.
Ans: i) Inter Culture: It is a type of agriculture in which more than one crop is grown simultaneously.
ii) Ginning: It is a process of separating seeds from the raw material which is known as lint.
iii) Tapping: Tapping is a process of wounding the bark of a rubber tree. From this cut on the
bark, latex oozes out which is collected in a pot.
iv) Setts: It is a piece of sugarcane with two or three joints which is sown.
v) Topping: It is the removal of flower - heads by plucking from a tobacco plant.
vi) Retting: It is a microbiological process of removal of jute fibre from its stalk. After harvesting
the jute, plants are submerged in stagnated water for two or three weeks. This loosens the
fibres to remove it from the stalk.
vii) Curing: After harvesting, the tobacco plants are stung on a bamboo pole for 10 to 15 days.
This activity creates the required aroma or flavour. This process is known as curing.
viii) Clonal planting: When tea bushes are raised from cutting of high-yielding mother plants
rather than from seeds for better yield, it is known as ‘clonal planting’, even in rubber.
4) Answer the following questions related to sugarcane cultivation in India:
a) What do you know about sugarcane plant? Why is it an important cash crop?

36 Universal Tutorials – X ICSE – Geography Volume 2 of 4


Chapter 10: Commercial or Cash Crops … 37
Ans: Sugarcane is a tropical plant and it belongs to the grass family. It has a thick jointed stem which
contains sugar. Its leaves are narrow and long. It can grow up to the height of three metres.
It is an important cash crop as it is the main source of sugar in India. It is a long duration crop
and its growing period is about 10 to 12 months.
b) What are the geographical requirements of temperature and rainfall for sugarcane
cultivation in India?
Ans: Temperature: For sugarcane cultivation, temperature between 20°C and 30°C is required. Very
high and very low temperatures slow down the growth of sugarcane plant. It cannot withstand
frost. Rainfall: Sugarcane grows ideally in areas having rainfall between 100 cm and 200 cm
annually. Irrigated lands are suitable for its growth, since it requires alternate spells of wet and
dry conditions.
c) Which soil is best suited for sugarcane cultivation?
Ans: Wet drained, rich, Alluvial soils of the Ganga plain and the black and laterite soils of South India
are ideal for sugarcane cultivation.
d) How is sugarcane propagated?
Ans: Though sugarcane can be grown from seeds but generally all commercial plantings are made
from stalk cuttings of two or three joints from the top of a healthy cane. These cuttings, known as
setts, are planted in furrows at a distance of about 30 cm. The plants start growing in about two
weeks.
e) What do you mean by ratoon cropping?
Ans: This is yet another way of growing a new crop of sugarcane. In this method, sugarcane is cut
close to the ground and its root is not plucked. After some time, it begins to grow again and
produces a second crop called ratoon. In ratoon cropping, there is no need for fresh plantation.
f) What is the advantage of ratoon cropping?
Ans: Ratoon crop matures earlier than plant crop. There is no extra expenditure on preparing the
fields and planting, hence, it is cheaper as compared to replanting the crop.
g) What is the disadvantage of ratoon cropping?
Ans: Generally, two ratoon crops are obtained from each planting. Ratoon cropping yields thinner
canes with lower sugar content. This is because sugarcane plant exhausts the soil. Risk of pests
and diseases also increases in ratoon cropping.
h) How is sugarcane processed after its harvesting?
Ans: After harvesting, sugarcane should be crushed within 48 hours, otherwise its sugar content will
decrease. After crushing sugarcane, juice is boiled with lime and then filtered.
i) What is produced from sugarcane juice?
Ans: Of the total production of sugarcane in India, 50% is processed into gur, 30% into white sugar
and the remaining 20% into khandsari. Its waste cane, after extracting juice, is used in the
manufacture of paper.
j) Why is the yield of sugarcane higher in the Deccan rather than Northern Plain?
Ans: Although the total area under cultivation of sugarcane is more in northern India, the yield per
hectare in the Deccan is higher because there are more favourable geographical conditions.
Also, there is more application of scientific methods of cultivation in south than in the north.
Moreover, in the north, extreme temperatures in summer and winter reduce the yield per acre.
There are more frost free days in south than in the north.
5) What is the economic importance of mustard and rapeseed in India?
Ans: i) Mustard and rapeseed oil is used as cooking medium in a large part of India.
ii) Their seeds and oil are used as condiment in the preparation of pickles and for flavouring
curries and vegetables.
iii) Leaves of these plants are used as green vegetable.
iv) The residue or the oil-cake of mustard and rapeseed is used as cattle feed.
6) a) What is linseed? State whether it is edible or non-edible.
Ans: It is a seed obtained from a fibre plant known as flax. It is a rabi crop. Linseed is non-edible. Its
oil is not suitable for human consumption.
Volume 2 of 4 Universal Tutorials – X ICSE – Geography 37
38

b) What are its uses?


Ans: i) It is widely used as drying oil and is also used for varnishes, paints, printing ink, oil-cloth,
water-proof fabrics, and linoleum etc.
ii) Linseed oil is used for seasoning of wood.
iii) Its oil-cake makes cattle feed.
7) a) Why is jute called ‘golden fibre’ in India?
Ans: Jute fibre is yellowish in colour. Moreover, it earns a lot of foreign exchange for India. It is thus
called ‘golden fibre’.
b) What are the different uses of jute in India?
Ans: Jute fabric or sack cloth is used for packing several commodities such as rice, wheat, etc., and
wool and cotton in bales, It is also used in the manufacture of carpets, rugs, upholstery, ropes,
strings and tarpaulins.
c) How is jute sown in India?
Ans: Jute is generally sown in February on lowlands, and in March to May on highlands in well
ploughed fields. For sowing, broadcasting method is used, i.e. seeds are thrown in shallow
furrows. But, now–a-days, drilling method is being used for sowing jute. When the plants are
15cm tall they are transplanted.
d) How is harvesting of jute done?
Ans: After sowing, the jute crop matures in about four to six months. Its harvesting takes place
between July and September, when they are 3–4m tall. They are harvested after they bloom but
before seeds are produced. The plants are cut close to the ground or they are uprooted by
flooding the fields.
8) a) How is tapping done in rubber cultivation?
Ans: The process of obtaining latex or sap from the rubber tree is known as tapping. In tapping, first, a
cut is made about 1 to 1.5 metre above the ground level and at an angle of 30° from the
horizontal. The cut is made half-way around the circumference of the tree and sloped down to
the right. A zinc pipe is adjusted in the vertical groove through which the latex comes out.
b) When is tapping done and when is it not done?
Ans: Tapping is done early in the morning because in the morning the latex of rubber flows freely.
Tapping is not done during rains as mixing of water may dilute the latex. Rubber trees are
tapped between 200 and 300 days a year. A rubber tree gives latex from 25 to 30 years.
c) Why is tapping considered a skilled job?
Ans: Tapping of latex from a rubber tree is really a skilled job because a tapper has to make a cut
which is only 2 mm deep. If the cut becomes deep, it will damage the cambium which is a paper-
like thin skin between the bark and the wood of the tree.
9) a) Discuss the importance of tea industry in India.
Ans: i) Tea is an important foreign exchange earner for our country.
ii) Tea industry provides employment to more than a million people in India.
iii) Through its demand, tea industry supports plywood, fertilizer and transport industries.
b) Which are the important areas of tea production in India?
Ans: Hill slopes in the Brahmaputra and Surma valleys in Assam and the hill slopes of the Nilgiris,
Anamalai and Palanis in Tamil Nadu, Karnataka, and Kerala are the chief areas of tea
production in India. After Assam, West Bengal is the second important tea growing state of our
country.
10) What are the three varieties of coffee grown in India?
Ans: Arabica: 75 per cent of the coffee produced in India is of this variety. This is a superior quality
and most expensive.
Robusta: The cost of production of this variety is less and the yield per acre is higher, hence it is
cheaper.
Liberica: This variety is cultivated in a small area. It is mainly used to make instant coffee.

38 Universal Tutorials – X ICSE – Geography Volume 2 of 4


Chapter 10: Commercial or Cash Crops … 39

11) What is the distinction between Cash crop and Plantation crop?
Ans:
Cash crop Plantation crop
This type of crop is grown for the purpose of The purpose of a plantation crop is also
earning cash on a small scale by individual earning cash but on a much larger scale that
farmers. resembles the factory production.
Its production does not require a huge Its production requires a huge capital.
capital.
It does not require a special skill to produce It requires special skill to produce and market
and market it. it.
Cotton, jute and tobacco are some cash Tea, coffee, rubber, etc. are some plantation
crops. crops.
A cash crop can be grown by manual labour. Production of a. plantation crop requires
huge machines and scientific methods of
cultivation.
12) What are the climatic conditions necessary for the cultivation of castor seed? Name two
prominent castor seed growing states in India.
Ans: i) Castor seeds are a subtropical crop. In the north it is a Kharif crop and in the south it is a
Rabi crop.
ii) Climatic conditions necessary for it are:
Temperature: 20°C to 25°C.
Rainfall: 50 to 75 cms. It is tolerant of drought conditions. It cannot withstand frost.
Soil: It grows best on deep loamy soil.
iii) Its seed contains 50 % oil. The states it is grown in are Andhra Pradesh, Gujarat and
Karnataka.
13) a) Why do tea farmers construct drains all over the tea plantation area?
Ans: Farmers of tea plantation construct one metre deep drains all over the plantation of tea to drain
out surplus water as stagnant water is harmful to the roots of a tea plant.
b) What is ‘clonal planting’ in tea cultivation?
Ans: In this method of tea growing, seeds are not planted, rather cuttings from good high-yielding
mother plants are used. This method is known as clonal planting.
14) Mention two methods of processing and curing jute.
Ans: i) Jute - After harvesting, the plants are tied into bundles. After 2 - 3 days these bundles are
submerged into water for retting and left for 2 - 3 weeks. Retting is a microbiological process.
It loosens the bark and facilitates removal of fibre from the stalk. A close watch is kept and as
soon as it is easy to separate the fibre from the stalk, retting is carried out. High temperature
quickens the process.
ii) After the retting is complete, the bark is peeled from the plant and the fibre is removed. After
this stripping, rinsing, washing and cleaning is done. Plenty of water is required for retting.
The fibre is then dried in the sun and pressed into bales. All this has to be done by hand.
They are then dispatched to jute mills.
15) How is rubber processed before marketing it?
Ans: The following steps are involved in the processing of rubber:
i) Coagulation of rubber by adding acids into it.
ii) Pressing of rubber to squeeze out water.
iii) Drying and smoking of rubber.
iv) Packing rubber in bales for marketing.

Volume 2 of 4 Universal Tutorials – X ICSE – Geography 39


40

16) State three reasons behind the abundance of tea plantations in Assam.
Ans: i) Tea is classified as an important beverage and a plantation crop. The main tea producing
areas are (1) North East India, (2) South India. In North East India, the Assam hill region is
the largest tea producing area in the world. The conditions here are most favourable because
this region receives heavy rainfall, about 250 cm, from the South West Monsoon winds, the
Bay of Bengal branch.
ii) The temperature conditions are favourable being above 20°C. Morning fog helps and frost
never occurs.
iii) Moreover, labour is available in plenty. Hence, hand picking of tea leaves is made easy.
Assam produces the most popular tea and has a pungent taste; it is strong and used for
blending milder ones. About 40% of the tea in the country is produced here.
17) Describe the conditions necessary for the growth of two non-edible oilseeds in India.
What is their use? Where are they grown?
Ans: i) The two non-edible oils in India are linseed and castor seed.
Linseed is obtained from a fibre plant known as flax. It is a Rabi crop. Its oil is not suitable for
human consumption. The climatic conditions required are temperature 10°C to 20°C.
Rainfall: 80 cm to 150 cm is adequate.
Soil: It is best grown on clayey, loamy soil of the alluvial region as well as black cotton soil of
the Deccan Trap.
ii) Castor seed is a Kharif crop and also poisonous. It is unfit for cattle and human consumption.
Temperature: It requires 20°C to 26°C. Frost is harmful.
Rainfall: It can resist drought. It grows best on deep loamy soil.
iii) Use of Linseed: Widely used as drying oil, in varnishes, paints, printing ink. Oil - cloth,
water-proof fabrics and linoleum. The oilcake makes cattle–feed.
Uses of castor seed: The oil is used to manufacture paints, varnishes, printing ink, soaps,
plastics, oil cloth and transparent paper. It is also used as lubricant for high - speed engines
and aeroplanes.
18) Name and describe the methods by which rubber is propagated.
Ans: Rubber can be cultivated or propagated in the following two ways:
i) Propagation by seeds: In this method, first of all, good quality seeds are germinated before
planting. After germination, seedlings are planted in the nurseries. Unhealthy plants are
eliminated in the early stage and good care is given to the rest of the healthy plants.
ii) Propagation by bud grafting: In this method, buds from high yielding rubber tree is grafted
on the seedling of about 5 cm high. The graft is then bound for a few weeks. A leaf is also
tied on the graft to provide shade. When new shoot starts coming out or the graft is
established, the original trunk is cut off just below the union. After this, the new shoot or the
grafted section becomes the main part of the tree.
19) Explain the increasing importance of Maharashtra and southern states for sugarcane
production.
Ans: Maharashtra and the southern states of Tamil Nadu, Karnataka and Andhra Pradesh are
becoming increasingly important for the production of sugar cane and sugar because of their
ideal geographical conditions.
The southern states and Maharashtra have well- planned plantations and the use of modern
scientific agricultural methods. The capital investment is tremendous due to the sugar lobby.
Modern sugar factories are located close to the plantations unlike in the northern states like Uttar
Pradesh, where the mills are a distance away, leading to a loss of sucrose due to delay in
crushing the cane.
Irrigation and fertilizers are widely used to ensure a good crop of sugarcane.
The Sugar cane Research Institute is located at Coimbatore and is successfully conducting
research on improved hybrid varieties with much higher sugar content.
The use of hybrid Coimbatore cane produces the maximum sugar cane and sucrose per
hectare.

40 Universal Tutorials – X ICSE – Geography Volume 2 of 4


Chapter 10: Commercial or Cash Crops … 41

20) a) What is the role of pruning in tea cultivation?


Ans: Periodic pruning keeps the bush at a convenient height and encourages the growth of new
shoots with soft leaves.
b) What are ‘flush’?
Ans: Shoots in a tea plant are known as flush. They consist of several leaves and a bud.
c) What do you mean by ‘fine plucking’?
Ans: Plucking of two leaves and a bud from the tea bush is known as fine plucking.
21) State the problems that are faced by sugarcane farmers in India.
Ans: The following are the problems faced by sugarcane farmers:
i) Canal irrigation is ideal for providing alternating dry and wet conditions for sugarcane. But in
India, canals are often non-perennial. This factor makes the supply of water uncertain.
ii) In south India, digging of canals is very difficult. Also, rainfall received is low in interior parts
of the Deccan.
iii) Sugarcane cultivation requires large quantity of manure and fertilizers as it is a soil-
exhausting crop.
iv) Cultivation of sugarcane on very small farms becomes highly uneconomic.
v) In many cases, farms growing sugarcane are away from the factories. A delay of more than 2
days, i.e. 48 hours between harvesting and crushing of sugarcane gives decreased sugar
content of the sugarcane.
vi) The support price determined by the government of sugarcane does not consider the quality
of sugarcane. So, there is no incentive for better crop.
22) What are the geographical conditions necessary for the growth of jute?
Ans: i) Temperature: Jute is a tropical crop and requires a high temperature between 24°C and
34°C. It also requires a high relative humidity of 80 per cent and above.
ii) Rainfall: It requires more water; hence, areas having 170 cm to 200 cm rainfall are ideal for
jute growing. Continuous drought is harmful to this crop.
iii) Soil: It is grown in the areas of new alluvium. Fertile, loamy and well-drained soil is ideal for
jute cultivation.
iv) Labour: It is a labour intensive crop. It requires large number of workers in planting, cutting
and getting fibre. So, densely populated areas like West Bengal are highly suitable for jute.
23) Describe the processing of jute right from its fields to the mills.
Ans: Making of jute is a labour-intensive process which involves the following stages:
i) Harvested plants are left on the fields for two or three days so that their leaves dry up and
shed. Then they are made into bundles.
ii) Then these bundles of 20 cm to 25 cm in diameter are immersed into water for retting and left
there for two to three weeks. This makes the outer bark loose and helps in the removal of the
fibre from the stalk. This process is called retting.
iii) When the retting process is considered complete, ten to twelve reeds are taken out from the
stalk of the jute plant and are beaten with a wooden mallet to loosen the fibre.
iv) The fibre, so obtained, is washed in clean water, wrung and spread out in the sun to dry.
v) Then this dry fibre is made into bundles and sorted according to quality and pressed into
bales and transported to manufacturers.
24) Why is rubber grown on flat or gently sloping land?
Ans: Due to the following considerations rubber is grown on gentle slopes:
i) On flat or gentle slopes, machines can be used to clear the original vegetation.
ii) Soil erosion becomes less on relatively gentle slopes, whereas on steep slopes soil erosion
has turned out to be a major problem.
iii) Terraced farming on steeper slopes may add to the cost of rubber cultivation because the
formation of terraces requires additional labour.
Volume 2 of 4 Universal Tutorials – X ICSE – Geography 41
42
iv) Therefore, tapping of latex becomes easier on gently sloping land.
25) What are the climatic and soil conditions required for the cultivation of tea?
Ans: Tea plant can be grown in varying climatic conditions and from sea level to 2000 metres of
altitude.
Temperature: An ideal temperature for the growth of tea crop is between 13°C to 35°C. Frost is
harmful for the tea plant. High humidity and dew help in the rapid growth new leaves of a tea
plant.
Rainfall: Tea plant requires heavy rainfall ranging between 50 cm and 250 cm. Rainfall should
be well-distributed throughout the year. Long spells of dry weather are not tolerated by tea
plants.
Soil: Tea requires very fertile soil containing organic matter with an element of iron. It requires
light loamy soil with a porous sub-soil for its growth.
26) Describe the major steps in the processing of tea.
Ans: i) Withering: This step removes the excess moisture from the tea leaves.
ii) Rolling: To get the characteristic flavour, the leaves are twisted to break the cells. This step
exposes the natural juices to fermentation.
iii) Fermentation: In this step, tea leaves are partially oxidised to change the colour of the
leaves.
iv) Drying: In this process, tea leaves are put on a conveyor belt and passed through an oven
under high temperature.
v) Sorting: Tea leaves are sorted according to decreasing size.
vi) Blending: To maintain consistency in the flavour of tea throughout the year, different teas
are blended together.
vii) Packing: Tea needs a lot of care while packing to protect its flavour. Generally, plywood
cases with a lining of aluminium foil paper are used in the packing of tea to keep it fresh and
to retain its flavour.
27) How does coffee planting take place? Discuss.
Ans: i) Before sowing the seeds of coffee, land is cleared from unwanted bushes and trees.
Terraces are formed on the hill slopes and contour drains are provided. Shady trees are
planted a year in advance.
ii) Coffee seeds are planted in specially prepared seed beds. After about 45 to 50 days, they
are transplanted in the fields.
iii) The same coffee plant is replanted in coffee estate when it is of six months to two years old.
28) Mention two problems faced by the sugarcane cultivations in India and state the role of
the Central Government in solving these problems.
Ans: Two problems faced by the sugarcane cultivations in India are as a follows:
i) Sugarcane cultivation requires a large quantity of manure and fertilizers as its is soil
exhausting crop.
ii) Sugar mills are far from the sugar-farms leading to delay in ‘crushing’ and a loss of sugar
content. The role of Central Government in solving these problems is very important as:
a) The support price which is determined by the Government for sugarcane would be
considered on the quality of sugarcane.
b) The Government has been trying to help the mills as many of them were old-fashioned, and
thus, closed. They are being relocated near the sugar–farms.
29) What are the geographical conditions required for the cultivation of groundnuts in India?
Ans: Groundnuts are grown in India as a Kharif and also as a Rabi crop.
Temperature: Groundnuts are grown best in the areas with temperatures ranging between 20°C
and 28°C.

42 Universal Tutorials – X ICSE – Geography Volume 2 of 4


Chapter 10: Commercial or Cash Crops … 43
Rainfall: Groundnut is a drought-resistant tropical leguminous crop. It can be grown in areas of
less rainfall i.e., 50 cm to 75 cm. This rainfall should be well-distributed during the flowering
stage. Groundnut needs dry and warm weather conditions at the time of ripening. Frost, severe
droughts and stagnant water are harmful to groundnut plants.
Soil: Well-drained light sandy loam, red and black soils.
30) State the economic importance of groundnut.
Ans: i) Groundnut oil forms an excellent cooking medium as refined oil and vanaspati ghee.
ii) It is used in the manufacture of soaps, lubricants, candles, and margarine.
iii) It is a leguminous plant which enriches the soil.
iv) Its oil-cake is used as cattle feed.
v) Groundnut is eaten raw and roasted. It is nutritious as it contains vitamins A and B.
Groundnut is thus an important cash crop.
31) a) Mention the climatic and soil conditions suitable for sesame (till) in India.
Ans: Temperature: It requires a temperature of about 21°C to 23°C.
Rainfall: Rainfall of 40 cm to 50 cm is suitable for sesame cultivation. Heavy rains and
prolonged droughts are negative to this plant.
Soil: Dry sandy soils and the black cotton soil are suitable for the cultivation of sesamum.
b) Which are the leading sesame growing states of India?
Ans: Major sesame producing states are Gujarat, Andhra Pradesh Tamil Nadu Karnataka
Maharashtra, Madhya Pradesh, Uttar Pradesh, Rajasthan, and Jharkhand.
c) What are the uses of sesame or give its economic significance?
Ans: Sesame is used in different ways as follows:
i) Sesame oil is used in cooking, perfumery and medicinal purposes.
ii) Its seeds are rich in protein, carbohydrates and minerals, so these can be eaten fried or with
sugar.
iii) Oil-cake of sesame makes an ideal food for mulch cattle and pigs.
32) What are the characteristics of Soyabean?
Ans: i) This temperate crop requires summer temperature of about 21°C & a rainfall of about 100cm.
ii) It is a versatile plant its beans may be eaten as vegetable or can be made into different types
of food. Soyabean is an important source of oil.
iii) It has been an ancient food crop of India, China, Japan, and Korea.
iv) Its beans contain proteins much more than meat.
v) Soyabean contains all the 22 amino acids required for a balanced and healthy growth.
vi) They are good source of iron and calcium and are low in cholesterol and starch.
vii) Flour made from Soyabeans is gluten-free.
33) What are the geographical requirements for the cultivation of sunflower seed? Also
mention its area of production.
Ans: Sunflower needs cool and moist climate at the time of sowing and early growth and warm, sunny
weather at the time of flowering and harvesting.
It is grown mainly in Madhya Pradesh, Uttar Pradesh, Gujarat, and Maharashtra.

MISCELLEOUS QUESTIONS / HOME WORK


1) What is meant by the term ‘agriculture’?
2) How is agriculture a consumer of industrial products?
3) Mention the advantages of mixed farming.
4) Name two disadvantages of shifting cultivation.
5) What is mixed farming?
6) Where is intensive commercial farming done in India?

Volume 2 of 4 Universal Tutorials – X ICSE – Geography 43


44
7) What do you understand by dry farming?
8) What are the geographical conditions suitable for the cultivation of rice?
9) Name the state that produces the highest quantity of rice in India.
10) Write a short note on upland rice.
11) Compare the climate conditions for wheat cultivation with those for rice cultivation.
12) In which region is ragi grown in India? Why?
13) What kind of soil is needed for the cultivation of millets?
14) State the climate conditions suitable for the cultivation of pulses.
15) How does the cultivation of pulses usually help in restoration of fertility of the soil?
Sugarcane:
16) Name the leading sugarcane producing State in India.
17) With reference to sugarcane cultivation in Uttar Pradesh answer the following:
i) The quality of the soil for its cultivation.
ii) The climate that is suitable for its growth.
18) Why is the sugarcane production increasing in Maharashtra?
19) What are the advantages of ‘Ratooning’?
20) Name the state in the Deccan region which is the largest producer of sugarcane.
21) Differentiate between Ratooning and Sett methods of sugarcane cultivation.
Oilseeds:
22) What geographical requirements and climate conditions are suitable for the cultivation of
groundnuts?
23) Name some of the important oilseeds in India. Describe where they are cultivated and the
purpose for which they are used.
24) With reference to groundnuts, answer the following questions:
i) Which two states in India are the leading producers of groundnut?
ii) Mention the main uses of groundnut.
iii) Name one state which is the largest producer of groundnut.
25) State the main uses of linseed oil?
26) Name one state where consumption of mustard oil is popular.
27) Name two non-edible oilseeds.
28) Give two uses of oilseeds, besides production of cooking oil.
29) Briefly describe how soyabean is cultivated.
Cotton:
30) What are fibre crops? Give two examples of plant fibre crops.
31) What conditions of soil favour cotton growth?
32) Why is dry weather necessary at the time of harvest?
33) Mention the climatic conditions necessary to grow cotton.
34) Write a short note on processing of cotton.
Jute:
35) Which is the most important fibre crop grown in West Bengal?
36) State the soil conditions that favour the growth of jute in Assam.
37) Why are floods beneficial for the growth of jute?
38) What are the climate requirements for growing jute?
39) In what way is Ganga-Brahmaputra Delta suitable for jute cultivation?
40) Name two articles made of jute.

44 Universal Tutorials – X ICSE – Geography Volume 2 of 4


Chapter 10: Commercial or Cash Crops … 45

PREVIOUS BOARD YEAR QUESTIONS:


1) Name one crop each with which the following processes are associated:
a) Retting b) Ginning [2000]
2) After oil extraction, what other uses can the oil seed be put to? [2000]
3) What is ‘crop rotation’? [2000]
4) Name two states which are leading producer of wheat in India. [2000]
5) Name one state which is leading producer of groundnuts. [2000]
6) Outline two remedial steps being taken by the government to boost agricultural production.[2000]
7) What is a Rabi crop? [2001]
8) Is wheat a Rabi or a Kharif crop? In which state of India is the largest amount of wheat grown?
[2001]
9) What is ‘ratooning’? [2001]
10) Give two of the advantages of rationing. [2001]
11) Name any two non–edible oilseeds grown in India. [2001]
12) State the economic significance of tobacco crop in India. [2001]
13) Why is it necessary to crush sugarcane within 24 hours of harvesting? [2001]
14) Name two medicinal plants. [2001]
15) Name four important medicinal plants grown in India. [2002]
16) Name the two major jute products. [2002]
17) Name one area in India where jute is cultivated widely. [2002]
18) Which state is the largest producer of sugarcane in India? [2002]
19) What are the two methods of cultivation of rice? Which one is known for its better yields? [2002]
20) Name an area of wheat cultivation in India and state why is it suitable for the cultivation of wheat.
(Mention two reasons). [2002]
21) Do you think that the cultivation of pulses is lagging behind in recent times? Give two reasons to
justify your view. [2003]
22) Mention two problems faced by the sugarcane cultivators in India and state the role of the
Central Government in solving these problems. [2003]
23) In the context of the Indian agriculture, answer the following: [2003]
a) Mention two features which reflect a shift from subsistence arming to commercial farming.
b) Give two characteristics of plantation agriculture are low as compared to world standards.
c) State two characteristics of plantation agriculture. [2001, 2003]
24) Why millet is called dry crop? [2000, 2001, 2003]
25) State two differences between intensive agriculture and extensive agriculture [2003]
26) Name the months when the Kharif and Rabi crops are:
a) Sown and b) Harvested [2004]
27) Name three coarse grains gown in India. Why are they so called? [2004]
28) Why is cotton grown widely in the Deccan Plateau? [2004]
29) Explain briefly the following terms:
a) Ginning b) Transplantation c) Broadcasting d) Ratooning [2004]
30) State the importance of the tobacco crop to the Indian economy. Which particular element is lost
in a large quantity from the soil during the cultivation of this crop? [2004]
31) What is meant by mixed farming? State one advantage of this type of farming. [2004]

Volume 2 of 4 Universal Tutorials – X ICSE – Geography 45


46
32) Why tea is considered a labour intensive crop? Name two States in India where tea is widely
grown. [2004]
33) State the conditions necessary for the cultivation of rubber [2004]
34) State one medicinal use each of the following:
a) Neem b) Brahmi [2004]
35) Mention two problems associated with agriculture in India. [2005]
36) Which oilseed is inedible by man? State any two uses of its oil. [2005]
37) Describe any two main features of Subsistence Agriculture. [2005]
38) Explain briefly why:
a) Tulsi is used as a medicinal herb
b) Wheat is grown as a Rabi Crop [2005]
39) Give two reasons for pulses being grown extensively in India. Name any two pulses. [2005]
40) Describe two geographical conditions which favour the growth of this crop. [2005]
41) State two geographical requirements for the growth of cotton. [2006]
42) Mention two differences between shifting agriculture and plantation agriculture. [2006]
43) a) Name two main varieties of coffee.
b) Why are the following trees commonly grown in a coffee estate?
(i) Dalap and Silver Oak (ii) Orange and Plum [2006]
44) State three important aspects of the Japanese method of rice cultivation. [2006]
45) a) Mention two differences between subsistence agriculture and commercial agriculture.
b) State two geographical requirements for the growth of rubber.
c) In which season is wheat grown in northern India? What are the rainfall and soil requirements
of this crop?
d) Explain Why:
i) Pluses are grown as rotation crops.
ii) Millets are referred to as dry crops.
iii) Tea plantations are found on hill slopes. [2007]
46) a) i) When are Kharf crops (1) sown and (2) harvested?
ii) Name a cash crop which is also a Kharf crop.
b) India is the largest producer of Tea. State the climatic factors necessary for its growth.
c) Explain the following methods of propagation and name the crop associated with each.
i) bud grafting ii) ratoon cropping.
d) i) What are the geographical conditions necessary for the cultivation of groundnuts?
ii) What climatic condition adversely affects the groundnut crop?
iii) Name two non-edible oilseeds grown. [2008]
47) a) Explain two important characteristics of plantation farming. Name one important plantation
crop.
b) Which state is the leading producer of Jute ? Give any two factors responsible for this.
c) i) The yield of sugarcane is higher in the Deccan. Give two reasons for this.
ii) Give one advantage and one disadvantage of ratoon cropping.
d) Define the following terms:
i) Ginning ii) Arabica. [2009]
48) a) Study the picture given and answer the following questions:

46 Universal Tutorials – X ICSE – Geography Volume 2 of 4


Chapter 10:
1 Commerc
cial or Cash Crops
C … 47

i) Name the e crop which is being plan nted. Give on ne benefit off this method
d of planting this
t crop.
ii) Mention the
t climatic conditions
c which favour the
t cultivation of the crop p being planteed.
b) Name the state in Indiia which lea ads cotton cu ultivation. Mention two cclimatic factoors which
affect the cotton cultivattion adversely.
c) Answer the e following qu uestions with h reference too sugarcane:
i) Mention two
t different ways in whicch it is propa agated.
ii) Why is a lot of labourr required forr its cultivatioon?
iii) Why must the sugar mills be nea ar the sugar fields?
f
d) Give geographical reasons for the fo ollowing:
i) Tea bush hes are prune ed at regularr intervals.
ii) Tapping of rubber tre ees is usuallyy done in the morning hou urs.
iii) Oil cake is a useful residue. [2010]
49) a) Mention any two proble ems of agricu ulture in India
a.
b) State two geographical
g requirementts for the groowth of wheat in India.
c) With references to the cultivation
c of coffee in Soouthern India:
i) Name the e two varietie es of coffee plants
p grown on commerccial scale.
ii) Why are coffee estate es inter-planted with oran nge trees, caardamom and d pepper vines?
iii) Name on ne state whe ere coffee is grown
g extensively.
d) Explain the following terrms:
a) Withering g b) Plantation Farming c) Drilling
D [2011]
50) a) What is the e importance of agriculturre in India? Mentain
M two factors.
f
b) Name a sta ate which prroduces shorrt staple cottton. Which climatic
c and soil conditions favour
the cultivatiion of cotton in the state mentioned?
c) Give geographical reasons for the fo ollowing:
i) Jute has tot be retted.
ii) Tea is grrown on hill slopes.
s
iii) Silver oa
ak and banan na trees are grown on co offee plantatioons.
d) Explain the following:
i) The propa agation of ruubber by the bud grafting method.
ii) The prop pagation of su ugar cane byy rotoonig.
iii) The prop pagation of paddy
p by tran
nsplantation. [2012]
51) a) Distinguish between Inttensive comm mercial farming and Extensive commercial farming.
b) With reference to rice cultivation, an nswer the following:
i) Name two o leading staates in the prroduction of rice.
r

Volume 2 of 4 Universal Tuto


orials – X ICS
SE – Geograph
hy 47
48
ii) Give two advantages of growing rice in nurseries.
c) i) Name two states where coffee plantations are found.
ii) What conditions of soil and climate are favourable for the cultivation of coffee?
d) Explain in brief the following:
i) Retting
ii) Ginning
iii) Broadcasting [2013]
52) a) How has poverty and fragmentation of land become problems of agricultural India?
b) Mention any two features of plantation farming stating two examples.
c) Mention three differences in the geographical conditions and cultivation of rice and wheat.
d) Give a geographical reason for each of the following:
i) Cotton grows widely in Maharashtra.
ii) Clonal planting is the best method for tea propagation.
iii) Oilseeds are an important commercial crop grown in India. [2014]

48 Universal Tutorials – X ICSE – Geography Volume 2 of 4


Chapter 11: Industries in India 49

Chapter 11: Industries in India (Mineral Based &


Agro–Based)
→ Manufacturing Industries
→ Need of Industrialization
→ Factors affecting the infrastructure of Industries
→ Classification of Industries
→ Distribution of Industrial Regions
→ Agro–based Industries
→ Textile Industry
→ Sugar Industry

Manufacturing Industries:
¾ The term ‘manufacture’ means the making of goods or wares by manual labour or by machinery,
especially on a large scale.
¾ Manufacturing may be defined as the processing of raw material to enhance its value.
¾ For example, the amount of raw materials used in the manufacture of a watch is small and coasts
very little, but the finished product (watch) is expensive and valuable.
¾ Similarly, the cost of raw materials used in the manufacture of pharmaceutical products is much
less than that of the final product (medicine).
¾ Manufacturing industries are engaged in processing and altering of raw materials and semi–
finished products into finished products. For example, wheat is not directly used in its raw form. It
has to be ground into flour and then baked to be used by humans.

Need of Industrialization:
¾ Industrial growth of a country determines the strength of a nation. A country is considered a
wealthy one when it succeeds in transforming its raw materials into a variety of manufactured
goods.
¾ Industrial development not only provides opportunities for employment but also helps in
modernizing agriculture which forms the base of the Indian economy.
¾ Moreover, industrialized countries have a favorable balance of trade as they earn a lot by
exporting their products.

Factors affecting the infrastructure of Industries:


¾ The infrastructure of an industry depends on two factors, geographical and commercial.

Geographical Factors:
 The constant and assured supply of raw materials.
 Supply, of the cheapest source of energy.

Volume 2 of 4 Universal Tutorials – X ICSE – Geography 49


50
 Proper supply of water.
 Efficient means of transport.
 An easy access to the market.
 Availability of skilled labour at reasonable wages.
 Climate also plays a major role in the location of an industry.

Commercial Factors:
 Capital in the setting up and expansion of an industry.
 Trained and motivated technical management.
 State Government also plays an important role to give concession in taxes.
 Industries are classified in various ways according to various dimensions.

Classification of Industries:
Classification of Industries

Raw Material Product Size Ownership Others

Light Heavy
Large Scale Medium Scale Medium Scale

Agro-based Mineral-based

Private Public Joint Cooperatives


Sector Sector Sector

Location and Market Functions

Village Cottage Primary Secondary Tertiary Ancillary


or Basic or consumer

Distribution of Industrial Regions:


¾ In India, four areas with higher concentrations of industries are:
 The Western Industrial Region: Mumbai–Pune and Vadodara–Ahmedabad belts.
 The Eastern Industrial Region: Damodar Valley belt.
 The Southern Industrial Region: Cities of Bangalore, Hyderabad, Madurai, Coimbatore and
Chennai.
 The Northern Industrial Region: The Ganga– Yamuna Valley belt.

Agro–based Industries:
¾ Those industries which their raw materials from agriculture are called agro–based industries.
Textile industry and sugar industry are examples of agro–based industry.

50 Universal Tutorials – X ICSE – Geography Volume 2 of 4


Chapter 11: Industries in India 51

Textile Industry:
 Textile has traditionally meant a woven fabric. The term comes from Latin word ‘texere’,
meaning to weave.
 On the basis of raw materials used, the textile industry can be divided as – Cotton textile,
woollen textile, silk textile and jute textile.
 Cotton textile industry is one of the greatest industries of India. The first textile mill set up near
Kolkata in 1818. But it really made a start in Mumbai when a cotton textile mill was set up there
exclusively out of Indian capital in 1854.
 Cotton industry provides living to farmers cotton–boll pluckers and workers engaged in ginning,
spinning, weaving dying designing and packing.
 The important centres of cotton textile industry in India are Mumbai, Ahmedabad, Coimbatore,
Madurai, Indore Nagpur, Sholapur, Kolkata, Delhi, Bangalore and Hyderabad.
 The problems faced by cotton textile industry are inadequate supply of raw material, out–
dated machinery and low productivity of workers, fast changing fashion and design
uneconomical unit, and competition in foreign markets and synthetic fibre.
 Handloom and Khadi industry retains our hoary trade of providing large scale employment in
one’s home and cottages.
 The first woollen mill was set up in Kanpur (U.P) in 1876. The important centres of woollen
industry are Dhariwall, Amritsar, Ludhiana, Delhi, Srinagar, Kanpur, Bangalore, Mumbai,
Ahmedabad, Jamnagar, Gwalior and Kolkata.
 Silk textile industry, a small scale industry, plays an important role in the economy of our
country as it provided employment to over 4 million people. There are two varieties of silk –
mulberry silk and non–mulberry silk.
 Man–made fibres, called synthetic fibres, are produced by chemical processes. They include
rayon, terylene, acrylic and nylon.
 Jute industry, the second agro–based industry after cotton textiles, provides employment to
over five lakh people.
 Jute is a long, soft and shining fibre that can be spun into coarse thread. Jute industry is
concerned with the production of Hessian (cloth used for wrapping bales of raw cotton, etc.)
gunny bags, carpets, and rugs.
 The jute industry is concentrated in west Bengal. The other states that produce jute–goods are
Bihar, U.P., Andhra Pradesh, Madhya Pradesh, and Jharkhand.
 Inadequate supply of raw material, outdated machinery, irregular supply from substitutes and
labour problem, competition in the world market are some problems faced by jute industry.

Sugar Industry:
 Sugar industry is one of the most important agro–based industries in India. It employs about
2.5 lakh skilled and unskilled workers. India is second in the world production of sugar.
 Gur or jaggery and khandsari are made from sugarcane juice by indigenous method, and sugar
is produced by sugar factories. Molasses, Bagasses and Press mud are the by–products of
sugar industry.
 Distribution of sugar cane: Maharashtra, northern India, peninsular India.
 Sugar industry in India suffers from various problems such as:
z Inferior quality with low sugar content.
z Short period of crushing season.
z Less and irregular supply of raw material.
z Imbalanced distribution of sugar mills.

Volume 2 of 4 Universal Tutorials – X ICSE – Geography 51


52
zWastage of some amount of cane due to pressure on sugar mills at the same time.
 Tendency of the Industry to Migrate to the South:
z In South India the favourable maritime climate free from the effects of summer loo and
winter frost is best suited for growing superior varieties of cane. So there is a longer
production period.
z The black soil here is more fertile than the alluvial soil of the northern India. Besides this,
the black soil is well drained.
z The cane is of superior quality with higher yield.
z The excellent transport facilities, especially in the states of Maharashtra and Tamil Nadu, in
relation to export markets have placed these states in a very advantageous position for the
further expansion of the sugarcane industry.
z The sugarcane farms in South India have bigger area and are managed by the cooperative
societies. These cooperative societies have access to better facilities like better seeds,
fertilizers, irrigation, etc.
z Sugar is a weight losing crop. i.e., its sucrose content goes on decreasing with time. Since
the factories in southern India are located near the fields, loss of sugar content due to
transportation is minimum.
z New machinery and crushing devices ensure more yield.

REVIEW QUESTIONS AND ANSWERS

Section I: [2 Marks]
1) What are large scale industries? Give two examples.
Ans: Industries are large or small on the basis of the number of workers employed and the amount of
capital invested. Industries employing a large number of skilled as well as unskilled workers and
investing huge capital for the purchase of raw materials, machinery and other expenditures are
called large scale industries. Iron and steel, and cotton textiles are the examples of large scale
industries.
2) What is a small scale industry?
Ans: An industrial unit which has an investment of not more than Rs.7.5 lakh, irrespective of number
of workers employed is known as a small scale industry.
3) What do you understand by village and cottage industries? Give examples.
Ans: i) Industries situated in rural areas and also catering to the demand of local markets are
considered village industries, for example, khadi, khandsari, gur, vegetable oil manufacturing
antis etc.
ii) Industries in rural areas, in which the members of the family, work as artisans and craftsmen
and work with stone, brass, ivory, wood and cane, etc. are known as cottage industries.
Sometimes, cottage industries are located in towns also.
4) What are the salient features of the cottage industries in India?
Ans: i) Cottage industries in India are traditional in their techniques and production of goods.
ii) Cottage industries depend on raw materials which are available locally.
iii) Units are established in the huts, cottages or houses of the producers.
iv) Labour is not hired in these industries; rather work is done by the members of the family.
v) Cottage industries provide subsidiary occupation with agriculture or animal husbandry.
5) Classify industries on the basis of ownership.
Ans: i) Public Sector Industries ii) Private Sector Industries
iii) Joint Sector Industries iv) Co-operative Sector Industries

52 Universal Tutorials – X ICSE – Geography Volume 2 of 4


Chapter 11: Industries in India 53

6) Differentiate between public sector industries and private sector industries.


Ans: Public sector industries are owned and controlled by the government or its agency. Railways,
post and telegraphs are public sector industries. Private sector industries are those which are
owned and controlled by private houses or individuals. For example, the Birla Jute Mills is a
private sector industry.
7) ‘Agriculture and industry are complimentary to each other.’ Discuss.
Ans: Agriculture and industry depend on each other for their growth. Agriculture provides raw
materials for certain industries, while industry contributes in raising the productivity of agriculture
by providing fertilizers, pesticides, machines, electricity etc., for use in agriculture. Agro based
industries such as cotton, jute, food processing depend on agriculture.
8) What are agro-based industries? Give examples.
Ans: Those industries which use agricultural products as their raw material are known as agro-based
industries. Cotton and jute textiles, vegetable oil and sugar industries are examples of agro-
based industries.
9) Explain why there is industrial concentration in Bihar, West Bengal and Orissa.
Ans: The area of concentration of industries spread over the three states of Bihar, West Bengal and
Orissa is known as Chhota Nagpur plateau or Damodar valley belt which is rich in various
minerals. So, availability of raw material, cheap labour and a dense network of roads and
railways attract the industries in this region due to lower economic costs.
10) What is ‘sericulture’? Also mention its two types.
Ans: The rearing of silkworms to produce raw silk is known as sericulture. Fresh mulberry leaves are
fed to the silkworms and thread is unravelled from the cocoons on small spinning machines.
Sericulture is of two types: (i) mulberry and (ii) non-mulberry. The mulberry sector accounts for
nearly 90 per cent of the natural silk produced in India.
11) Name the by-products of the sugar industry. Also discuss how they are commercially
important.
Ans: The by-products are molasses and bagasse. Moslasses, dark brown syrup, is used in the
manufacture of fertilizers, alcohol, run, yeast, etc. After extraction of juice crushed sugarcane is
called bagasse. It is used as cattle feed, organic fertilizer, fuel, and raw material in paper mills.
12) With the help of an example, explain how agro based industries are different from
mineral-based industries.
Ans: Those industries which use agricultural products as their raw materials are known as agro-based
industries, for example, cotton textiles, vegetable oil and sugar industries.
Industries which depend for their raw materials on minerals are known as mineral-based
industries, for example, iron and steel and ship-building industries.
13) With the help of an example each, differentiate between Basic and Consumer industries.
Ans: Basic industries are those industries on which various other industries depend eg. Iron and steel
industry.
Consumer industries are those industries which produce various items to fulfill the day-to-day
requirement of the consumers eg. Sugar industry/ Plastic industry.
14) What industries are included under the term ‘textile industry’? What are the raw materials
for each of them?
Ans: Textile is a broad term which includes all the five fibres–cotton, jute, wool, silk and synthetic
fibre. The textile industry of India is the first manufacturing industry in India. It occupies an
important place in terms employment generation. It is the oldest among the modem
manufacturing industries.
15) What factors have favoured the localization of the cotton textile industry in Mumbai?
Ans: i) India’s first successful modern cotton textile mill was set up in Mumbai. It is known as the
‘Lancashire of India’ as the black cotton soil in the hinterland of Mumbai produces cotton
which is the basic raw material for this industry.
Volume 2 of 4 Universal Tutorials – X ICSE – Geography 53
54
ii) Mumbai’s location as an international port helps in the import of good, long–staple cotton,
machinery and the export of finished goods.
iii) Banking facilities and cheap hydroelectric power available from the Tata hydroelectric grid
from the Western Ghats are advantageous to Mumbai’s cotton industry.
16) What makes Khadi and Handloom sector of the textile industry still very important even in
this modern large-scale industrial era? Give two reasons to justify your answer.
Ans: Khadi and handloom sector of textile is competing with modern industry with its innovative
colours, styles, diversification, choice, rate and corporate ethics. The following are the reasons
of its importance:
i) It is a widespread industry which provides large employment and contributes about one-fifth
of the total cloth production.
ii) They can be started with low investment by using local raw material and local talent
encouraging optimum use of national resources.
17) Explain why, Sugar mills in the cooperative sector have an advantage over those in the
private sector.
Ans: The cooperative movement has made greater headway in the south than in north. Sugar is
seasonal industry and so labour cannot be employed in this industry throughout the year. That is
why sugar mills in cooperative sector have an advantage over those in the private sector. The
sugar mills are owned and managed by the farmers so they have work throughout the year.
There is growing demand in Northern India to nationalize the sugar industry.
18) What are cooperative industries? Which category of industries is ideally suited to it?
Ans: Industries can be classified on the basis of sources of management, raw material, size, location
and nature of product.
i) The co-operative industries are based on a management basis. These are, as the name
suggests, owned and managed on a co-operative basis by those who are producers of the
raw materials of these industries.
ii) The government has set up cooperative societies like ‘IFFCO’ and ‘KRIBHCO’. The industries
ideally suited for these are sugar mills run by farmers producing sugarcane. Also, the dairy
industry, specially in Gujarat, is a co-operative industry.
19) Name the important centres of silk in Karnataka and Assam.
Ans: In Karnataka – Bangalore and Mysore.
In Assam – Nagaon, Kamrup and Goalpara.
20) Mumbai–Pune region is the most important industrial region of India. Subtantiate the
statement giving two reasons.
Ans: Mumbai-Pune region is the most important industrial region of India because of the following
reasons: (i) Availability of raw materials (ii) Sufficient power resources, (iii) Availability of
transport facilities, (iv) Proximity of port.

Section II: [3 Marks]


1) Why is Karnataka famous for the silk industry?
Ans: The following conditions in Karnataka favour the growth of silk industry in Karnataka:
i) Suitable climatic conditions for silk production.
ii) Availability of soft water in large amount.
iii) Mulberry is grown as bush plantation.
2) Why more than 60 per cent of Sugar mills of India are located in the sugar belt comprising
Uttar Pradesh and Bihar?
Ans: It is because of the following reasons:
i) The Ganga plain has fertile soil and heavy rainfall suitable for sugarcane cultivation.

54 Universal Tutorials – X ICSE – Geography Volume 2 of 4


Chapter 11: Industries in India 55
ii) Cheap labour is easily and regularly available from this densely populated plain of India.
iii) Coal, the main source of energy used in sugar factories is available from nearby coal mines
of Bihar.
iv) This sugar belt is also a large consumer of sugar. So easy market is available.
3) Why sugar industry has shown tendency to shift towards the south? Give four reasons.
Ans: i) Sugar mills in the south are in the co-operative sector. Thus, people do more work and get
better dividends here under co-operative movement.
ii) Labour is comparatively cheaper in south India.
iii) In south, sugar mills are closer to the fields. This proximity prevents the loss of sugar content
in transportation.
iv) There is a strong sugar lobby in Maharashtra which invests in sugar industry and tries to get
maximum returns.
4) What are the four special features of the cotton textile industry in India?
Ans: The four special features of cotton textile industry in India are:
i) It is oldest and largest industry in India.
ii) It is the most widespread industry found in most of the states of India.
iii) This industry provides employment opportunities both in rural and urban areas.
iv) This industry accounts for the largest proportion of foreign exchange.
5) State four geographical factors which should be kept in mind while setting up an agro-
based industry.
Ans: Four geographical factors to be considered are as follows:
i) Proximity to growing areas.
ii) A well developed transport system.
iii) Facilities for proper storage of the raw materials as well as finished products.
iv) There should be a good demand for the product.
6) Despite being the largest producer of sugarcane, why is India ranked second in world
production of sugarcane?
Ans: i) India despite being the largest producer of sugarcane, is ranked second in its production as
the yield per hectare is low, compared to other sugarcane producers in the world. This has
resulted in low productivity and shortage of supply of sugarcane to mills.
ii) As this industry is primarily agro-based and hence seasonal, there are fluctuations in the
output of sugarcane. Fluctuations in the production of sugarcane occur because of the erratic
nature of the monsoons.
7) Why is there an increasing demand for handloom materials?
Ans: i) The Khadi and handloom industry supplies more than one-fifth of the cloth produced in the
country. These products are in great demand are relatively cheap and help to build the Indian
economy with foreign exchange earnings.
ii) These products are run by the cooperative sectors. More than half of its production comes
from local sources and with low investment. The Handloom and Khadi industry provides
employment to 100 lakh people in the country and curbs migration to cities.
8) State two non–geographical factors that determine the location of industries in India.
Ans: The infrastructure of an industry depends on geographical factors and non-geographical factors.
The two important non-geographical factors are:

Volume 2 of 4 Universal Tutorials – X ICSE – Geography 55


56
i) Capital: The most important factor in setting up an industry, as most of the industries
requires huge investments. Financiers and capitalists are available in big cities. That is why
industries are set up in metropolises.
ii) Technical know how is necessary. The government plays an important role in the planning
and location of industries, besides solving problems of public pollution and disparities. They
set up industrial zones where it supplies land, water and electricity.
9) How do the cottage industries play an important role in the Indian economy?
Ans: i) Cottage industries provide jobs to millions of people. Thus, these industries create not only
wages for people but also check their migration to urban areas.
ii) These industries can be started with low investment. So, these units help in earning
additional income.
iii) Use of local raw material in these industries helps in the optimum utilisation of national
resources.
iv) Their products earn a lot of foreign exchange for the country.
v) These industries generate seasonal as well as perennial employment for labour.
Thus, cottage industries play a significant role in our national economy.
10) Discuss the factors affecting the location of industries.
Ans: Important factors affecting the location of individual industries can be enumerated as follows:
i) Availability of raw materials: Those industries which use heavy and bulky raw material of
small value tend to be located near the source of raw material. It is because, raw material
which is weight loosing, if transported from a distance will increase the cost of the final
product. Hence, cement industry, sugar industry and iron smelting industry are attracted
towards the sources of raw materials.
ii) Power resources: To run the machinery, fuel is a must. Coal is the cheapest source of
power but it is bulky and involves high transport costs. Those industries which use large
quantities of coal are generally found near the coal mines. Electrochemical and
Electrometallurgical industries which require a large amount of cheap electric power are
installed in areas producing electricity in abundant quantity. Transmission of electricity, of
course, has helped in the dispersal of industries away from coal mining areas.
iii) Labour supplies: Each industry requires a particular type of cheap but adequate supply of
labour. Industries which require highly skilled labour are attracted towards big urban centres.
In some cases, labour with inherited skills is immobile and industry goes there to get their
services, for example, glass industry in Firozabad and dyeing and printing industry in
Farrukhabad. In industries where a large number of workers are required, labour migrates to
the industrial centre because labour is mobile and seeks bright prospectus for regular work at
high wages. Maruti car factory in Gurgaon has attracted labour from almost all parts of the
country.
iv) Transport: Cheap and efficient means of transport are essential for carrying raw materials
and labour to the factory and finished products to the market. That is why big industries are
always located near railway junctions or highways. Ports also developed as industrial centres
due to the facility of import and export at cheaper costs.
v) Market: Market is the ultimate destination of all products manufactured in industries.
Nearness to the consumers not only saves transport cost but also helps the industry to know
about changing habits and preferences of consumers.
vi) Government policies: For balanced economic development, government encourages
industries in backward areas by granting subsidies, tax concessions, loans, cheap land and
electricity, etc.
vii) Miscellaneous factors: Finance, momentum of early start, climate and personal
preferences, etc., are some of the other factors which also affect the localisation of industry.
11) Why cotton textile mills are concentrated in and around Mumbai and Ahmedabad?
Ans: The following reasons are responsible for concentration of a large number of cotton textile mills
in Mumbai and Ahmedabad:

56 Universal Tutorials – X ICSE – Geography Volume 2 of 4


Chapter 11: Industries in India 57
i) Raw materials: Raw cotton is easily available from the nearby cotton growing areas of the
Deccan plateau.
ii) Climate: Mumbai and Ahmedabad enjoy maritime climate in which thread of cotton does not
break.
iii) Source of power: The Western Ghats provide suitable conditions for the generation of cheap
hydro-electricity, required for this industry.
iv) Transport facility: Both Mumbai and Ahmedabad are connected to the rest of India by
developed means of transportation.
v) Labour: These textile centres being old, skilled and unskilled labour are easily available
here.
vi) Port facility: Mumbai and Kandla are the ports which handle import of machinery and export
of cotton textile products.
vii) Finance: There is no dearth of financial and banking institutions to make available finances
for the growth of this industry.
12) What major problems are being faced by the Indian cotton textile industry? Explain.
Ans: i) Old and outdated machinery: Cotton textile industry is one of the oldest industries of India.
So it has a major problem of old and outdated machinery which are inefficient and, hence,
uneconomic.
ii) Fluctuations in the production of raw material: Production of cotton is uncertain. It
fluctuates depending on the climatic conditions. It makes the supply of raw material irregular.
iii) Poor quality of cotton: Fine quality of cotton is not produced in India. For manufacturing fine
and costly cloth, we have to import fine quality cotton from other countries.
iv) Competition in global market: Our products face tough competition in international market
from countries like Japan, Korea, the USA and Taiwan both in cost and quality. These
countries are equipped with latest and advanced machinery.
v) Rivalry: Strikes, lock-outs and market rivalry have also made the industry sick.
13) Discuss the problems of the sugar industry in India at present.
Ans: i) It is a seasonal industry in which labour cannot be employed throughout the year. This
decreases the efficiency of workers.
ii) In general, sugarcane is grown by small cultivators who sell their produce directly to the mill
owners. Thus, farmers get less return, as they are not organised.
iii) Delay in transporting sugarcane to the mills in some parts of the country also results in the
lowering of the sucrose content.
iv) In large parts of India, especially in the north sugarcane grown is of lower quality.
v) Sugar mills are old methods of production are inefficient and uneconomical.
vi) The by-products of sugar industry are also not utilized properly.
14) What are the problems faced by the sugar industry in North India? Give any two reasons.
Ans: India ranks second in the world production of sugar. The northern sugarcane producing states
are Uttar Pradesh, Bihar, Punjab and Haryana.
i) The industry faces problems as the crushing season is short. Therefore, our sugar factories
do not have work all the year round. This leads to a higher cost of sugar compared to the one
produced in other countries.
ii) The areas producing sugarcane are far away from factories. This increases the cost of sugar
and decreases the sucrose content. The sucrose content decreases rapidly after 24–48 hours
of harvesting the cane.
15) What are the types of silk produced in India? Where is it exported? Where does India
import its silk from?
Ans: i) There are two varieties of silk (a) Mulberry and (b) Non-Mulberry.

Volume 2 of 4 Universal Tutorials – X ICSE – Geography 57


58
Mulberry silk is found in Mysore, Bangalore, Belgaum, Srinagar and Himachal. Non-mulberry
varieties are tassar, en, and muga found in Assam, Bihar, Orissa and Meghalaya.
ii) Most of the silk products are of high-grade artistic designs. A large quantity of silk goods is
exported to Sri Lanka, Singapore, Hongkong, Malaysia, USA, UK and other European
countries, East African countries, and Middle East countries.
iii) India produces about 60% of raw silk. The rest is imported from Japan and Italy. Imported silk
has a high import duty and is of superior quality.
16) With reference to Khadi, state the following:
i) What is its historical importance?
ii) What measures have been taken for its improvement?
iii) What is its role in the Indian economy?
Ans: With reference to the Khadi industry.
i) The Khadi industries are the traditional primitive industries of India. They played an important
role in India’s freedom struggle. The people adopted Khadi clothes instead of foreign goods.
These industries are carried out by the village people on a small scale.
ii) Remedial steps are taken by the government to improve the industry. They have established
the All India Handloom Board where credit facilities are available from banks and financial
institutions. They began access on mill cloth to encourage handloom production.
iii) The Khadi industry has now become a major foreign exchange earner. To boost the
economy, trade fairs are organised for Khadi products.
17) Discuss the problems faced by Handloom and Khadi industries.
Ans: a) The quality and quantity of raw materials are not satisfactory.
b) The craftsmen are poor and have no technical knowledge.
c) The looms are outdated.
d) The products are unable to keep up with the fast changing latest fashions.
e) The Khadi and handloom products face stiff competition from mill–made cloth.
f) There are no proper facilities for cheap credit.
18) Why Karnataka is famous for the silk industry?
Ans: The following conditions in Karnataka favour the growth of silk industry in the state:
i) Suitable climatic conditions for silk production.
ii) Availability of soft water in large amount.
iii) In Karnataka, mulberry is grown as bush plantation.
19) Explain why:
i) Carpet-making as a cottage industry has developed in the Kashmir valley.
ii) The pure silk handloom industry is important in Bangalore.
Ans: i) Carpet-making as a cottage industry has developed in Kashmir valley because in this area,
sheep are reared for wool which is essential material for weaving the carpets skilled
traditional craftsman is also available. Besides woollen carpets are in great demand.
ii) The pure silk industry is important in Bangalore because of the large scale rearing of silk-
worms on the mulberry leaves.

58 Universal Tutorials – X ICSE – Geography Volume 2 of 4


Chapter 11: Industries in India 59

MISCELLEOUS QUESTIONS / HOME WORK


1) What is meant by ‘agro-industry’? Name the leading agro-industry in India.
2) Mention the problems of small-scale and cottage industries.
3) Why are cottage industries important in India?
4) Mention any two special features of the cotton textile industry in India.
5) What factors have favoured the localization of the cotton textile industry in Mumbai?
6) What are the problems of the cotton textile industry?
7) Why is Kolkata an important jute textile producing centre?
8) Name two important centres for each of the following industries and state the facilities they are
enjoying at these centres:
a) Cotton Industry b) Silk Industry c) Woollen Industry d) Jute Industry
9) Name two other types of silk apart from the mulberry.
10) Why Uttar Pradesh has been relegated to the second place in terms of sugar production?
11) What are the reasons for higher output of sugar in Tamil Nadu?
PREVIOUS BOARD YEAR QUESTIONS:
1) a) Name and define two important by-products of the sugar industry.
b) Give two reasons by why the state of Punjab is the largest producer of woolen textiles.
c) Mention three main problems of the Jute textile industry in India.
d) With reference to the cotton, textile industry:
i) Which is the country’s most important manufacturing center?
ii) State two geographical reasons for its importance. [2011]
2) a) Give two reasons why the woolen industry has not developed as well as the cotton industry in
India.
b) State two factors which favour the silk industry in Karnataka.
c) Mention three problems of the sugar industry in India.
d) In what way does the cotton industry contribute to the economy of India?
Mention any three relevant factors. [2012]
3) a) Name any two large sugar producing states, one each in north and south India.
b) Name an agro-based industry based in the following industrial centres:
i) Ahmedabad ii) Mysore
c) State three favourable conditions responsible for the growth of the jute industry in West
Bengal.
d) Give geographical reasons for the following:
i) The woolen industry is a localized industry in India.
ii) It is necessary to crush sugar cane within 24 hours of harvesting.
iii) Sericulture flourished in Karnataka. [2013]
4) a) Name two textile industries using any animal fibres. Name an important State where these
industries are located.
b) Give two reasons for each of the following:
i) Kolkata is an important cotton manufacturing centre even though West Bengal is not a
leading producer of cotton.
ii) The wool industry is not as well developed as compared to the cotton industry in India.
c) Mention three problems of the jute textile industry in India.
d) Mention three factors that have helped the sugar industry flourish in the peninsular region
rather than in northern regions of India. [2014]

Volume 2 of 4 Universal Tutorials – X ICSE – Geography 59


60

Chapter 12: Mineral Based Industries


Mineral Based Industries
→ Iron and Steel Industry
→ Heavy Engineering
→ Petrochemical
→ Electronics

Synopsis:
™ Those industries which are based on minerals are called mineral–based industries. Iron and steel
industry, cement industry, and chemical industry are examples of mineral–based industries.

Iron and Steel Industry:


¾ Iron and steel industry is a basic industry and is a key to other industries.
¾ Iron is one of the most abundant chemical substances in the earth’s crust. But it is never found in
pure form in the earth’s crust.
¾ Almost all iron occurs in ores. To obtain iron, is combined in reducing agents, coke and limestone,
at high temperatures. The molten material runs off into moults or pigs.
¾ Steel is produced by refining iron and mixing it with other metals. In general, steel is an alloy of
iron and carbon.
¾ Almost all kinds of steel contain some manganese. Other elements used in alloy steel include
nickel, aluminium, chromium, copper, silicon etc.
¾ There are three chief methods of steel making – the basic oxygen process, the electric furnace
process, and the open–hearth process.
¾ Most steel is produced from molten pig iron, which contains impurities such as carbon,
phosphorous, silicon and manganese.
¾ All these impurities are reduced by melting the pig iron and oxidizing the impurities.
¾ The steel, so obtained, is cast into huge ingots which are rolled into different shapes and sizes, for
example, drain–pipes, gutter covers, weights, railings etc.
¾ In India, the beginning of the iron and steel industry was made with the setting up of Tata Iron and
Steel Company.
¾ The major iron and steel plants are at Jamshedpur (Jharkhand), Burnpur (West Bengal), Bhilai
(Chhattisgarh), Rourkels (Orissa), Durgapur (West Bengal), Bokaro (Jharkhand), Bhadravati
(Karnataka), Vijaynagar(Karnataka), Vishakhapatanam(Andhra Pradesh) and Salem(Tamil Nadu).
¾ The Tata Iron and Steel Company (TISCO) was started in 1907 at Sakchi in the Singhbhum
district of Jharkhand.
¾ The place, to set up the plant, was chosen because there were deposits of iron ore, coal and
manganese, (the raw materials) for the production of steel.
¾ The Kharki and Subarnarekha rivers were the source of water supply.
¾ The Bhilai Iron and Steel Plant, the first public sector plant, was started in 1957 in the Durg district
of Chhattisgarh.

60 Universal Tutorials – X ICSE – Geography Volume 2 of 4


Chapter 12: Mineral Based Industries 61
¾ It was set up in the collaboration with the USSR. The place, to set up the plant, was chosen
because there were deposits of iron ore, coal and limestone, for the production of steel.
¾ The other reasons for chosen the place were availability of cheap and abundant labour, the river
Tendula for water supply, Korba thermal power station for electricity supply and Kolkata–Nagpur
railway line for transportation purpose.
¾ Durgapur Steel Plant was started in 1959 at Durgapur in the Burdwan district of West Bengal. It
was set up in the plant, to set up the plant, was chosen because of the following factors:
¾ Easily availability of raw material such as iron ore, coal, limestone, dolomite etc.
¾ Damodar River for water supply.
¾ Abundant labour and good transport facilities.
¾ Roukela Steel Plant was started in 1959 in Sundargarh district of Orissa. It was set up with the
help of a German firm Krupps and Deemang. The place to set up the plant was chosen because
raw material such as iron ore, coal, and limestone are easily available.
¾ Cheap hydroelectricity is available from Hirakud power project and water supply from Brahmani
River.
¾ It is situated on Mumbai, Kolkata railway route.
¾ Bokaro Steel Plant was established in 1972 with USSR. It is located in the Hazaribag district of
Jharkhand. The geographical factors for setting up this plant are as under:
¾ Raw material is easily available.
¾ Anundant labour available from nearby areas.
¾ Cheap and regular hydroelectricity and water are available from Damodar river
¾ Indian Iron Steel Company (IISCO) came under public sector in 1972. It produces rails, hot–rolled
galvanized sheets.
¾ Vijayanagar Steel Plant has been set up at Hospet near Bellary district of Karnataka.
¾ The Vishakhapatnam Steel Plant in Andhra Pradesh and the Salem Steel Plant in Tamil Nadu are
the two new steel plants of India.
¾ The major problems which are being faced by iron and steel industry are as under:
 Lack of capital investment.
 Lack of good quality coal.
 Lack of sophisticated and modern techniques of steel production.
 Lack of specialization of producing items.
 Low quality of products has not been able to complete in international market.
 Less demand due to high cost, problems of skilled labour, inadequate supply of power, and low
quality of iron–ore are some of other problems being faced by iron and steel industry of India.

Heavy Engineering Industries:


¾ As the name depicts, heavy industries require bulky raw material, huge capital for investment, a
large number of cheap and technically skilled labour.
¾ Water is also one of the basic requirements of industries but heavy industries require water in
huge quantity.
¾ Automobiles, Railway Locomotive, Ship–building, Heavy machines and Tools, Aircrafts, Heavy
Electrical Industry.
¾ Automobile industries manufacture or assemble automobiles such as buses, cars, trucks,
motorcycles, scooters tractors, etc. India is at seventh place in the production of automobiles.

Volume 2 of 4 Universal Tutorials – X ICSE – Geography 61


62
¾ Maruti Udyog Limited, Ashoka Leyland, and Hindustan Motors Ltd. are some of the automobile
industries.
¾ Railway Locomotives produce coaches, wagons and engines. This industry requires a bulk of iron
and steel. The Diesel Locomotive Works at Varanasi in U.P., Chittranjan Locomotive Works in
West Bengal. The Integral coach Factory at Perambur in Tamil Nadu. The Rail Coach Factory at
Kapurthala in Punjab are the public sector units of Railway Locomotives.
¾ The ship–building industry assembles ships by purchasing some parts from other industries and
producing some by of its own. The four major public sector shipyards are the Cochin Shipyard at
Kochi, Hindustan Shipyard Ltd. at Vishakhapatnam.
¾ Aircraft industries manufacture aircrafts such as MIG or Pushpak. The Hindustan Aeronautics
Limited is an aircraft industry which has its units at different places like Kanpur, Bangalore, and
Lucknow.
¾ Heavy Machines and Tools, as its name depicts, produces machines and tools used by other
industries such as textile, cement, fertilizers, paper, etc.
¾ The Hindustan machine Tools, Bangalore, Heavy Engineering Corporation of India and the Praga
Tools Limited Secunderabad are examples of Heavy Machines Tools Industry.

Heavy Electrical Industry:


¾ It consists of equipments used for the generation, transmission, distribution and utilization of
power.
¾ It includes items such as generators, boilers, turbines, transformers, switch gears, cables etc.
¾ There are three companies in the public sector for manufacturing these equipments:
 The Heavy Electrical Ltd. at Bhopal.
 The Bharat Heavy Electrical Ltd. (Six units)
 The Industan Cables Factory – Ruprarainpur, West Bengal.
Petrochemical:
 Petrochemical industries produce petrochemicals, i.e. chemicals derived from petroleum or
natural gas.
 Petrochemicals are used in making of such products as detergents, fertilizers, medicines,
paints, plastics, synthetic fibres, and synthetic rubber.
 The basic materials of the industry are ethylene, propylene, butadiene, benzene, toluene,
xylene, and gases carbon monoxide and hydrogen.
 Indian Petrochemicals Corporation Limited (IPCL) and Petrofils Co–operative Limited (PCL)
are two big organizations of petrochemical industry. IPCL manufactures petrochemicals like
polymers, and PCL manufactures polyester filament yarn.
 The development, manufacture, and sales of electronic products make up one of the largest
and most important industries in the world.
Electronic Industry:
 The electronic industry is also one of the fastest growing of all industries. The Bharat
Electronics Limited, the electronics Corporation of India Limited and the Indian Telephone
Industries, etc, are electronic industries.
 The Bharat Electronics Limited produces defence equipments. The Electronics Corporation of
India Limited produces transistorized modular systems for nuclear applications and for use in
medical, agri. Ind. field.
 The Indian Telephone Industries produces equipments required by post and telegraph
departments and overseas communication services.

62 Universal Tutorials – X ICSE – Geography Volume 2 of 4


Chapter 12: Mineral Based Industries 63

REVIEW QUESTIONS AND ANSWERS:

Section I: [2 Marks]
1) What are key and consumer industries? Give examples.
Ans: i) Those industries on which so many other industries depend are called basic or key
industries. For example, iron and steel industry or heavy machinery industry are key
industries as they provide steel and machines to rest of the industries.
ii) Consumer or secondary industries are those which produce primarily goods for consumption
by people. TV, washing machines, electronic items, bicycles, furniture, watches, utensils,
cosmetics etc. are the examples of the consumer industry.
2) How do you differentiate between heavy and light industries?
Ans: Industries which use heavy or bulky raw materials and also produce bulky products are known
as heavy industries, for example, iron and steel industry. On the other hand, industries whose
raw materials and finished products both are not heavy are called light industries, for example,
computer, T.V. or watch manufacturing units.
3) Why is Iron and Steel Industry considered a basic or key industry?
Ans: Steel industry is considered a basic industry because all other industries depend on it for their
machinery.
Steel is required in the manufacture of all types of machines and their products right from a small
safety pin to a huge ship. Steel is omnipresent in all equipments, engineering goods and
household items.
4) When was the first steel plant set up in India?
Ans: The history of the modem iron and steel industry in India began in 1870 when the Bengal Iron
Works Company set up its plants at Kulti (Burnpur)in West Bengal.
5) What are the four sections of an integrated iron and steel plant?
Ans: i) Mixing of raw materials: In this section, iron ore, coking coal and limestone are mixed in the
proportion of 4 : 2 : 1. Manganese is also mixed.
ii) Making pig iron from iron ore: In a high steel structure, called blast furnace, iron ore is
changed into pig iron by melting.
iii) Purification: Impurities of pig iron are oxidized to convert it into steel.
iv) Rolling of steel: Huge ingots of steel are rolled into different shapes and sizes such as rods,
beams, sheets, plates, nails, etc.
6) What is the difference between an Integrated Steel Plant and a Mini Steel Plant?
Ans: In an integrated steel plant, all the four processes are carried out in one complex from mixing of
raw material to shaping the metal.
Mini steel plants, on the other hand, do not do all the activities rather they use steel scrap in
electric furnaces and make liquid steel, which later on is turned into steel ingots.
7) a) Where is the Indian Iron and Steel Company (IISCO) located?
Ans: The IISCO is located at Burnpur near Asansol in West Bengal. It is about 210 km away from
Kolkata.
b) When was the IISCO nationalized?
Ans: Originally, this plant was set up in the private sector, 1874. It was taken over by the Government
on 14 July 1904 with a view to improve its working. The company is now wholly owned
subsidiary of the SAIL.
8) Discuss the locational advantages available to the IISCO.
Ans: i) High quality iron ore is obtained from Chhota Nagpur Plateau which is 277 km away.
ii) Fuel in the form of coal is obtained from Raniganj and Jharia coalfields.
iii) Gangpur in Orissa supplies limestone.
iv) Water is available from the Damodar River which is just 4 km away from this plant.
v) Kulti and Bumpur are well connected with roads and railways.
vi) Finished products are imported through Kolkata port.
Volume 2 of 4 Universal Tutorials – X ICSE – Geography 63
64

9) Where is the Bokaro Steel Plant located? When and with whose help this plant was set
up?
Ans: The Bokaro Plant is located in Bihar on the right bank of the Damodar River, near the confluence
of the Damodar and Bokaro rivers. This plant was established in 1972 with Soviet assistance.
10) Discuss the location and geographical advantages available to the Vijaynagar Steel Plant.
Ans: The Vijaynagar Steel Plant has been set up near Hospet in Bellary district of Karnataka.
i) High quality iron ore is available in Karnataka.
ii) Coal is obtained from Madhya Pradesh and Andhra Pradesh.
iii) Water is available from Tungabhadra Project, just 32 km away.
iv) Limestone and dolomite are available within a distance of 200 km.
v) The Karnataka Electricity Board provides 130 to 150 MW of electricity.
11) What are heavy inorganic chemicals? Also discuss their uses.
Ans: Sulphuric acid, nitric acid, soda ash and caustic soda are included in heavy inorganic chemicals.
Sulphuric acid is used in synthetic fibres, plastics, paints, and fertilizers.
Caustic soda is used in the manufacture of paper, soap, rayon, textiles, rubber, and in oil
refineries. Soda ash is used in glass, paper, soap, textiles, and detergents.
12) State two advantages of Petrochemicals. Name any two Petrochemical products.
Ans: Two advantages of Petrochemicals are that they are used as basic material in many industries
and in manufacturing synthetic fibres. Two petrochemical products are lubricating oil and plastic.
13) i) State the importance of the Heavy engineering Industries in the industrial development
India.
ii) Mention two main requirements of Heavy Engineering Industries.
Ans: i) Heavy Engineering Industries provide us with product essential for infra-structural growth of
the nation. They provide us with heavy vehicles, railways and machines without which
industrial growth is not possible.
ii) Two main requirements of Heavy Engineering Industries are: (a) Proximity to raw material
centres. (b) Easy availability of large capital.
14) What is the status of India’s automobile industry? State two aspects.
Ans: i) The real development of the automobile industry began in 1947 with the establishment of
Premier Automobiles Ltd. at Mumbai, and Hindustan Motors Ltd. at Uttarpara, Kolkata.
Ever since then it has progressed. It tends to be located near areas of the iron and steel
industry.
ii) Port cities are considered favourable locations for export-import facilities. The industry has
become market oriented requiring ready market facilities for cartage, sales, banking credit
and skilled labour.
15) Why are petrochemical industrial units found mainly in Maharashtra and Gujarat? Give
two reasons.
Ans: i) Petrochemicals are those chemicals and compounds which are derived from petroleum
resources. The units are mostly found in Maharashtra and Gujarat as these regions are
mostly abundant in petroleum and coalfields.
ii) The petrochemicals raw materials are easily available e.g. for the Udex plant at Koyali
(Gujarat) raw material is available from the nearby refinery. It is also economically stable in
these two states.
16) Name four labour intensive industries.
Ans: i) The industries can be classified on the basis of the source of its raw materials, management,
size, location, nature of products. There are large scale medium and small-scale industries.
ii) In the labour-intensive industries local labour is employed. They use hands instead of
machines for manufacturing. Such kinds are (i) bidi making, (ii) cracker making, (iii) shoes
and (iv) carpentry.

64 Universal Tutorials – X ICSE – Geography Volume 2 of 4


Chapter 12: Mineral Based Industries 65

17) Where are the proposed steel plants being located?


Ans: i) The iron and steel industry is an index of a nation’s business activity. Therefore it is a basic
industry.
ii) Three more iron and steel plants are being constructed by the government at three locations,
at Bailaidila in Visakhapatnam, Vijaynagar in Andhra Pradesh and at Salem in Tamil Nadu.
The plant at Salem was set up to produce stainless steel sheets.
18) State three advantages of the decentralisation of industries.
Ans: The decentralisation of industries has the following advantages:
i) It provides employment opportunities in every region of the country thus keeping a check on
the concentration of population in certain pockets in the country.
ii) It removes regional disparities in the task of industrialization of the country.
iii) It leads to a more equitable distribution of national income in the country.
19) What factors determine the location of petrochemical industries? State two of them.
Ans: i) Petrochemicals are those chemicals and compounds which are derived from petroleum
resources. The industry derives its raw materials from coal and LPG. The factors determining
this industry are its major raw materials collected from coal and petroleum fields. These
industries are concentrated in petroleum and coal field regions.
ii) They are cost effective, and economically stable, and cheaper as they are produced on a
mass scale. Its raw material is easily available. Fertilizers are produced from it.
Short Answers: (3 Marks)
20) a) Differentiate between mini steel plant and integrated steel plants.
Ans:
Integrated steel plant Mini steel plant
An integrated steel plant is one where all Steel plants that do not process at one plant
processes are carried out in one complex are known as mini steel plants or furnaces.
from handling of raw material, coke-making, They use steel scrap in the electric furnaces
steel making and rolling and shaping of the to make liquid steel which is turned into
steel. ingots.
b) State two advantages of integrated steel plants.
Ans: i) All the processes of steel making and steel shaping are done at one complex.
ii) Products are produced on large scale. So cost of production is less.
c) Mention main disadvantages of mini steel plants.
Ans: i) Products are not produced on large scale so cost of production is high.
ii) Big machines can not be manufactured in mini steel plant.
21) Give four reasons why the iron and steel Industry is concentrated in the northeastern part
of the country.
Ans: The iron and steel industry is concentrated in the north-eastern part of the country due to the
following reasons:
a) All the essential raw materials (iron ore) etc. is available in these areas.
b) There is a good network of railways and roads in these areas so finished goods can be easily
distributed with in the country and also exported to other countries.
c) Labour, power and other facilities are also available in this region.
d) The water supply is made by the Damodar River and its tributaries.
22) Describe in brief about the Bhilai Steel Plant, with reference to location, raw materials
Ans: It is located at Bhilai in Madhya Pradesh. It was set up with the help of U.S.S.R. It enjoys the
following localisation facilities:
a) It is situated on the Kolkata-Nagpur railway line. It provides transport facilities.
b) It meets its iron ore requirements from the iron ore mines of Durg, Chanda and Bastar district
of M.P.
c) It gets coking coal from Korba coalfields of M.P. and Jharia coalfields of Bihar. It also gets
electric power from Korba Thermal Power Station.

Volume 2 of 4 Universal Tutorials – X ICSE – Geography 65


66
d) The manganese comes from Bhandara and Balaghat districts of M.P. and limestone from
Nandini mines.
e) It gets water from Tandula and Gondli water reserviors.
f) The Vishakhapatnam Port provides the sea outlet facilities.
g) It gets thermal power from Korba Thermal Power Project.
Main Products: (a) Rails (b) Pig iron (c) Merchant products (d) Heavy structural.
23) With reference to the Visveswaraya Iron and Steel Plant, answer the following questions:
i) What was the ‘status’ and former name of the Visveswaraya Iron and Steel Plant?
ii) Where is it located?
iii) What is its specialty?
iv) Name the power stations that supply electricity to this plant.
v) What does it get from Mandi Gudda, 32 km away?
vi) From where does it get its other raw materials?
Ans: i) The Visveswaraya Iron and Steel Plant was formerly a private sector company named
Mysore Iron and Steel Company.
ii) It is located at Bhadravati in Karnataka.
iii) It specializes in producing pig iron, steel castings and special steel and alloy.
iv) Electricity to this plant is supplied from the Mahatma Gandhi Hydroelectric Power Station and
Sharavati Hydroelectric Power Plantation.
v) It gets limestone from Mandi Gudda, 32 km away.
vi) It gets its other raw materials from following places: Iron-ore: Kemmanguddi near Bhadravati.
Manganese: Madhya Pradesh and Maharashtra.
24) Why most of our steel plants are concentrated around Chota Nagpur Plateau region?
Ans: The following geographical factors have contributed to the localisation of this industry around
Chota Nagpur Plateau region:
i) Iron ore, the basic raw material for this industry, is easily available from mines spread over
Jharkhand, West Bengal and Orissa.
ii) Jharkhand and West Bengal are the home of coal, the cheap source of energy for the
industry.
iii) Other minerals required for iron and steel industry, e.g., manganese, dolomite and mica, etc.,
are available in Chota Nagpur Plateau.
iv) Regular supply of power is made available from nearby Damodar Valley Corporation.
v) Cheap labour is available from the tribal areas of Jharkhand, Bihar, Madhya Pradesh,
Chhattisgarh and Orissa.
vi) Financial institutions in Bihar; Jharkhand, and West Bengal provide ample finance to this
industry.
vii) This region is connected to the rest of India by important roads and railway routes.
viii) From Kolkata port there is a facility of importing machinery and exporting products.
ix) Policy of the Government in localisation of public sector steel plants in this backward region
has also been responsible for the concentration of steel plants around Chhota Nagpur
Plateau.
25) What are the salient features of Mini Steel plants in India?
Ans: There are more than 216 mini steel plants in addition to large–scale integrated iron and steel
plants. These steel plants have the following characteristics:
i) Mini steel plants use electric arc furnace and induction furnace to produce steel. Thus, coal is
conserved.
ii) These units use steel scrap/ sponge iron from big steel plants as raw material and, thus, help
in recycling of iron and its optimum utilization.

66 Universal Tutorials – X ICSE – Geography Volume 2 of 4


Chapter 12: Mineral Based Industries 67
iii) Mini steel plants do not require huge investments as they do not carry out all the processes of
steel making. They can produce mild steel, alloy steel including stainless steel.
v) Small steel plants can be easily constructed in industrial towns to meet the special steel
requirement reducing transport costs.
26) What are the locational advantages available to the Visveswaraya Steel Plant in
Bhadravati?
Ans: The locational advantages are as follows:
i) Iron ore is available to this plant from Kemmangundi mines in Bababudan Hills just 41 km
away.
ii) Limestone is available from Mandiguddu quarry which is 23 km to the east of this plant.
iii) Manganese is made available from Madhya Pradesh, Chhattisgarh and Maharashtra.
iv) Hydroelectricity is available from the Mahatma Gandhi Hydroelectricity Project and the
Sharavati Hydroelectricity Project.
v) Requirements of water are met from Bhadra as this plant is situated on the left bank of this
river.
vi) Bhadravati is connected by railways with ports and important centres of trade and commerce.
27) What suitable geographical conditions are available at Jamshedpur Steel Plant?
Ans: The Tata Iron and Steel Company (TISCO) is the oldest steel plant of India. Its location at
Jamshedpur is due to the following factors:
i) The basic raw material iron ore is available from Mayurbhanj (Orissa) and Singhbhum
(Jharkhand). This bulky raw material, which is required in large quantities being in proximity,
saves the transport cost.
ii) Manganese is available from Keonjhar and Mayurbhanj and limestone from Gangpur
(Orissa).
iii) Coal, a cheap source of energy, is obtained from Jharia (Jharkhand) and hydroelectricity from
the Damodar Valley Corporation (DVC).
iv) Huge requirements of water are met from Subarnarekha and Kharkoi rivers where at their
confluence; this steel plant is set up.
v) Jamshedpur is connected by Kolkata-Mumbai railway route.
vi) Kolkata port which handles its export and import is also not very far.
28) What are the situational advantages available to the Bhilai Steel Plant?
Ans: i) High grade iron ore is available from the district of Durg, Chanda and Bastar. The Dhalli-
Rajhora Range is the famous source of iron ore just 97 km away.
ii) Coal is obtained from Jharia, Raniganj and Korba coal-fields.
iii) Manganese ore is obtained from Balaghat district.
iv) Limestone is supplied from Nandini quarries which are about 25 km away.
v) The Korba Thermal Power Station provides cheap electricity.
vi) This plant gets water from the Tendula Canal.
vii) Bhilai is connected with Mumbai-Kolkata railway route.
29) a) Where is the Durgapur Steel Plant located? When and with whose help it started
production? (b) What are the facilities available to the Durgapur Steel plant?
Ans: a) The Durgapur Steel Plant was set up in Burdhman district of West Bengal with the help of
British, the former West Germany and the former Soviet Union. It started production in 1959.
b) i) The Durgapur steel plant obtains iron ore from Singhbhum (Jharkhand) and Keonjhar
(Orissa).
ii) Coal mines of Raniganj and Jharia are just 111 km away from this plant.
iii) Limestone and dolomite are obtained from Birmitrapur and manganese from Barbil in
north Orissa.
iv) The D.V.C. provides electricity and water required by this steel plant.
v) Durgapur is located on Deihi-Kolkata railway line.
vi) It is connected by a canal with the Hooghly river and Kolkata port.
Volume 2 of 4 Universal Tutorials – X ICSE – Geography 67
68

30) a) Where is the Rourkela Steel Plant located? With whose assistance this plant was set
up?
Ans: The Rourkela Steel Plant is located in Sundergarh district of northern Orissa. It was set up with
the help of a German firm Krupp and Demog.
b) What are the geographical advantages available to the Rourkela Steel Plant?
Ans: i) This plant obtains iron ore from Bonaigarh, Mayurbhanj and Keonjhar districts.
ii) Coal is available from Jharia, Talcher and Korba coal-fields.
iii) Limestone from Birmitrapur and dolomite from Hirri quarries in Madhya Pradesh are
available.
iv) Cheap hydroelectricity is available from the Hirakud Power Project and huge amount of water
from the Brahmani River.
v) Rourkela is situated on Mumbai-Kolkata railway route.
31) What are the geographical factors which determine the location of Bokaro Steel Plant at
Bokaro?
Ans: i) The Bokaro Steel Plant gets iron ore from Noamundi, Bonaigarh, Barsha and Kiriburu.
ii) Bokaro and Jharia coal-fields which are 65 km away from this plant supply coal.
iii) Limestone and dolomite are available from Bhavanathpur and Palamau,
iv) Cheap and regular hydroelectricity and water are available from the Damodar River.
32) What are the major problems being faced by iron and the steel industry of India?
Ans: The major problems faced by iron and steel industry of India are as follows:
i) Lack of good quality coking coal.
ii) Per worker lower production in the plants of the SAIL.
iii) Lack of sophisticated and modem techniques of steel production.
iv) Lack of specialisation in producing items.
v) Low quality of our products has not been able to compete in international market.
vi) Less demand due to high cost, problems of skilled labour, inadequate supply of power and
low quality of iron ore are some of the other problems being faced by iron and steel industry
of India.
33) Answer the following questions on the Chemical Industry of India:
a) What is the importance of the chemical industry in India?
Ans: i) The chemical industry provides raw material to other industries like textiles, leather, soap,
paints, varnishes, detergents, rubber, medicines, dyes, glass, fertilizers, etc.
ii) Export of chemicals and drugs helps to earn foreign exchange for India.
iii) By its several products this industry helps in raising the standard of living.
b) Name the centres of heavy chemicals in India.
Ans: Centres for the manufacture of heavy chemicals are: Kolkata, Mumbai, Kanpur, Delhi, Chennai,
Amritsar and Bangalore.
34) With reference to the iron and steel industry in India give logical explanations for the
following:
i) The location of this industry is governed by its close proximity to raw material.
ii) This industry is not found in western India.
iii) Most of the iron and steel’ plants are ‘Public Sector Undertakings.
iv) Mini steel plants are becoming more popular.
Ans: i) Iron and steel industry use heavy and bulky raw material of small value so it is located near
the source of raw materials to reduce its high transport costs.
ii) This industry is not found in western India because of lack of raw material such as iron ore in
this region.

68 Universal Tutorials – X ICSE – Geography Volume 2 of 4


Chapter 12: Mineral Based Industries 69
iii) Most of the iron and steel plants are Public Sector Undertaking because of the following
reasons:
a) An iron and steel plant requires huge investment, basic infrastructure, particularly efficient
means of modem transport and communication.
b) It does not create enough jobs commensurate with huge investment.
c) It requires continual updating of technology.
d) A long waiting time before it begins to yield dividends.
iv) Mini steel plants are becoming more popular because they do not do all the activities rather
they use steel scrap in electric furnaces and make liquid steel, which later on is turned into
steel ingots. They can produce mild steel, alloy steel including stainless steel.
35) Why are most of the heavy engineering industries connected to the Chota Nagpur region?
Give three reasons.
Ans: i) The heavy engineering industries are set up in the Chota Nagpur Plateau region as it requires
heavy and bulky raw materials. The Heavy Engineering Corporation Ltd. in Ranchi produces
castings, forgings and rolls for tools and building plants for the iron and steel industry found
here.
ii) They require enormous amounts of power which is generated from the Dye, i.e. Damodar
Valley Project, for this industry.
iii) Intensive labour is required providing considerable employment in this area. These are
available in the states of Bihar, Orissa and West Bengal. Cheap labour is found due to
illiteracy and high population here.
36) Name three production units of the petrochemical industry, in terms of their location, and
their products.
Ans: Petrochemicals are those chemicals and compounds which are derived from petroleum
resources. The three production units are:
i) The first petrochemical complex was established by Union Carbide India Ltd. at Trombay.
The technology and management is done by the multinational. It produces polypropylene,
ethyl acetate, butyl spirit, etc.
ii) The Indian Petrochemical Corporation Ltd. (IPCL) at Jawaharnagar near Vadodara in Gujarat
manufactures and distributes various petrochemicals such as polymers, synthetic organic
chemicals and polyester fibres.
iii) Petrofils Cooperative Ltd. (PCL) is a joint-venture company of the Government of India and
the Weavers Cooperative Society. It produces polyester filament yarn at its three plants
located at Vadodara, Naldhari in Gujarat.
37) What is a consumer industry? Give an example of it. What does growth of a consumer
industry indicate?
Ans: i) Consumer industries are also called secondary industries. These industries are the ones that
convert the raw materials into primary goods for direct use by the consumers.
ii) Bakeries, paper, textiles and sugar are consumer industries. These use the raw materials
directly for the finished products.
iii) The growth in this industry leads to growth in the fast food industry leading to changes in the
consumer’s nutrient intake and public health. It leads to a surge in spending and has a direct
effect on the consumer price index.
38) Why was Bhadravati in Karnataka selected for setting up a steel plant? Give two reasons.
Ans: The factors responsible for the setting up of the Bhadravati or Visveswaraya Iron and Steel Plant
at Bhadravati in Shimoga district of Karnataka are:
i) The availability of raw materials like iron ore is obtained from Kemmangundi in Chikmagalur
district, Manganese comes from Shimoga and limestone from Bundiguda that is just 25 kms
away.
ii) Water is obtained from the Bhadravati River. Hydroelectric power is from the Sharavati and
Mahatma Gandhi hydroelectric projects; charcoal was earlier used for smelting iron ore. The
plant is situated on the main railway line.
Volume 2 of 4 Universal Tutorials – X ICSE – Geography 69
70

39) With reference to the development of the aircraft industry in India, state the following:
i) Why is it only in the government sector?
ii) Three locations and their manufactures.
iii) Two other uses of these aircraft other than for defence purposes.
Ans: Aircraft Industry
i) The aircraft industry started with a private company, but was taken over by the government in
1942. Different parts of the aircraft are manufactured at different places due to security
reasons.
ii) The three main manufacturing centres of HAL are
a) Bangalore: Aerospace division, Aircraft division, Engine division, Foundry and forge
division, Overhaul division, Helicopter division
b) Kanpur: Transport Aircraft division
c) Lucknow: Accessories division
d) Nasik: Aircraft division
e) Hyderabad and Korwa: Avionics division
f) Koraput: Engine division
iii) Aircraft are used for defence purposes. Other than that they are used for transport with
aircraft for this purpose manufactured at Kanpur. The Bangalore complex manufactures Light
Krishak and Pushpak aircraft used for agricultural purposes. They also produce the Kiran jet
trainer, jet fighters and helicopters.
40) State the importance of Electronics in the field of space technology and entertainment.
Ans: Electronics is a modern industry. It has given new dimensions to medical treatment, space,
communication equipment, and entertainment industries.
i) Our space technology is supported by the electronic industry. We have successfully launched
indigenously built satellites such as the Apple, and INSAT series. The Indian Space
Research Organisation (ISRO) and National Remote Sensing Agency (NRSA) have become
pillars of this industry where components and subsystems are electronic.
ii) Entertainment: The television and audio industries too bloomed in 1990 as a result of the
progress made by the electronic industry. BPL, Videocon, Onida and Philips together had a
market share of 83% in TV sales, the audio industry also flourished. The main centres are
Mumbai, Kolkata, Chennai and Pune.
41) State three important factors that govern the setting up of a heavy engineering industrial
region.
Ans: i) The heavy engineering industries are set up according to certain factors. As they use heavy
and bulky raw materials so they should have close proximity to the raw material used.
ii) Cheap and highly skilled labour which is technically sound. These are generally brought from
nearby villages. The industry also requires good means of transport as there is large cost
involved.
iii) They require enormous amounts of power be it hydel or thermal. So the power plants should
be available to these regions.
42) Answer the following questions on the Chemical Industry of India:
a) Where are organic chemicals manufactured in India?
Ans: Organic chemicals are produced at Mumbai, Kolkata, Jamshedpur, Kulti, Jharia, and Hirapur.
b) Where in India are fine chemicals produced?
Ans. Fine chemicals in India are produced at:
i) The Hindustan Antibiotics Ltd., Pimpri (Maharashtra).
ii) The Synthetic Drugs Plant at Hyderabad.
iii) The Hindustan Organic Chemicals at Kolaba.

70 Universal Tutorials – X ICSE – Geography Volume 2 of 4


Chapter 12: Mineral Based Industries 71

43) a) What does Ashoka Leyland produce? Name three centres where its units are
established.
Ans: Ashoka Leyland produces trucks and buses. Its units are at Chennai (Tamil Nadu), Jamshedpur
(Jharkhand), and Pune (Maharashtra).
b) Name a few well-known automobile industries.
Ans: Maruti Udyog Limited, Ashoka Leyland, Hindustan Motors Limited, Premier Automobiles etc.
c) What does Maruti Udyog Ltd. produces? Where are its units?
Ans: Maruti Udyog Ltd. produces a range of cars such as Maruti 800, Zen, etc. Its units are at
Gurgaon in Haryana.

MISCELLEOUS QUESTIONS / HOME WORK


1) Name the basic raw materials used in iron and steel industry.
2) Which is the largest mineral-based industry in India? Why is it called a key industry?
3) List the three basic steps involved in steel making.
4) What factors affect the location of an integrated steel plant?
5) What is an integrated steel plant?
6) What are mini steel plants?
7) Name the major steel plants in the Public Sector.
8) Why is the iron and steel industry concentrated in the Chotanagpur region? Which is the oldest
and privately owned steel plant?
9) What are the main requirements needed for setting up a heavy engineering industry?
10) When and where was the machine tool industry started?
11) What are the basic requirements of the automobile industry?
12) Why are traditional raw materials being replaced by petrochemical products?

PREVIOUS YEARS BOARD QUESTIONS:


1) Give two reasons to explain why there is a need for rapid industrialization in India. [2000]
2) Why is there an increasing demand for handloom materials? [2000]
3) Name a state famous for mulberry silk. [2001]
4) Mumbai-Pune region is the most important industrial region of India. Substantiate the statement
giving two reasons. [2001]
5) Give two reasons to show why the sugar industry has flourish in Uttar Pradesh and Bihar.
[2002]
6) Give two reasons for the following:
“The silk handloom industry is important in Mysore. [2002]
7) Give four reasons to justify that the rayon textile industry in India has a great future. [2003]
8) Why is the woolen textile industry not as well developed as the cotton textile industry in India?
[2003]
9) With the help of an example each, differentiate between Basic and Consumer Industries. [2004]
10) The ‘Khadi and Handloom Sectors of the Textile Industry cannot be ignored’. Give two reasons
justifying this statement. [2004]
11) Give reasons why:
a) The woollen industry is concentrated in North India.
b) Tree plantation is essential in and around Heavy Industrial areas. [2005]
12) State four geographical factors which should be kept in mind while setting up an agro-based
industry. [2005]
13) Mention three main problems faced by the cotton textile industry in India. [2005]
14) Name three by-products of the sugar industry. Give one important use of each. [2005]
Volume 2 of 4 Universal Tutorials – X ICSE – Geography 71
72
15) Mention two problems faced by the sugar industry in India. [2006]
16) State two steps taken by the government to overcome the problems faced by the jute industry in
India. [2006]
17) Mention three factors that favour the concentration of the cotton textile mills in Mumbai. [2006]
18) State three reasons why the woollen textile industry has not developed in India. [2006]
19) a) State two reasons of the concentration of the sugar industry in Uttar Pradesh.
b) Mention two ways in which the agro-based industries have affected the economy of India.
c) i) Which is the largest jute producing State in India? Name two centres of jute industry in
that State.
ii) Name two jute products.
d) State three main problems fed by the cotton textile industry in India. [2007]
20) a) From where does the iron and steel plant of Bhadravati get its iron-ore?
b) Copy the names of the following two places and write the name of the most important product
associated with each:
i) Raniganj ii) Digboi. [2000]
21) a) i) Name the foreign collaborators of the following iron and steel plants:
1) Bhilai 2) Rourkela 3) Durgapur 4) Bokaro.
ii) Explain the term ‘mini-steel plant’.
iii) Give any two reasons favouring the location of iron and steel plants in North-Eastern part
of Deccan.
b) Give one important centre of production for each of the following:
i) Diesel Locomotive ii) Aircraft iii) Ship-building. [2001]
22) a) With reference to Tata Iron and Steel Company, answer the following:
i) When and where was it set up?
ii) From where does it get its supply of iron ore, coal, limestone and manganese?
b) Name the four centres of iron and steel in the public sector which are located in a single
geographical region. With whose collaboration was each one of them set up? [2002]
23) With reference to the iron and steel industry in India, give logical explanations for the following:
a) The location of this industry is governed by its close proximity to raw material.
b) This industry is not found in western India.
c) Most of the iron and steel’ plants are ‘Public Sector Undertakings.
d) Mini steel plants are becoming more popular. [2003]
24) With the help of an example, explain how agro-based industries are different from mineral-based
industries. [2003]
25) State two main uses of heavy chemicals. In which two industries are fine chemicals mainly
used? [2004]
26) Why are the Iron and Steel Industries concentrated in the Damodar Valley region? [2004]
27) With reference to the Cement Industry in India, answer the following questions:
a) State the importance of this industry.
b) Name two important raw materials used in the industry.
c) Name two centres of the industry in Haryana.
d) Mention any two problems faced by the industry. [2004]
28) State the importance of Electronics in the field of:
a) Space Technology b) Entertainment. [2005]
29) Where do the Bhilai Iron and Steel Industry get its supply of?
a) Iron-ore b) Coal c) Limestone d) Manganese. [2005]
30) a) State the importance of the Heavy Engineering Industries in the industrial development of
India.

72 Universal Tutorials – X ICSE – Geography Volume 2 of 4


Chapter 12: Mineral Based Industries 73
b) Mention two main requirements of Heavy Engineering Industries. [2005]
31) State two advantages of Mini Steel plants. [2006]
32) a) State two geographical factors which were taken into consideration for the setting up of the
Durgapur Iron and Steel Plant.
b) Mention any one problem faced by the Iron and Steel Industry in India. [2006]
33) Name the following in India:
a) A product of the heavy engineering industry.
b) Two petrochemical units.
c) A centre of the silk industry. [2006]
34) a) Mention two reasons for the concentration of steel plants in the Chota-Nagpur Plateau region.
b) Name four petrochemical products.
c) What is the importance of the Heavy Engineering industry for the industrial development of
India? Name two Heavy Engineering industrial units in India.
d) i) What are integrated steel plants?
ii) Name one integrated steel plant in the Public Sector. From where does this plant get its
requirement of iron ore and coal? [2007]
35) a) Mention two reasons why the sugar industry has developed in Maharashtra.
b) Give two reasons for the importance of the silk industry in India.
c) Give three factors that favour the cotton industry in Kolkata.
d) i) Explain why sugarcane must be crushed within 24 hours of harvesting.
ii) Name four sugar milling centres in the northern plains. [2008]
36) a) Mention two reasons for the importance of the Electronic industry in India’s development.
b) What is a petrochemical industry? Mention two reasons why petrochemical products are
replacing traditional raw materials.
c) i) Mention two characteristics of a mini steel plant.
ii) From where does the integrated steel plant at Jamshedpur get its iron ore and coal?
d) Name the following:
i) A shipbuilding yard on the west coast of India.
ii) A centre where diesel locomotives are manufactured.
iii) The foreign collaborator of the iron and steel plant at Rourkela. [2008]
37) a) Why is the silk industry considered as a small scale industry? Name the two types of silk
produced in India.
b) Name two important silk-weaving centres in Karnataka.
c) Name any two by-products of the sugar industry. Give two uses of each.
d) Explain two factors affecting the development of the cotton textile in India. [2009]
38) a) Which city is known as the electronic capital of India? Name any two major centres of
electronic products.
b) Most of the petrochemical units are in Maharashtra or Gujarat. Give two reasons to explain
why.
c) With reference to Visheswaraya Iron and Steel Ltd., answer the following:
i) Name the state in which it is located.
ii) From where does it get iron ore and power supply?
d) Give one important centre of production of each of the following.
i) Integral Coach Factory (ICF)
ii) Bharat Heavy Electricals Ltd. (BHEL)
iii) The Hindustan Machine Tools (HMT) [2009]
39) a) What is sericulture ? Name any two types of silk.
b) Classify industries on the basis of the nature of products. Give one example of each.

Volume 2 of 4 Universal Tutorials – X ICSE – Geography 73


74
c) i) Mention two reasons for the importance of the cotton textile industry.
ii) Mention one reason responsible for its poor performance.
d) Give geographical reasons for the following:
i) Kolkata has many cotton mills though cotton is not grown in West Bengal.
ii) Karnataka is an important centre for silk.
iii) India produces very little cane-sugar though it is one of the largest producers of sugar-
cane in the world. [2010]
40) a) Mention Iwo reasons for the development of the petrochemical industry in India.
b) What is the difference between a public sector industry and one which is in the private
sector? Give an example of an industry in each of the two sectors.
c) i) How is it advantageous for a mini steel plant (1) to use electric furnaces (2) not to be
located close to the location of the raw material?
ii) From where does the integrated steel plant at Bhilai get its requirement of iron and coal?
d) Name the following.
i) A city in India where MIG aircraft are manufactured.
ii) A centre where railway coaches are manufactured.
iii) The foreign collaborator of the iron and steel plant at Durgapur. [2010]
41) a) Name and define two important by-products of the sugar industry.
b) Give two reasons why the state of Punjab is the largest producer of woollen textiles.
c) Mention three main problems of the Jute textile industry in India.
d) With reference to the cotton textile industry:
i) Which is the country’s most important manufacturing centre?
ii) State two geographical reasons for its importance. [2011]
42) a) Which iron and steel industry of India is located away from the main coal areas ? What is the
main source of energy in the absence of coal?
b) Give two geographical reasons for the growth of IT industries in Bangalore.
c) With reference to the Bokaro Steel Plant, from where does it get its
i) coal ii) iron ore iii) water supply?
d) Name one important centre each for the production of the following:
i) Tractors ii) Electronic goods iii) Petro chemicals. [2011]
43) a) What are petrochemicals? Name any two products made from petrochemicals.
b) Why has the electronics industry grown in importance?
c) Mention three advantages that a mini steel plant has over an integrated steel plant.
d) Name:
i) A city on the east coast of India which has a ship building yard.
ii) The iron and steel plant set up with German collaboration.
iii) A city which has a plant manufacturing Maruti cars. [2012]
44) a) i) What is the significance of the Electronics Industry in recent times?
ii) Name two cities that have leading Software Companies.
b) Name the steel plants that were set up with Russian collaboration.
c) Explain three reasons as to why there is a large concentration of iron and steel plants in the
Chhota Nagpur Region.
d) What industrial product are the following centres noted for?
i) Gurgaon ii) Perambur iii) Chittaranjan [2013]
45) a) Name one integrated iron and steel plant in the private sector. Where does it obtain its iron
and coal from?

74 Universal Tutorials – X ICSE – Geography Volume 2 of 4


Chapter 12: Mineral Based Industries 75
b) Name two raw materials used in the petrochemical industry and state two advantages of
petrochemical products.
c) Give a reason for each of the following:
i) Vishakhapatnam is a leading centre for ship-building.
ii) Mini steel plants cause less pollution than integrated steel plants.
iii) The electronic industry has made an impact on both entertainment and education.
d) Name a manufacturing centre for each of the following industries:
i) Engines for MIG aircraft
ii) Diesel locomotives
iii) Software [2014]

Volume 2 of 4 Universal Tutorials – X ICSE – Geography 75


76

Chapter 13: Transport


→ Indian Transport
→ Road transport
→ Rail transport
→ Water transport
→ Air transport

Indian Transport:
™ India is a land of vast distances from north–south as well as from east–west. So development of
cheap and efficient means of transport is essential for the progress of the nation. Transportation is
mainly divided into three major.
 Road Transport
 Railway Transport
 Water Transport
 Air Transport

Road Transport:
Importance:
z Roads are important as they are necessary in the transportation of people and goods over
short and medium distances. One such road was constructed by Sher shah, which
connected. Amritsar to Delhi is called Sher Shah Suri Marg now.
Advantage:
z They are cheap and easy to construct and maintain.
z Roads can be constructed in hilly areas as they can negotiate steep slopes and sharp
bends.
z Road can act as feeder to railways
z Roads are more flexible than railways Roads are more accessible to different places than
railways.
Disadvantage:
z Roads are not suitable for long distance travelh
z Heavy goods cannot be transported easily by roads.
z Rapid increase of vehicles on the road created more air pollution.
z It is more expensive than rail transportation
z On the basis of importance and maintenance Indian roads can be divided into.
„ The Golden Quadrilateral: The Golden Quadrilateral Connects Delhi – Mumbai –
Chennai – Kolkata by 6 lane super high way
„ National high way: The main roads were constructed and maintenance by Central
Public works Department is called National High way. They connect state capitals, big
cities and important posts.

76 Universal Tutorials – X ICSE – Geography Volume 2 of 4


Chapter 13: Transport 77
„ North South Corridor: East–West Corridor and North–South Corridor Compromise
National High way Connecting Srinagar to Kanyakumari and East–West Corridor
Connecting slicher (Assam) to Probonder (Gujrat).
„ Fast west Corridor: Express Ways have been constructed to construct to ensure soft
and smooth travel between important towns. Two important express ways are Mumbai –
Pune express way and the Kolkata – Durgaur express way.

Rail Transport:
¾ Railway system is the life line of the countries inland transport. It serves the needs of both
passenger and freight. The first rail way was started in India in 1853 between Mumbai and Thane.
 Indian railways comprise 3 gauges:
z Broad Gauge: It has a distance of 1.676 meters between two rails. More than 70% of the
Indian railways is broad gauge.
z Metre Gauge: The distance between the rails is 1 metres 25% – of the railways is metre
gauge.
z Narrow Gauge: This rail is two of types:
„ One is 0.762 metres.
„ Another is 0.610 metres. This type is restricted on hilly areas only.

Limitations:
z There is a big operational problem to the Indian Railway to shift from one gauge to another
gauge. It is time consuming and as well as expensive also
z The tracks are not able to carry increased loads and accidents are becoming quite frequent
z Repair and maintenance of bridges constructed is very important.

Water Transport:
¾ The inland water ways and coastal waterways are the most important water ways are the
cheapest means of transport for large and bulky loads. Water ways are unable to compete with
road and railways because of its slow speed. One important inland waterway is Ganga.

Factors affect water transport:


 Regular flow of Sufficient Water.
 Silting of the river bed reduced the depth and creates problems for navigation.
 Water in rivers is sometimes reduced due to the diversion of water for irrigation.
 The presence of waterfalls, sharps bends along the rivers course minders the developments of
waterways.
 There should be sufficient demand to make it profitable.
Port:
z A port is a place on the coast with docks where cargo can be unloaded and distributed
through land. There are 12 major ports in India.
z Ports of West Coast:
„ Mumbai Port (Nhava Sheva / JNPT)
„ Kandla (Gujrat)

„ Kochi (Kerala)

„ Marmagao (Goa)

Volume 2 of 4 Universal Tutorials – X ICSE – Geography 77


78
„ New Mangalore (Karnataka)
Port of East Coast:
„ Tuticorin (Tamil Nadu)

„ Chennai (Oldest artificial howrbour)

„ Vishakhapatnam (land locked Port in)

„ Paradeep (Orissa Andhra Pradesh)

„ Kolkata and Haldia Port (W.B.)

Air Transport:
¾ Air transport is the fastest mode transport. Aircrafts of one type or another have made it possible
to reach the most remote parts of the earth when speed and time are important constraints. This
mode of transport is indispensable.
Limitations:
z Cost of air transport provision of air terminal facilities as well as cost of take up and landing
rights are all very expensive. Large scale air transport is still quite luxurious.
z It has limited carrying capacity, and the amount of freight that can be carried is restricted by
the lack of space.
z Weather conditions can also hamper air transport.

Major International Airports:


 Indira Gandhi International Airport (New Delhi)
 Netaji Subhas Chandra Bose International Airport (Kolkata)
 Sahar International Airport (Mumbai)
 Meenambakkam International Airport (Chennai)2

REVIEW QUESTIONS AND ANSWERS:


1) Name the organization who is responsible for the development, maintenance and
management of highways in India.
Ans: National Highways Authority of India (NHAI).
2) Name two projects undertaken by NHAI.
Ans: i) Golden Quadrilateral ii) North–south and East–west corridors
3) Name any two major Express Highways of India.
Ans: i) Ahmedabad – Vadodara expressway ii) Delhi – Gurgaon expressway
4) Name the areas where density of roads is low.
Ans: Rural areas.
5) Give any two advantages and two disadvantages of roadways.
Ans: Advantages:
i) Road transport is a fast, flexible and demand responsive mode which is capable of providing
door to door service.
ii) It links remote and inaccessible areas such as hilly, tribal, desert and backward areas.
Disadvantages:
i) Riding quality of roads is poor, bridges are often weak and distressed.
ii) A large section of the highways has inadequate road pavement thickness.

78 Universal Tutorials – X ICSE – Geography Volume 2 of 4


Chapter 13: Transport 79

6) a) Name the
i) India’s first expressway that opened in 2001.
ii) Expressway which is the part of GQ Highway Project.
b) Give the special features of Delhi–Gurgaon expressway.
Ans: a) i) Ahmedabad–Vadodara expressway ii) Delhi–Gurgaon expressway
b) Its special features are SOS telephones at every 1.5 km, CCTV surviliance, and a 32 – lane
toll plaza at the Delhi–Haryana border.
7) What is width of:
a) broad gauge?b) metre gauge? c) narrow gauge?
Ans: a) 1.676 mm b) 1.000 mm c) 762 mm and 610 mm
8) What is the another name of board gauge railway line on the west coastal lowland?
Ans: Konkan Railway Line.
9) What is meant by an expressway?
Ans: Expressways are cemented six–lane roads, designed to provide smooth high–speed movement
without any obstacles like traffic or speed breakers.
10) What are the chief means of transportation in India?
Ans: India has every mode of transport system: b land, air and water
Land: i) Roads ii) Railways
Air: i) National ii) International
Water i) Inland (river, canals, backwaters, creeks etc.)
ii) Seas and ocean routes.
11) What is the name of the National Airlines of India?
Ans: Air India.
12) Why are railways important?
Ans: Railways are important because they are the main arteries of inland transport. They are the
lifelines of the country for large scale movement of traffic, freight and passengers.
13) What are National highways?
Ans: National Highways are roads which connect one state with the other and are of national
importance. They are constructed and maintained by the central government.
14) Name the places that are connected by:
a) Golden Quadrilateral.
b) North-South and East-West corridors.
Ans: a) Golden Quadrilateral connects Delhi, Mumbai, Chennai and Kolkata.
b) North-South corridor connects Srinagar to Kanyakumari and East-West corridor connects
Silchar to Porbundar.
15) Name four international ports of India.
Ans: a) Mumbai b) Kochi c) Kandla d) Vishakhapatnam.
16) The great plains have more railways than the Himalayan Mountains. Why?
Ans: The great plains have more railways due to:
a) High density of population b) Rich in agriculture
c) Greater industrial activity d) Level land or plain land.
17) Enumerate the problems faced by our railways.
Ans: A Various problems faced by our railways are as follows:
a) Passengers travel without tickets due to lack of proper vigilance.
b) They pull chains unnecessarily resulting in the delay of trains.
c) Thefts and damages of railway property is common.
d) Railway accidents due to the negligence of authorities is a major problem of Indian railways.
e) The signalling and the safety systems are all outdated.

Volume 2 of 4 Universal Tutorials – X ICSE – Geography 79


80

18) How does road transport score over railway transport?


Ans: a) Road transport requires less investment than railway.
b) Maintenance cost is also low.
c) Road transport provides access to difficult terrains.
d) Roads facilitate the fast transportation of perishable goods and thereby stimulate their
production.
19) What are the three types of railway gauges in India? What is the disadvantage of three
gauge- railway system?
Ans: Three gauges are:
a) Broad gauge: width between the rails – 1.67 m.
b) Metre gauge: width between the rails – 1.0 m.
c) Narrow gauge: width between the rails – 0.76 m.
Different gauges create difficulties in the smooth flow of traffic. If gauges all over become
uniform, it will ensure higher speed, cheaper transport and will reduce the inconvenience of
changing trains during a journey.
20) Give a merit and a demerit for each of the following transport systems in India.
a) Road transport b) Rail transport
c) Inland water transport d) Sea transport
e) Air transport
Ans: a) Road Transport
Merits: (i) The roads link the rural areas to the urban areas. (ii) They can be constructed in
hilly, desert and forest areas.
Demerits: (i) They are unsuitable for long distances. (ii) Carriage of heavy and bulky
commodities over a long distance is very costly.
b) Rail Transport
Merits: (i) It is suitable for long distance. (ii) Heavy and bulky commodities can be
transported by railways.
Demerits: (i) Unsuitable for short distance, as it is expensive. (ii) Unsuitable for perishable
items.
c) Inland water Transport
Merits: maintenance cost is less.
Demerits: (i) Waterways are very slow. (ii) As the water is being utilized for irrigation, there is
less water in the rivers.
d) Air Transport
Merits: (i) It is the fastest mode of transport. (ii) It can go over unsuitable topography and hilly
terrains.
Demerits: (i) It is an expensive mode of transport. (ii) It does not connect the rural areas.
21) State any three merits of roadways.
Ans: a) Construction cost of roads is much lower than that of railway lines.
b) Roads can traverse comparatively more dissected and undulating topography.
c) Roads can negotiate higher gradients of slopes and as such can traverse mountains such as
the Himalayas.
22) Where and why is the rail transport the most convenient means of transportation in the
northern plains?
Ans: Rail transport is most convenient means of transportation in the northern plains due to level land,
high population density rich agricultural resources and greater industrial activity.

80 Universal Tutorials – X ICSE – Geography Volume 2 of 4


Chapter 13: Transport 81

23) Differentiate between district and rural roads?


Ans:
District Roads Rural Roads
These roads connect towns and large villages These roads link villages with district roads.
with one another and with district
headquarters.
These are mostly unsurfaced and lack bridges These are mostly unsurfaced, narrow and
and culverts. zig–zag tracks unsuitable for heavy
mechanized traffic
24) a) Name the scheme which was launched to provide connectivity to rural areas as part of
a poverty eradication measure.
b) What are the special features of Mumbai–Pune expressway?
Ans: a) Pradhan Mantri Gram Sadak Yojna
b) The special features of Mumbai–Pune expressway are :
i) Six–lane concrete highway with 7m – wide divider. An extra lane provided on each side as a
hard shoulder.
ii) separate tunnels for traffic.
iii) No two–wheelers, three–wheelers or tractor vehicles allowed.
iv) Provision of motels, workshops, toilets, emergency phones, first aid etc.
v) Complete fencing to avoid humans and animals crossing the expressway.
25) a) What is the Golden Quadrilateral (GQ) Project?
b) Give any two economic benefits of the GQ
Ans: a) The Golden Quadrilateral (GQ) is the largest express highway project in India. It is the first
phase of the National Highways Development Project. It consists of building 5,846 km of four
/ six lane express connecting Delhi, Mumbai, Kolkata and Chennai.
b) The economic benefits of the Golden Quadrilateral project are:
i) It will interconnect many major cities and ports.
ii) It will give a stimulus to truck transport throughout India.
iii) It will help in the industrial growth of all small towns through which it passes.
iv) It will provide vast opportunities for transport of agricultural produce from the hinterland to
major cities and ports for export.
v) It will provide job opportunities in its construction.
26) What are the National Highways? Also write the names of its two major projects?
Ans: National Highways are highways which run through the length and breadth of the country and
connect state capitals, port towns, industrial and mining area and cities and towns of national
importance. These highways are maintained by the Central Government
i) Golden Quadrilateral.
ii) North–south and East–west Corridors.
27) a) Why the traffic on National Highways has been growing?
b) Explain what steps are being taken by the Government to provide hindrance free traffic
movement
Ans: a) The traffic on National highways has been growing due to industrialization in the country.
b) The Government is taking steps to utilize latest technologies and improved management
techniques to provide hindrance–free traffic movement. This is being done by widening
roads, grade separation, construction of bypasses, bridges, rail–road crossing etc.
28) Differentiate between :
a) National Highways and State Highways
b) Broad gauge and metre gauge.
Ans: a) i) The National Highways are constructed and maintained by the Central Governments.
While the State Highways are constructed and maintained by the State Government.

Volume 2 of 4 Universal Tutorials – X ICSE – Geography 81


82
ii) The National Highways are main highways running through the length and breadth of the
country connecting major ports, state capitals, large industrial and tourist centres. While
the State Highways provide linkages with the National Highways, district headquarters,
important towns, tourists centers and minor ports with in the state.
iii) The National Highways facilitates inter–state transport and movement of defence
personnel and materials in strategic areas. While the State Highways provide connectivity
to important towns and cities as wells as with National Highways or State Highways of the
neighbouring states.
iv) The National Highways has about 70.548 Km. length while the length of state Highways is
about 128000 km.
b)
Broad Gauge Meter Gauge
The distance between rails is 1.676 mm The distance between rails is 1,000 mm.
The total route length is 51.082 km. Its total route length is 9,442 km.
29) a) What is the width of (i) single lane and (ii) multi–lane National highways?
b) When was the NHAI constituted? Also explain its responsibilities
Ans: a) (i) 3.75 m (ii) 3.5 m.
b) The NHAI was constituted in 1988.
It is responsible for the development maintenance and management of National Highways. It
is currently undertaking the developmental activities under NHDP in phases. In addition to
this NHAI is also responsible for implementing other projects on National Highways, primarily
road connectivity to major ports in India.
30) State any two advantages of airways?
Ans: i) Air transport is the fastest and comfortable mode of transport.
ii) It is vital for our internal transport as well as for the links with other countries.
iii) It is essential for defence of the country and plays a crucial role in times of emergency.
iv) Remote areas which are not served by railways or roads have to depend on air transport
alone.
31) State any two disadvantages of airways?
Ans: i) It is very costliest mode of transport.
ii) It depends on weather conditions.
iii) The airport infrastructure demands heavy investment, large expenditure on servicing,
replacement and renewals.
iv) Air transport is run on petroleum (non–renewable source of energy).
32) Express these in the full form?
a) IWAI b) NHDP c) NHAI d) EMUs
Ans: a) Inland Waterways Authority of India
b) National Highways Development Project
c) National Highways Authority of India
d) Electric Multiple Units.
33) Give any two advantages and two disadvantages of water transport?
Ans: Advantages of Waterways:
i) It is the cheapest mode of transport.
ii) It is suitable for heavy and bulky goods.
Disadvantages of Waterways:
i) Water transport depends on the conditions of weather.
ii) Water transport is limited to area where rivers are navigable and oceanic routes exist.

82 Universal Tutorials – X ICSE – Geography Volume 2 of 4


Chapter 13: Transport 83

34) a) Name the :


i) biggest port of India
ii) first corporatized port of India.
b) When was Airport Authority of India established?
c) Why has the railway system been divided into 16 zones?
Ans: a) i) Mumbai ii) Ennore
b) April 1, 1995
c) The growth of Indian Railways in the past 150 years has been phenomenal. Its huge size has
put pressure on the centralized management system. Therefore, to ease this pressure, the
railway system has been divided into 16 zones

MISCELLEOUS QUESTIONS / HOME WORK


1) Name the organization who is responsible for the development, maintenance and management
of highways in India.
2) Name the Mughal ruler who built the famous Grand Trunk (GT) road.
3) What are advantages of transport system? (Any three)
4) Name two projects undertaken by NHAI.
5) Name any two major Express Highways of India.
6) Name the areas where density of roads is low.
7) Give any two advantages and two disadvantages of roadways.
8) a) Name the:
i) India’s first expressway that opened in 2001.
ii) Expressway which is the part of GQ Highway Project.
b) Give the special features of Delhi-Gurgaon expressway.
9) Name the two cities which have Metro rail network.
10) Give the number of Zones the Indian Railways have.
11) What is the width of:
a) broad gauge? b) metre gauge? c) narrow gauge?
12) What is the another name of board gauge railway line on the west coastal lowland?
13) Differentiate between district and rural roads.
14) When and why was the Border Road Development Board setup?
15) a) Name the scheme which was launched to provide connectivity to rural areas as part of
poverty eradication measure.
b) What are the special features of Mumbai-Pune expressway?
16) a) What are International Highways? Give the purpose of such type of highways.
b) How many categories are there of the International Highways? Explain.
17) What is meant by an expressway?
18) What are the chief means of transportation in India?
19) Explain the role of roads in the economic f development of the country.
20) a) What is the Golden Quadrilateral (GQ) Project?
b) Give any two economic benefits of the GQ.
21) What are the National Highways? Also write the names of its two major projects.
22) When and where did the railways begin in India?
23) Differentiate between:
a) National Highways and State Highways
b) Broad gauge and metre gauge.
24) a) What is the width of (i) single lane and (ii) multi-lane National highways?
b) When was the NHAI constituted? Also explain its responsibilities.
25) Write two advantages and two disadvantage of railways.
26) Explain the three categories of track system followed in India.
27) What is the name of the National Airlines of India?
Volume 2 of 4 Universal Tutorials – X ICSE – Geography 83
84
28) What is Pawan Hans?
29) State any two advantages of airways.
30) State any two disadvantages of airways.
31) Express these in the full form
a) IWAI b) NHDP c) NHAI d) EMUs
32) When and where did the air transport begin in India?
33) Explain the role of Air India in the air transport.
34) On which river system does the National waterway No 1 lie?
35) Write the number of major and non-major ports of India.
36) Give any two advantages and two disadvantages of water transport.
37) In which state the Nehru Trophy Boat Race (Vallankali) is held?
38) Name the:
i) biggest port of India ii) first corporatised port of India.
39) On the outline map of India, show the Major sea ports.
40) Which is the second largest port in terms of volume of traffic handled and is the oldest artificial
harbour on the east coast of India?
41) Why is road transport difficult in the northern mountainous areas?
42) What are the advantages of railways compared to roadways?
43) Give two reasons to explain why water transport is considered to be cheap.
44) What are the advantages of air transport?
45) Name the international airports of India’s metro cities.
PREVIOUS YEARS BOARD QUESTIONS:
1) a) Why is road transport favoured in the northern plains of India?
b) Give reasons to explain the lack of rail transport in Northern India.
c) Name the following.
i) An important inland waterway of north east India.
ii) One expressway with its terminal cities.
d) Mention two advantages and one disadvantage of air transport. [2011]
2) a) Why has the importance of inland waterways declined? Give any two reasons.
b) Name two areas where helicopter services may be used?
c) Mention any three problems being faced by the Indian Railways.
d) What is the Golden Quadrilateral? Mention any two ways in which it will help in the economic
development of the country. [2012]
3) a) Give one disadvantage of air transport. Why is it still a popular means of transportation in
India?
b) i) Give two reasons why peninsular rivers are not ideal for navigation.
ii) Name a port on the east coast which is often hit by cyclones during the months of October
and November. [2013]
4) a) Why is road transport in India considered more useful than rail transport?
State two reasons in support of your answer.
b) Mention one advantage and one disadvantage each of inland waterways. [2014]

84 Universal Tutorials – X ICSE – Geography Volume 2 of 4


Chapter 14: Sources of Waste 85

Chapter 14: Sources of Waste


What is waste?
¾ Waste is a general term used to describe any material that is discarded because it has served its
purpose and is no longer useful. Waste can also be any material that remains at the end of a
process and has no further use and so it is thrown away.
¾ According to the Environment Protection Act, 1990, waste is defined as:
 any substance which constitutes a scrap material, or an effluent, or another unwanted surplus
substance arising from the application of any process; and
 any substance or article which requires to be disposed of as being broken, worn out,
contaminated or otherwise spoiled.

Characteristics:
¾ ‘Waste’ is a matter.
¾ It occupies some space;
¾ It produces odour and smell;
¾ Waste has weight.

Types of waste:
Solid wastes :
z It includes garbage, i.e. food left-overs, decaying fruits and vegetables, crop residues, etc,
and vegetables, crop residues, etc., and rubbish, including cans, bottles, corroded metal
pieces, plastics, wrappers, ashes, body parts of dead animals, etc.
Liquid Wastes:
z it includes sewage from toilets of houses, hospitals, restaurants, offices and factories, etc,,
and oil spills.
Gaseous Wastes:
z It includes fuel exhausts containing carbon dioxide, nitrogen oxides, carbon monoxide,
sulphur dioxide etc. and smog which is formed as a hazy mixture of gases when fuel
exhausts react with sunlight.

Waste can also be classified into the following categories:


Toxic
z Toxic wastes are the wastes which pose a serious threat to human health and environment.
z These wastes result from industrial processes, from the use of chemical fertilisers in
agriculture, from the biomedical waste generated in the hospitals, radioactive wastes and
the electronic waste or e-waste generated by broken or unwanted electrical appliances
such as computers, mobile phones and electronics.

Volume 2 of 4 Universal Tutorials – X ICSE – Geography 85


86

Non-Toxic
z Non-Toxic wastes are the waste includes garbage, i.e., food left-overs, decaying fruits and
vegetables, crop residues, etc.. and rubbish, including cans, bottles, corroded metal pieces,
plastics, wrappers, ashes, body parts of dead animals, etc
z Some of the toxic wastes are arsenic, cyanide, lead, cadmium, nickel, beryllium, uranium
and mercury and their compounds, chlorinated solvents, asbestos, organo-chlorine
pesticides, photographic wastes, plating sludges, pesticides residues, waste paints and
lubricants. Burning produces oxides of sulphur and nitrogen which become toxic at high
concentration.
z The waste produced by households, shops, offices, restaurants and schools that do not
pose a serious problem to animals, plants or to the environment is called Non-toxic Waste.

Sources of Waste
Domestic Waste:
 Wastes generated due to domestic activities are called domestic wastes. Food leftovers, fruits
and vegetable peels, bits of paper, packets, polythene bags, bottles, empty metal and
aluminium cans scrap metals, glass pieces, cotton, rags, discarded clothes, ashes from
burning coal, sewage from toilets, batteries, expired medicines, chemicals etc. are some of the
examples of domestic wastes.

Industrial Waste:
 Wastes from various types of small and large scale industries are called industrial wastes. For
example, in a textile industry, wastes may be in the form of fibres used to wipe off Oil. Industrial
wastes include chemicals (lead, mercury, arsenic paints, sand paper, paper products, industrial
by-products. metals, etc.)
 Industries use a great deal of fuels for energy that produce waste gases and other materials.
Sulphur dioxide and nitrogen oxides emitted from power stations cause health hazards
because of their link with acid rain.
Mining:
z Wastes generated during mining operations or excavations are called mining wastes.
z These can be toxic or hazardous
Cement industry :
z Cement manufacturing industries produce various coarse and fine particles. The dust
emitted by cement industry a potential health hazard.
Oil Refineries:
z Petroleum industry can cause environmental problems. These industries include
petrochemical plants and oil refineries. Their waste include inorganic sulphur compounds,
hydrocarbons and organic acids.
Food Processing Units:
z Remnants, waste products of dairies, breweries and meat processing units produce organic
wastes.

86 Universal Tutorials – X ICSE – Geography Volume 2 of 4


Chapter 14: Sources of Waste 87

Paper Industry:
z Some of the wood chips, cellulose fibres and chemicals are rejected as waste material in
paper industry. The effluents contain chlorine, sulphur dioxide etc., which are considered
highly poisonous for the fish.
Chemical Industries:
z These include manufacturing industries, alkali manufacturing, fertiliser, pesticides and
several other industries. The effluents from these industries contain acids which have
corrosive effects.
Metal Industries:
z These industries discharge effluents containing copper, lead, chromium, cadmium, zinc etc.
which are toxic to man as well as to aquatic life.

Agricultural waste
 Agricultural wastes include the following :
 There are two types of crop residues
Field residues:
z These are the materials left in an agricultural field or orchard after the crops have been
harvested. These include straw from Barley, beans, oats, rice, rye wheat and stalks from
corn, cotton, sorghum, soyabean and alfa-alfa
Process Residues:
z These are the materials left after the processing of the crops into useful products.
Animal Waste:
z These include excreta of farm animals, animal slurry and animal bedding such as poultry
litter
Processing Waste:
z It includes the waste produced by agro–based industries like sugar, dairy, edible oil, food
processing, coir setting, jute retting and pulp and paper. Processing waste comprises waste
water and solid wastes generated during the manufacturing processes.
z For example, in the production of sugar, the extracted sugarcane juice is subjected to a
series of purification processes for making it free from dirt, colour and organic matter. In this
purification process the solid waste, so separated is known as press mud. It requires safe
disposal.
Fertillisers:
z Agricultural fertilisers have components of nitrogen, phosphorus or potassium. These
chemicals can affect the surface water during drainage and storm runoff.
z In some fields, the water reaches the groundwater level and these chemicals contaminate
the groundwater.
Pesticides and Insecticides:
z Pesticides and insecticides are toxic substances. They are used to eliminate some of the
insects or pests.
z Some of the insecticides like DDT and aldrin, affect humans and animals directly or through
the food which they consume.

Volume 2 of 4 Universal Tutorials – X ICSE – Geography 87


88
z Agricultural wastes can cause environmental problems, if they are not disposed of properly.
They become breeding ground for harmful insects and rodents. Burning of wheat or rice
straw gives rise to smoke which pollutes the environment.

Municipal waste
 Municipal waste is the waste generated in a municipality or a local government area. Such
waste is produced by shops, offices, restaurants, schools, courts, libraries, banks, hospitals,
parks, domestic etc. and is collected from public waste bins.
Sewage:
z Municipal sewage is the liquid waste which is extremely foul in nature. Domestic sewage is
the waste water from kitchens, bathrooms, lavatories, laundries and laboratories. In addition
to the mineral and organic matter in the waste water, domestic sewage also comprises
human excrement such as urine, faeces, soapy wastes, food wastes, paper dirt, dirty water
and numerous other substances.
DegradabIe and Non-degradable Wastes
z Municipal wastes generated in various offices include used paper cuttings, xerox paper bits,
carbon papers, typewriter ribbons, broken pens, pencil shavings, groundnut husks, paper
packets, tissue papers, wrappers, glass pieces, rubber, cotton pads, bottles, vegetable
matter, cooked items, etc.
z Municipal, domestic and agricultural solid wastes that can be degraded by micro-organisms
are called degradable or biodegradable wastes. Examples of this type of waste are
vegetable wastes, stale food, tea leaves, egg shells, dry leaves, etc.
z Biodegradable wastes can be further classified into:
„ Simple Biodegradable Waste: These wastes are easily broken down by natural
processes of decomposition, Leaves, vegetable peels, plant remains, faecal remains,
waste water, dead plants arid animals all belong to this category.
„ Complex Biodegradable Waste: The waste material that comes under this category is
not easily decomposed. They are resistant to natural processes of decomposition.
However, over a long period of time, they can be decomposed. For example, glass
bottles take a million years to decompose, leather shoes take 30-40 years, tin cans take
50-100 years. We must be careful while disposing of waste material under this category.
z Biodegradable wastes can easily serve as alternate sources of energy. Gobargas (biogas)
is an example of conversion of biodegradable wastes into energy.
z Non-degradable or non biodegradable wastes are the wastes which cannot be degraded or
broken down through microbial activities.
z Example crude petroleum, plastics, styrofoam products, cans, glasses, polymer, synthetic
pesticides, radioactive fall-out, some industrial effluents and metals like lead

Bio–medical wastes
 Bio-medical wastes are the wastes that are generated during the diagnosis, treatment and
immunisation of human beings or animals. These wastes include:
z needles, syringes, soiled dressings;
z pathological wastes such as blood, tissues, body parts, body fluids, human foetuses and
chemicals used for pathological tests;
z infectious wastes like of cultures and stock of infectious agents;
z wastes from surgery and autopsy;

88 Universal Tutorials – X ICSE – Geography Volume 2 of 4


Chapter 14: Sources of Waste 89
z used and discarded medical instruments such as scalpels, blades and other glass material,
empty plastic bottles, polythene bags, tubes, gloves, etc.;
z expired medicines including pills, capsules, chemicals;
z containers having innocuous or inert gas and aerosol from hazardous wastes.

Nuclear waste
 Nuclear waste is the radioactive waste generated from nuclear energy industry
 Radioactive elements such as uranium and radium have highly unstable atomic nuclei, whose
disintegration results in radiation emission which may be highly injurious. During nuclear tests,
radioactive dust may encircle the globe at altitudes of 3000 metres or more.
 This dust often comes down o the earth mixed with rain. Some of it percolates down through
the soil into groundwater reservoirs or is carried into streams arid rivers.
 The leakage of nuclear radiations from nuclear reactors and nuclear research laboratories is a
significant source of nuclear waste.
 Medical X-rays constitute about 18 per cent of artificial radiations used in radiotherapy for
diagnostic purposes. These rays are highly penetrating

Volume 2 of 4 Universal Tutorials – X ICSE – Geography 89


90

Chapter 15: Need for Management of Waste


Introduction:
 We move waste from one place to another but never get rid of it completely. Nature has
recycled waste materials for millions of years.
 Human population has increased so rapidly during the last century that the environment is now
threatened by our activities and the wastes they produce. Much damage is done to the
environment by the pollution of air, degradation of soils and contamination of water sources
 There is a close connection between waste, pollution and the damage to the environment.
Waste causes pollution, which in turn causes the damage. Hence, there is an urgent need for
management of waste

Transmission of Diseases
Waste on land :
 Various diseases spread on an epidemic scale due to waste accumulation on land and water
bodies. Vectors like flies, mosquitoes, rodents and pet animals transmit these diseases. The
waste is a breeding ground for such vectors.
 Hence, there is an urgent need to manage waste disposal in an effective manner. Here is a list
of common diseases spread by mosquitoes, flies, rodents and pet animals.
 Housefly: Typhoid, diarrhoea, dysentery, cholera, gastro-enteritis, etc
 Sandfly: Kala-azar, (Black fever) sandfly fever, etc.
 Tsetse fly: Sleeping sickness.
 Mosquitoes: Malaria, filaria, yellow fever, dengue, chikungunya, encephalitis etc.
 Rodents: Plague, salmonellosis etc.
 Pet animals
z Dog – Rabies, hydrated diseases, etc.
z Cat – Dermatophytosis, anthrax, etc.
 Industrialisation and urbanisation pollute water in the following manner.
 Sewage contains organic matter that cannot be decomposed
 Industrial and commercial waste has toxic agents including metal salts and complex synthetic
organic chemicals.
 Fertilisers and pesticides produce pollutants. Human beings are affected by pollution.
 There are also other pollutants like radioactive substances.
Human beings are affected by pollution:
z by drinking contaminated water,
z by using contaminated water for purposes of personal hygiene and recreation.

Common water borne diseases


 Viral: Viral hepatitis, diarrhoea etc.
 Bacterial: Cholera, typhoid, dysentery.
 Protozoan: Amoebiosis, diarrhea

90 Universal Tutorials – X ICSE – Geography Volume 2 of 4


Chapter 15: Need for Management of Waste 91
 Helminthic: Roundworm, thredworm.

Greenhouse effect and global warming:


 The rate of absorption of solar radiation by earth and its emission back into space as infrared
waves balances the heat on the earth. This phenomenon plays a very important role in
maintaining surface temperature of the earth. The carbon dioxide and other gases form a
blanket around the globe which prevents the passage of infrared waves from the earth back
into space.
 Concentration of solar radiation produces much heat, making the earth a very warm place. This
phenomenon is similar to that of a greenhouse in which the glass enclosed area gets heated
up due to the insulation from the rest of the environment. The warming up of the atmosphere is
due to the greenhouse effect. Hence, Global Warming is also known as Greenhouse Effect.

Greenhouse gases
 There are five gases which are mainly responsible for the Greenhouse Effect and Global
Warming. These gases are known as Greenhouse Gases. They are:
z Carbon dioxide (CO2)
z Methane (CH4)
z Nitrogen oxide (Nitrous oxide)
z Chlorofluorocarbons (CFC).
z Water vapour.
 Then, the sea level will rise by a few metres and most of the cities on the seashore may be
submerged and coastal eco-life will be adversely affected.

Effects of Global Warming are:


 Global temperature is likely to rise by 2°C to 5°C during the next century.
 Due to rise in temperature by 2°C to 5°C, there is a chance of melting of ice caps on the
Earth’s poles.
 As the increase in temperature will be uniform all over the surface of the world, there will be
serious climatic changes. This will bring various changes in wind arid rain pattern.
 Higher temperature will cause rise in transpiration, which in turn will effect the groundwater
table.

Depletion of Ozone Layer


¾ The atmosphere is mainly divided into five layers
 (i) Troposphere (ii) Stratosphere (iii) Mesosphere (iv) Ionosphere (v) Exosphere.
 In the second layer, La, the Stratosphere which lies at the height of 20 km to 50 km from the
Earth’s surface, lies the Ozone layer. In spite of its low density, the Ozone layer plays an
important role in our life.
 Due to the presence of Ozone layer, ultraviolet rays and Infrared rays from the sun cannot
reach the Earth surface directly. Ozone layer absorbs the harmful ultraviolet rays from the sun
and protects the life on the Earth from their harmful effects.
 Causes of ozone layer depletion: when the Oxides of Nitrogen (NO and NO2) come in contact
with Ozone (O3), their chemical reaction destroys Ozone layer. Besides this, supersonic
aeroplanes move through the stratosphere and emit huge amount of Nitrogen gas which
depletes the Ozone layer. Another important causative factor of Ozone layer depletion is
Chlorofluorocarbons (CFCs), which have strong power to damage the Ozone layer.

Volume 2 of 4 Universal Tutorials – X ICSE – Geography 91


92
 All the developed and developing countries are using CFCs–type chemicals as refrigerants in
aerosol, paints, plastics, foam and thermal insulating materials in spray and packaging
industries. During the use of such materials, a lot of CFCs ultimately get dispersed into the
atmosphere
 The ultraviolet rays cause genetic disorders which ultimately affect heredity. Increased
concentration of ultraviolet rays disturb ecological balance in marine ecosystem. Green algae,
fish and other animals on continental shelves get affected by ultraviolet rays.
 Plastic become brittle when they come in contact with ultraviolet rays.

Acid Rain
¾ Acid Rain means the presence of excessive acids in rainwater. Burning of coal, wood or
petroleum produce sulphur and nitrogen. These two react with oxygen and are converted into their
respective oxides–sulphur dioxide and nitrogen dioxide, which are soluble in water.
¾ During rain, these oxides react with large quantities of water vapour in the atmosphere to form
acids like sulphuric acid, sulphurous acid, nitric acid and nitrous acid. These acids, when they
precipitate together with rain or snow form acid rain.

Effects of Acid Rain


 Acid rain increases acidity in the soil and destroys forests and crops.
 It corrodes buildings, monuments, statues, bridges, fences and railings. For example acid rain
produced by the pollutants from the Mathura oil refinery has been turning the white marble
surface of the Taj Mahal into yellow.
 It poses a serious threat to human health, since it contaminates air and water.
 It affects the human nervous system by causing neurological diseases.
 Aquatic species are affected due to acid rain.
 Acid rain affects the plant growth, Plant leaves get burnt and dry.

Soil Health
¾ Soil is the foundation for a healthy biosphere. Precipitation from air as acid rain and dry deposition
of pollutants on land surface contribute to soil pollution.
¾ Chemicals and minerals in the soil react with chemical pollutants. These pollutants combine with
plant nutrients and the plants are consumed by animals.
¾ Polluted soils cause reduction in mineralisation and decomposition processes. Transformation of
sulphur, nitrogen, availability of phosphorus, biological nitrogen fixation in soil is affected by acid
rain. Destruction of the soil is synonymous with the destruction of the biosphere

Waste management
¾ One of the easiest ways of management of waste is the ‘3-R’ system, Le. Reduction, Reuse and
recycle. We can reduce our use of resources. We can reuse the materials for packaging, that is
use the same product several times, for example, bottles, containers, etc. We can recycle
materials such as glass, paper and metals (like aluminium, cans and steel) from old articles.
¾ We can recycle the used items to make new material, e.g. cardboard from used paper. There is
need to manage the waste properly. Therefore, public awareness of the health hazards of waste is
necessary.

92 Universal Tutorials – X ICSE – Geography Volume 2 of 4


Chapter 16: Impact of Waste Accumulation 93

Chapter 16: Impact of Waste Accumulation


Introduction:
¾ Accumulated solid wastes when left uncared, start decomposing. A number of pathogenic
(disease-causing) bacteria, virus and fungi grow in these wastes.
¾ Flies, insects, rodents etc. live in the accumulated waste heaps and carry germs of various
diseases to human habitations.
¾ Decomposition of wastes produces harmful gases that pollute the air around us.
¾ During rains, rainwater may take the decomposed waste along with pathogens (disease causing
germs) to our water bodies (rivers, ponds, wells etc.) and cause water pollution.
¾ All this leads to outbreak of epidemics and other health hazards.

Spoilage of Landscape:
¾ Much of the worlds solid waste is simply dumped onto vacant land and left to decompose. Open
dumps not only ruin the natural beauty of the land but also provide a home to rats and other
disease carrying organisms.
¾ Both open dumps and landfills may contain poisonous substances that seep into the groundwater
or flow into streams and lakes.
¾ Burning of coal, fuel wood or petroleum produces sulphur and nitrogen which react with oxygen
and are converted into their respective oxides–sulphur oxide and nitrogen dioxide.
¾ A chemical reaction occurs between the acid of the acid rain and the buildings. It exerts a
pressure on the monument surface leading to corrosion of its body.
¾ Many monuments are affected by acid rain. Examples are: the Parthenon of Athens, the
colosseum of Rome, the Taj Mahal of Agra.

Pollution
¾ Pollution is caused by man by the addition of waste toxic chemicals through the atmosphere into
the biosphere.
¾ The main sources of waste are domestic, commercial, industrial, municipal and agricultural
wastes. Agriculture and the food processing industry are considered to be the largest contributors
to the total annual production of solid wastes.
¾ The handling of solid wastes is a problem because most disposal methods cause harm to the
environment. Both open dumps and landfills may contain toxins that seep into the soil and the
water bodies and cause soil and water pollution respectively.
¾ The uncontrolled burning of accumulated waste creates smoke and other air pollutants that
release toxic substances into the environment and cause air pollution.
¾ Industrial waste contains harmful chemicals, particulates (small particles) and toxic heavy metals
such as lead and mercury. These toxic chemicals and heavy metals get deposited in animal
tissues and harm living things along the food chain.
¾ For example, grass gets some toxic chemicals from the soil. Animals eat such grass and get
affected by toxins.

Volume 2 of 4 Universal Tutorials – X ICSE – Geography 93


94
¾ These animals directly or indirectly pass on these toxins to the human beings through their dairy
products or meat.
¾ As accumulated waste decomposes, it produces a large quantity of methane gas. This is highly
explosive, if not managed properly.

Eutrophication:
 It is the process of depletion of oxygen from water bodies occurring either naturally or due to
human activities.
 The process of eutrophication takes place due to introduction of nutrients and chemicals
through discharge of domestic sewage, industrial effluents and fertilisers from agricultural
fields. Algae and phytoplankton use carbon dioxide, inorganic nitrogen and phosphate from the
water as food.
 They serve as food for microscopic animals (zooplankton). Small fish feed on these
zooplanktons and large fish in turn consume these small fish.
 When nutrients become abundant due to waste accumulation, the growth of phytoplankton and
algae increases. Consequently, the penetration of oxygen, light and heat into the water body is
reduced. This causes death of most of the aquatic organisms, draining water of all its oxygen.

Health hazards
Spread of Disease Through Contamination:
 Several incidents around the world have demonstrated the potential harm of accumulation of
waste on human health. Waste that is not properly managed is a serious health hazard.
 Waste dumped near a water source percolates through the soil into the water bodies and
contaminates the water.
 Direct dumping of untreated waste in rivers, seas and lakes results in the accumulation of toxic
substances in the water bodies and further in the food chain through plants and animals.
 The water logging results in breeding of mosquitoes in the stagnant water which spread
diseases like malaria and chikungunya.

Effects of Toxic Particulate Materials


 Lead: Affects blood system, causes behavioural disorders and can also cause death.
 Cadmium: Causes cardiovascular diseases and hypertension, kidney damage.
 Nickel: Causes respiratory problems, lung cancer.
 Mercury: Causes nerve and brain damage, kidney damage.
 Beryllium: -Causes berylliosis. Affects mucus membrane of eyes and lungs. Causes
shortness of breath, weight loss, lung cancer and affects heart.
 Asbestos: Causes asbestosis, shortness of breath and lung cancer.
z Arsenic is another chemical that has been shown to cause cancer.
z Radioactive waste, although present in small quantities, remains externally harmful to
human health for many years

Effect on Terrestrial Life


¾ Terrestriai life includes all the organisms that live on land – human beings, plants and animals.

94 Universal Tutorials – X ICSE – Geography Volume 2 of 4


Chapter 16:
1 Impact of Waste Accum
mulation 95

Effec
cts on Hum
man Bein
ngs:
 A
Accumulationn of solid waste looks ugly, smells fouul, an acts in
nsects, rats a
and other aniimals that
s
spread disea
ases. Burning g of waste in the open dump yards causes smo oke and foul smelling
a
air.
 Sanitary
S dfills are not fit for human
land n settlements
s because methane
m and carbon dioxiide gases
s
start coming up in the firsst two years.

Effec
cts on Pla
ants :
 W
Waste accum mulation hass dangerous effect on plant life. Plant life is affe ected either by direct
d
deposition off harmful toxiins from wasstes or indirectly through soil.
 The
T toxins ca ause:
z different tyypes of leaf injuries.
z premature e leaf fall.
z Decrease e in transpirattion.
z Reduction o photosynthesis.
n in the rate of
z reduction in biological nitrogen fixaation.
z dust deposited on le eaves blockk the stoma ata of plantts. These d decrease the e rate of
transpiration and inhibbit the absorp
ption of nutrie
ents from so oil.
z smoke em mitted by burrning of wasste causes re eduction in rooot and shoo
ot lengths, number
n of
leaves and number off grains per spike
s in case
e of crops like
e wheat.

Effec
cts on Animals and
d Birds :
 S
Scavangers and stray animals like dogs,
d rats, pigs
p and cow ws are direcctly affected by waste
w
when they fe
eed on the waste for fo ood. Sometimes these animals con nsume toxins s or non-
d
degradable s
substances like plastic ca
arry bags preesent in the waste
w and diee due to choking.
 The
T wastes consumed
c byy animals alsso lead to maany diseasess and other pproblems.

Effec
ct on Aq
quatic Liife:
 W
Waste accum mulation can cause signifficant damag
ge to aquatic life, both fre
esh water andd marine.
 Two
T categoriies of waste that cause greatest
g dam
mage to aquaatic life are – pesticides, which
w run
o agricultural lands and
off d industrial and
a domestic c wastes tha
at are improp perly dispose
ed of into
w
water bodiess.

Biom
magnificattion:
 T
The tem Biomagnifica
B ation meanss increasin ng the
c
concentrationn of variouss toxic subsstances alon ng the
f
food chain. Toxic substances at the level of primary
p
p
producers geet concentra ated at each trophic le evel as
t
they move up p the food ch hain.
 The
T phenom menon of concentrated toxict deposition at
t higher tro
the ophic level iss known as bio-accumulation.
 A small amo ount of toxicc constituennt which is neither
e
excreted norr metabolised, gets incre eased as th he food
c
chain movess upward from m one trophic level to th
he next
a the toxicc constituentss become co
and oncentrated.

Volume 2 of 4 Universal Tuto


orials – X ICS
SE – Geograph
hy 95
96

Example:
 Carelessness and the deliberate dumping of wastes and oil spills in the seas and oceans
pollute water and damage beaches, Marine pollution is a great threat to sea-life (plants and
animals). Oil spills decrease the penetration of light and hamper the photosynthesis process.
They also retard the rate of oxygen uptake by water and adversely affect the development of
marine organisms, increase their susceptibility to disease and affect their reproductive
processes.
 They also lead to gastrointestinal irritation, liver and kidney problems and damage the nervous
system.

Case study: Minamata tragedy


 Minamata, a coastal town of Japan, had a factory of Vinyl Chloride in 1952. it used to
discharge effluents with methyl mercury into the sea. Methyl mercury can break the barrier
between blood cells and nerve cells. Thus, it reaches brain to cause progressive irreversible
damage.
 In 1953 some fishermen fell ill in Minamata and their illness was detected to be caused by
consuming fishes caught from the Minamata Bay. Fishing in Minamata Bay was banned in
1957, as mercury was identified to be the reason for toxicity. The epidemic in Minamata is now
known as Minamata Disease. Mercury in Minamata contaminated and killed different sea bird
species feeding on fishes.
 Mercury contamination also results from wastes of other industries like paper and pulp
industry, chlorine industry, pesticide industry etc. Pesticides with mercury add a considerable
proportion of mercury to natural water bodies.

96 Universal Tutorials – X ICSE – Geography Volume 2 of 4


Chapter 17: Safe Disposal of Waste 97

Chapter 17: Safe Disposal of Waste


Segregation
¾ In industrialised countries like Japan, the waste is segregated before it is disposed of. Even in
colonies various types of dustbins are used to segregate glass, metals, paper, cloth, etc., and
each type is handled separately by reusing it, recycling it or disposing it in any other accepted
waste disposal method.
¾ It should be the duty of each household to segregate domestic waste into different dustbins like
biodegradable and non-biodegradable and then convert biodegradable ones into other useful
products like compost or gobar gas.
¾ Urban residential colonies should undertake collective efforts for safe disposal of domestic waste
as well as sweepings from the gardens and public parks. These sweepings can be converted into
compost and used for the maintenance of these gardens and parks.

Dumping
¾ In this method, waste materials are dumped in open low lands far away from the city. This method
is not environment friendly.
¾ However, this is the cheapest method and does not need much planning. The open pits spoil the
sight of the area and become a breeding ground for mosquitoes, files insects etc. that are the
carriers of harmful diseases.
¾ They give out foul odour. The burning of waste material in the open dumps pollutes the air.
¾ Another danger of open dumping is that rainwater could carry the harmful substances to the
nearby streams, ponds or lakes and if the water seeps down it could pollute the groundwater.

Sanitary Landfill
 In this method, the waste is packed and dumped daily at the site and is covered with earth to
prevent insects or rodents from entering into the landfill. The waste then is subjected to
bacterial decomposition. Physical, chemical and biological reactions take place generating
different gases like carbon dioxide, methane, ammonia and hydrogen sulphide.
 The sanitary landfill system of disposing of waste is essentially a biological method. The waste
undergoes the following five phases:
z During the first phase of operation, aerobic bacteria deplete the available oxygen and as a
result the temperature increases.
z In the second phase, anaerobic conditions become established and hydrogen and carbon
dioxide are evolved.
z Phase three establishes population of bacteria and the beginning of methanogenic activity,
i.e. production of methane from the decomposition of organic matter.
z In the fourth phase the methanogenic activity becomes stabilized.
z The fifth phase depletes the organic matter, and the system returns to aerobic state.

Advantages
z It is free from air pollution from burning.

Volume 2 of 4 Universal Tutorials – X ICSE – Geography 97


98
z The health problems are minimised since files, rats and other pests cannot breed in the
landfill because of the covered wastes a it is mostly free from fire hazards.

Plantation at Landfill Site


 A vegetative cover should be provided over the landfill site in accordance with the following
specifications:
z Locally adopted non-edible perennial plants that are resistant to drought and extreme
temperatures should be planted.
z The plants grown should be such that their roots do not penetrate more than 30 cms. This
condition should apply till the landfill is stabilised.
z Selected plants should have the ability to thrive on low-nutrient soil with minimum nutrient
addition.

Municipal Waste Management:


Collection of Municipal Solid Wastes
 The following steps should be taken by the municipal authorities:
z Organising house-to-house collection of municipal solid wastes.
z Devising collection of waste from slums and squatter areas or localities including hotels,
restaurants, office complexes commercial areas.
z Bio-medical wastes and industrial wastes should not be mixed with municipal solid, wastes.
z Horticultural and construction wastes should be separately collected and disposed of.
z Waste (garbage, dry leaves) should not be burnt.
z Stray animals should not be allowed to move around waste storage facilities.

Storage of Municipal Solid Waste


 Storage facilities as following:
z Storage facilities should be set up and established by taking into account quantities of
waste generation in an area and its population density. The storage facility is to be so
placed that it is accessible to users;
z These facilities are to be so designed that wastes stored are not exposed to open
atmosphere and are aesthetically acceptable and user-friendly;
z Storage facilities or ‘bins’ should have ‘easy to operate’ design for handling, transfer and
transportation of waste.
z Manual handling of waste should be avoided.

Transportation of Municipal Solid Wastes:


 Vehicles used for transportation of wastes should be covered. This prevents the wastes from
being scattered. Waste should not be visible to public, nor exposed to open environment

Segregation of Municipal Solid Wastes:


¾ The municipal authorities should undertake phased programme to ensure community participation
in waste segregation.

98 Universal Tutorials – X ICSE – Geography Volume 2 of 4


Chapter 17: Safe Disposal of Waste 99

Composting:
¾ Composting of waste is an aerobic (in the presence of air) method of decomposing solid wastes.
¾ The process involves decomposition of organic waste into humus known as compost which is a
good fertiliser for plants.
¾ The composting process produces carbon dioxide and heat which can be used for various
purposes like cooking.
¾ The organic wastes from households are made to undergo decomposition in such a way that
bacteria and other micro-organisms break them down and produce a safe, clean and soil-like
material called compost.
¾ The micro-organisms help to stabilise the organic matter. For example, fungi starts working in the
first week after dumping of the material.
¾ Moisture content is an important factor in aerobic composting.
¾ It may be necessary sometimes to add water to maintain moisture content.

Mechanical Method:
 In the mechanical process, used in Bengaluru and adopted by other cities in the country.
 The waste material is placed in layers about one metre deep. The material is riot turned at all
but it decomposes completely in about five months. This method 0f composting is known as
the Bengaluru Method.

Manual Method
 In India, both the manual as well as mechanical methods of composting are used.
 In the rural areas composting refuse and night soil mixtures are dumped together which
produce manure for the fields. Layers of vegetable waste and night soils are alternated in a
shallow hole dug in the ground. The mixture is turned regularly for about three months to
provide air to the mixture. Then the compost is left for another month without turning for the
process to take effect. This method of composting is known as the Indore Method.

Advantages of Composting
 The major benefits of composting are:
z It enhances soil nutrients and water retention capacity of soils.
z It suppresses plant diseases.
z It rejuvenates poor soils by adding humus.
z It absorbs odours and degrading volatile organic compounds.
z It prevents pollution by preventing pollutants in storm water run-off from draining into water
resources.

Drainage and Treatment of Effluents


¾ Waste water, both domestic and industrial, has undesirable components which are harmful to the
environment and human health. These contaminants need to be removed from waste waters so
that the treated water can be used for beneficial purposes.
¾ The treatment of waste water is carried out to the required degree in three basic stages.

Volume 2 of 4 Universal Tutorials – X ICSE – Geography 99


100
¾ These are – Primary, Secondary and Tertiary treatment stages. However, the specific stages
required for water treatment may vary depending on the quality of the waste water and the desired
quality of water, as well as the cost of treatment.

Primary Treatment:
 The primary treatment of waste water includes physical separation of suspended solids as well
as odour and colour.
Screening and Settling:
z It is the first stage of primary treatment which is used to remove coarse solids like sticks,
rags, boards and other large objects from waste water. Here, the waste water is passed
through screens such as a wire mesh, a perforated sheet or a set of parallel steel bars
which remove the coarse particles.
z Waste water also contains inorganic solids such as pebbles, sand, silt, egg shells, glass
and metal fragments along with heavier organic matter such as bonechips, seeds etc.
Together these materials are known as grit.
z To remove these impurities, the waste water is made to enter a grit chamber where the flow
velocity of water decreases. This reduced flow velocity of waste water allows the grit to
settle down. The grit is then removed from the grit chamber either by hand or mechanically.
z Water free from suspended coarse solids and grit then flows into a sedimentation tank.
Sedimentation:
z In this process, the water is first of all stored in large basins. Storing the water for a long
time in reservoirs, allows sand particles, clay, silt and other suspended materials to settle at
the bottom of the basin as sludge. The sludge is removed as underflow either by vacuum
suction or by taking it to a discharge point at the bottom of the basin for withdrawal. The
clear liquid obtained above is called the overflow and contains no readily settleable matter.
Coagulation or Flocculation:
z In this process, the sedimented water is subjected to a chemical process. The primary
purpose of this process is to remove turbidity from water. Turbidity is a cloudy appearance
of water caused by small particles suspended therein. Water with high turbidity is difficult to
disinfect properly. By flocculation turbidity as well as many suspended bacteria in the water
is removed.
z A coagulant such as aluminium sulphate or alum is added to the water. The coagulant
facilitates the agglomeration or flocculation of fine particles suspended in water. The
coagulant reacts with water and forms a bulky precipitate. The agglomerated particles,
known as floccules, are again separated by sedimentation or filtration.
Filtration:
z It is a physico-chemical process for separating suspended impurities from water by passing
it through a barrier of sand matrix of fibrous material or coal particles. This process removes
microorganisms as well as the remaining fine particles

Secondary Treatment of Water


 Water treated by primary treatment still contains a large number of pathogenic arid non-
pathogenic organisms as well as high quantity of organic matter. The organic matter has a high
demand for oxygen which must be reduced so that the waste water becomes suitable for
discharge into water bodies. Therefore, it has to undergo the process of secondary treatment.
 In this method, the organic matter is biologically degraded by microorganisms. Hence, it is also
known as biological treatment.
100 Universal Tutorials – X ICSE – Geography Volume 2 of 4
Chapter 17: Safe Disposal of Waste 101
 The waste water after primary treatment enters a tank where the organic matter is brought into
contact with sludge, heavily laden with micro-organisms. Air is introduced into the tank in the
form of bubbles through diffusers. The micro-organisms in the sludge digest the organic matter
in the waste water coming from the primary treatment tank and reduce the HOD (Biological
Oxygen Demand) to an acceptable low level.
 After this, the water leaves the aeration tank and carries with it the suspended biomass, i.e.,
the sludge having organic matter.
 A part of this biomass is used again to consume more organic matter from waste water and
hence named ‘activated sludge’.
 The remaining sludge is removed from the system and disposed of as landfill. The treated
water, free from impurities flows to another tank. This method of waste water treatment is
known as ‘active sludge’ method.

Tertiary Treatment of Water:


 Tertiary treatment, also called Advanced Waste Treatment, is the collective name given to the
processes that are used to improve the quality of water that has received primary and
secondary treatment.
 The pollutants removed in this process may include nutrients such as phosphorus or nitrogen,
which may support algal growth in water bodies; and organic materials such as pesticides and
insecticides which give a peculiar colour, taste and odour to water and lead to health problems.
 The purpose of tertiary treatment is to provide water quality adequate for industrial, agricultural,
domestic and recreational uses.
 The purpose of the treatment is also to minimise the probability of human exposure to viruses
and other pathogens in order to get rid of the micro–organisms, chlorine or bleaching powder is
added to the water to disinfect it. Sometimes disinfection is carried out by using ultra-violet
light, ozone or silver ions.

Incineration
¾ Incineration is the process of controlled high temperature oxidation of primarily organic
compounds that release thermal energy and produce carbon dioxide and water.
¾ In short, incineration involves burning of wastes at a very high temperature. The waste to be burnt
is fed into an incineration chamber (or kiln) and combustion consumes or destroys the organic
component.

Advantages of Incineration
 incineration as a system of waste disposal has the following advantages:
z It is a useful technology to deal with large quantities of organic hazardous wastes that have
high calorific value and cannot be dealt with by other methods.
z Incineration kills pathogenic organisms and reduces the volume of the waste.
z It is useful to dispose of petroleum and plastic wastes in chemical industries.
z Incineration of combustible waste produces much heat that can be used to produce steam
from water which in turn can be utilised to generate electricity.

Scrubber
 Scrubber is a device that employs spray of water to catch pollutants during emissions. In this
process, water is introduced into a spray tower by means of a spray nozzle which allows
downward flow of water. As the polluted gas flows upwards, the particulates present in the gas

Volume 2 of 4 Universal Tutorials – X ICSE – Geography 101


102
ccollide with the water droplets
d sprrayed from the spray nozzles.
n Under the influence of
g
gravitational quid droplets containing th
force, the liq he particulates settle at the bottom.

Electrostatic
c Precip
pitators (ESPs)
(
¾ ESP
P is a device that is used
d for removin
ng a wide ran nge of particu
ulates including mists fro
om gases.
ESP
Ps are used in
i power plan nts, smelterss, cement pla
ants and blasst furnaces.
¾ Elecctrostatic pre
ecipitator works on the e principle of
o electrosta
atic precipita
ation, i.e. electrically
e
charrged particulates presen nt in the polluted gas arre separated
d from the ggas stream under
u the
influ
uence of the electrical fielld.
¾ In th
he removal process,
p the polluted gas is made too enter the Electrostatic
E Precipitator.. The gas
flow
ws upwards between
b the high voltage wire and gro
ounded colleecting surfacee. The high voltage
v in
the wire lonises the gas and d the negativve ions move towards thhe grounded surface and d pass on
theirr negative ch
harge to the dust
d particles.
¾ The en these neg gatively chargged dust particles are drrawn toward
ds the positivvely charged
d particles
colle e, where they finally get deposited. Knocking
ector surface K on these platess makes the e particles
fall into
i a hopper tray for disp
posal.

Adva
antages of
o ESPs
 T ges of using ESPs are th
The advantag he following:
z ESPs are economical to operate.
z They provvide high efficiencies, uptto 99 per cen
nt.
z They are dependable and predicta able.
n produce a moisture plume.
z They do not

102 Universal Tuto


orials – X ICS
SE – Geograph
hy Volu
ume 2 of 4
Chapter 18: Reduce-Reuse-Recycle 103

Chapter 18: Reduce-Reuse-Recycle


Introduction:
¾ In the modern industrial world, waste has become an environmental and public health hazard.
Waste can be effectively managed by using the following three strategies:
 Reducing the Waste;
 Reusing the Waste; and
 Recycling the Waste.

Reducing the waste


¾ Change of Process: By changing a modem method to make the best use of raw materials
reduces the waste generation in industries. In zinc electroplating chlorides are used in the process
instead of using the sulphate salt in order to eliminate the production of cyanides.
¾ Waste Concentration: By using scientific techniques such as precipitation and evaporation the
amount of liquid waste can be reduced. Incineration can be used to get rid of inflammable wastes.
¾ Segregation of Waste: First of all, non- hazardous waste are separated from hazardous waste
rather than dumping them together.

Reusing the Waste:


¾ In our houses and in industries many materials are discarded as wastes. These materials have
some value, for instance glass, metal pieces, rubber, wood fibre and paper products.
¾ In developing countries like India, some of the waste materials like old glass bottles, steel tyres, tin
cans are reused. For example, shoes or chappals are made from old tyres, water bags are made
from leather, lamps are made from tin cans, etc. Many waste collectors roam about in residential
areas and industrial units to collect the solid wastes.
¾ They segregate them and supply them to specialised artisans who make utility articles from such
material and make a living from their skill.
¾ For example, a beautiful garden (Rock Garden) has been created by Nek Chand in Chandigarh
using waste products like tin cans, bottles, broken pieces of crockery, etc. Waste collectors, thus,
help in making new production processes. Thus, they reduce the burden of waste disposal.
¾ Some solid wastes from the industry can be utilised directly. Flyash from power plants is used as a
cement substitute.
¾ Bricks are made from flyash. Flyash is also used in making of the roads and filling up low-lying
areas.

Recycle of Waste:
¾ Besides reusing the materials by using physical processes, we also use recycling process by
treating the waste before it is used in a manufacturing process.
¾ Example: In India, we have tonnes of bagasse from sugarcane during a particular season.
Bagasse can be used in the manufacture of paper pulp. This would save trees which are normally
used for making paper pulp. Bagasse is also used for making packaging material of dairy

Volume 2 of 4 Universal Tutorials – X ICSE – Geography 103


104
products. Paper industry recycles pieces of wood from furniture industry, used and discarded cloth
and used paper.

Plastic:
 Plastics are recycled by plastic manufacturers. About 80 per cent of the plastic waste is
recycled in India,
 Plastic is non- biodegradable. The bonds of carbon in plastic are impossible to break down
through a physical or chemical process. They have to be incinerated, recycled or buried in
landfills.
 The plastic bags which are extensively used in India are made from recycled plastic. The
recycled plastic bags are harmful because the melting of plastic and plastic products breaks
some polymer chains into smaller units which are harmful.

Paper:
 The paper industry segregates waste paper from a huge discarded dump. Waste paper has
grit, sand, ink, tar, paper clips, plastic coatings, rubber bands, etc.
 Most of the used paper is made into cardboard, paperboard, paper bags, etc. Recycled paper
is used for printing only a few times because with each recycle the fibres become weak.
 The recycled product is weaker than similar product made from fresh fibres.
 Waste paper is fed on to a conveyor belt. The conveyor belt feeds the paper to a hydropulper
where it is smashed in water at about 38°C.
 The force in the hydropulper is so great that the sheets of paper are rapidly broken down to a
slurry.
 The fibres in the paper are retained and unacceptable materials are drained off. The pulp of
waste paper is passed over a riffer system. It is then fed into another unit for the removal of dirt
and tar.
 The paper slurry is thickened before it is fed into the paperboard machines.

Government initiatives
¾ The Central and State Governments own, control and develop a country’s forests, dams, major
irrigation systems, power stations, industries, means of transportation, railways, roads, ports, etc.
¾ The Government is not just the protector of the country’s environment but also has a major
responsibility for sustaining environmental conscience.
¾ In India, the Ministry of Environment and Forests is the main nodal agency for generating
environment consciousness and making and implementing schemes for environmental protection.
¾ The Government’s environmental policy focuses on the following areas:
 To check degradation of land and water through Wasteland Management and Restoration of
river water quality programmes;
 To provide for conservation of natural resources by direct action such as declaration of
reserved forests, biosphere reserves, wetlands, mangroves and protection of endangered
species;
 To monitor development through Environmental Impact Assessment Studies of major project
proposals; and
 To make laws and acts for environment protection and to initiate penal measures against those
who violate these laws.

104 Universal Tutorials – X ICSE – Geography Volume 2 of 4


Chapter 18: Reduce-Reuse-Recycle 105
¾ The Environment Protection Act (1986) empowers the Central government to coordinate actions
of State Governments, plan and execute a nationwide programme for the prevention, control and
abatement of environmental pollution.
¾ The government has a major role in environmental protection. It is the government’s duty to find
out ways and means of improving efficiency of existing technologies and introduce new eco-
friendly technologies.
¾ Eco-friendly technology is based on renewable resources as raw material as well as energy; and
transformation through highly efficient biotechnology to produce environment friendly products.
¾ For example, to reduce vehicular pollution in Delhi, the government initiated the development and
use of CNG in automobiles instead of petrol and diesel. This has reduced air pollution in Delhi to a
considerable extent.
¾ The government sets up various committees under the charge of experts to evaluate the impact of
various projects on environment. An important issue which has been hotly debated in recent
decades is the adverse effects of constructing big dams.
¾ Environmental activists have been urging the Government not to construct big dams. For the
construction of such big dams large areas of forest and agricultural lands get submerged in water.
These developments have adverse environmental and socio -economic consequences.
¾ Building of a dam disrupts the ecosystem of the river and destroys the aquatic as well as
terrestrial life around it.
¾ Another serious consequence of building big dams is the large scale displacement of people from
their homes. Displacement disrupts the entire life-cycle of the people. People lose their ancestral
home as well as their community life and traditional occupation. They have to face the
uncertainties of resettling in unfamiliar and often inhospitable locations.
¾ The Government has to ensure that environmental standards are strictly followed to prevent any
damage to the environment caused by the construction of big dams.

Social Initiatives
¾ Environmental protection is not the responsibility of the government alone. All sections of the
society have to participate in this endeavour. It is ultimately the society that suffers due to
environmental degradation.
¾ Therefore, the society has to play an important role in maintaining environmental standards in the
following ways:
 Society is made of individuals together. So it is the duty of each individual to see that his / her
actions do not pollute the environment.
 If air and water resources in an area are unfit and do not meet the acceptable standard, the
people of the area can organise themselves and force the responsible agencies to take
necessary action.
 If suitable action is not forthcoming they can, under the laws of the land, file a Public Interest
Litigation (PIL) and get their problems solved.
 Groups of individuals together can make a huge difference in maintaining environmental
standards. For example, group housing societies can initiate steps for waste management by
making provisions for segregating wastes, taking measures for recycling wastes like making
compost pits, etc.
 They can also take measures for reducing the use of electricity and finding alternative sources
of energy.
 Air pollution, specially vehicular pollution can be uinimised by adopting car pool method. This
will also save huge amounts of money spent on importing petroleum from other countries.

Volume 2 of 4 Universal Tutorials – X ICSE – Geography 105


106
z Community can play a significant role in environmental protection by creating awareness
and educating people about the need to conserve and manage natural resources.
z Group Housing Societies and Residents Welfare Associations (RWAs) can play a
significant role in conserving natural resources. For example:
„ they can make provisions for rainwater harvesting in their colonies to conserve rain
water;
„ they can provide infrastructure for using solar energy in place of electricity;
„ they can make arrangements for collecting and segregating waste and turning the
biodegradable waste into compost:

Individual Initiative:
¾ The role of every individual in environmental protection is of great importance because if every
individual contributes substantially, the effect will be visible not only at the community, city, state
or national level but also at the global level.
¾ It can be done by following ways:
 Use carry bags made of paper or cloth instead of polythene.
 Help more in pollution prevention than pollution control.
 Use eco-friendly products.
 Cut down the use of chlorofluorocarbons (CFCs) as they destroy the ozone layer. Do not use
styrofoam cups that have chlorofluorocarbon (CFC) molecules in them which destroy ozone
layer.
 Use chemicals derived from peaches and plums to clean computer chips and circuit boards
instead of CFCs.
 Use CFC-free refrigerators.
 Save electricity by not wasting it when not required because electricity saved is electricity
generated without polluting the environment.
 Adopt and popularise renewable energy sources.
 Promote reuse and recycling wherever possible and reduce the production of waste,
 Use mass transport system. For short- visits use bicycle or go on foot, Decrease the use of
automobiles.
 Use rechargeable batteries. Rechargeable batteries will reduce metal pollution.
 Use biodegradable dish washing liquid, laundry detergent and shampoo. This will reduce
eutrophication of water bodies.
 Use organic manure instead of commercial inorganic fertilizers.
 Plant more trees, as trees can absorb toxic gases and purify the air by releasing oxygen.
 Reduce the use of paper by using computer storage system. Recycle used paper. This would
reduce demand on wood and save trees.

106 Universal Tutorials – X ICSE – Geography Volume 2 of 4


Chapter 18: Reduce-Reuse-Recycle 107

REVIEW QUESTION AND ANSWER:

Section I: [2 Marks]
1) What is waste? Which is most common place of disposal?
Ans: Waste is any material or substance that is of no further use and has been discarded.
Most of the domestic waste is dumped on land in the form of refuse.
2) Mention any four waste producing sources.
Ans: Industry, agriculture, household and municipal.
3) Mention any three types of domestic wastes.
Ans: Paper, plastic goods and batteries.
4) Mention any three types of industrial wastes.
Ans: Mining, oil refineries and nuclear power plant.
5) Mention any two kitchen wastes.
Ans: Fruit and vegetables peels.
6) Mention any two fuels used by cement industry.
Ans: Coal and petroleum.
7) Define tanneries.
Ans: Tanneries are the factories where leather is made from the skirts of dead animals. These skins
are tanned with chemicals.
8) “Tanneries are major source of pollution.” Justify.
Ans: Tanneries are major source of pollution because tanneries use harmful chemicals.
9) Mention the major raw materials used in the generation of nuclear power.
Ans: Uranium, plutonium and Thorium.
10) What is agricultural waste?
Ans: Unusable materials, liquid or solid, produced as a result of various agricultural operations is
termed as agricultural waste.
11) Mention any two agricultural wastes.
Ans: Waste from agriculture, poultry and harvest waste.
12) What is incineration?
Ans: The process of burning waste in large furnaces is known as incineration.
13) Mention the types of waste on the basis of decomposition.
Ans: Biodegradable and Non-biodegradable
14) What is biodegradable waste? Give two examples.
Ans: Biodegradable waste includes that material which breakdown or decompose in the soil, e.g.,
fruits and vegetables.
15) What is non–biodegradable waste? Give two examples.
Ans: Non-biodegradable waste includes that material which does not breakdown or decompose in the
soil, e.g., polythene bags and synthetic clothes.
16) What is complex biodegradable waste?
Ans: The waste material which does not decompose easily and has resistance to break down are
called Complex biodegradable waste.
17) What is simple biodegradable waste?
Ans: The waste that gets decomposed quickly through natural processes like dead remains of
animals and plants, faecal matter etc. are called Simple biodegradable waste.
18) What is biomedical waste?
Ans: Biomedical waste (BMW) consists of solids, liquids, sharps and laboratory waste that are
potentially infectious or dangerous and are considered biowaste.

Volume 2 of 4 Universal Tutorials – X ICSE – Geography 107


108

19) Name the places that generate the biomedical waste.


Ans: Hospitals, clinics and nursing homes.
20) What type of waste is released from the thermal plants?
Ans: Gaseous waste.
21) Why is styrofoam harmful to the environment?
Ans: It is not recyclable and also releases toxic chemicals like styrene, especially when heated.
22) a) What is municipal waste?
b) Mention any two non-biodegradable municipal wastes.
Ans: a) Municipal waste includes solid and liquid waste generated by homes and offices.
b) Plastic and glass.
23) “We should not throw plastic bags in open”. Give two reasons.
Ans: i) Plastic is a non-biodegradable waste which does not breakdown or decomposes in the soil.
ii) Plastic waste clogs the drains.
24) “The agricultural waste, if handled or managed properly, can be useful to man”. Justify by
giving two reasons.
Ans: i) The agricultural waste can be used to generate energy.
ii) Animal waste alongwith other plant material can be used in a compost to make manure.
25) How textile industries cause pollution?
Ans: A large amount of water is used to dye the flberes. When this water released as waste, it gets
mixed with a number of chemicals and becomes toxic.
26) What do you understand by landscape spoilage?
Ans: Landscape spoilage refers to the unpleasant looks presented by an area. It adversely affects the
health of people and their living standard and environment also.
27) “The waste accumulation damages the aesthetic of the area.” Justify by giving two
reasons.
Ans: i) When the waste is dumped in an improper way, the landscape of the area gives unpleasant
look.
ii) The uncontrolled dumping of urban waste damages the beauty of the countryside.
28) How indiscriminate dumping of waste cause adverse effects on public health?
Ans: i) Diseases like diarrhea, dysentery, malaria and plague are the, result of such dumping of
wastes.
ii) Areas of waste accumulation normally become the breeding place of insects, rats, flies etc.
29) What are the main sources of Municipal Waste? Mention any four.
Ans: Household waste, commercials, street sweeping, hotels and restaurants, clinics and
dispensaries, construction and demolition, horticulture and sludge.
30) What is open dumping?
Ans: An open dumping is defined as a land disposal site at which solid wastes are disposed off in a
manner that does not protect the environment, are susceptible to open burning and exposed to
the elements, vectors, and scavenges.
31) Mention any two health effects of open dumping.
Ans: i) Rodents, insects and other vermin attracted to open dump sites may pose health risks. Dump
sites with scrap tires provide an ideal breeding ground for mosquitoes, which can multiply 100
times faster than normal in the warm stagnant .water.
ii) If solid waste is burnt it may release harmful gases which can harm the health of nearby
people.
32) Name the gases contributing potentially to Global Warming and Climate Change
phenomenon.
Ans: Carbon dioxide, methane and nitrous oxide, CFC also.

108 Universal Tutorials – X ICSE – Geography Volume 2 of 4


Chapter 18: Reduce-Reuse-Recycle 109

33) Mention any four sources of industrial waste.


Ans: Chemical units, thermal power plants, petrochemical plants and oil refineries.
34) What is a radioactive pollution?
Ans: The radioactive pollution is defined as the physical pollution of air, water by the radioactive
materials. The ability of certain materials to emit the proton, gamma rays and electrons by their
nuclei is known as the radioactivity.
35) What is Eutrophication?
Ans: The process by which a body of water acquires a high concentration of nutrients, especially
phosphates and nitrates is known as eutrophication.
36) Name the diseases leads by polluted air.
Ans: Asthma, bronchitis, emphysema and lung cancer.
37) Mention any two harmful gases.
Ans: Carbon dioxide and hydrogen sulphide
38) Mention any four sources that lead to radioactive pollution.
Ans: Nuclear power plants, nuclear weapons, transportation, disposal of nuclear waste and uranium
mining.
39) Mention any three sources of marine pollution.
Ans: Drilling of oil, accidental and deliberate discharge of crude oil and dumping of industrial waste
into ocean.
40) What is biomagnifications?
Ans: The increasing concentration of a substance, such as toxic chemical, in the tissues of organisms
at successively higher levels in a food chain. As a result of biomagnifications, organisms at the
top of the food chain generally suffer greater harm from persistent toxin or pollutant than those at
lower level.
41) Name the gases released in the process of mining.
Ans: Methane and carbon monoxide.
42) Mention any two serious effects of radiation.
Ans: i) Radiation can lead to severe health hazards.
ii) Effects of non-ionizing radiation at low levels are certain but ionizing radiation at high levels
can cause cancer and increase in chromosome damage.
43) How does groundwater get polluted?
Ans: Various types of chemicals are released as waste by the factories and agricultural practices.
Many of these chemicals are soluble in water. The water slowly seeps down to join underground
water.
44) Name any two organisms that spread the virus among people.
Ans: Mosquitoes and flies
45) How do workers in matchbox factory get affected from pollution?
Ans: Workers that work in the matchbox factory is exposed to phosphorous which causes bone
diseases.
46) What is oil spilling?
Ans: Release of liquid petroleum/ mineral oil into the environment is called oil spilling.
47) Mention any four sources of oil pollution.
Ans: Oil spill, garbage dumping, accumulation of toxic materials and industrial waste.
48) Which is the most serious pollutant in the air?
Ans: Sulphur dioxide.
49) Mention any two harmful effects of carbon monoxide.
Ans: i) It affects the oxygen carrying capacity of blood.
ii) In some cases it can have negative impact on the vision also.

Volume 2 of 4 Universal Tutorials – X ICSE – Geography 109


110

50) Name a pollutant which is released by vehicles.


Ans: Lead.
51) What is Waste Management?
Ans: Waste management is the collection, transport, processing, recycling or disposal of waste
materials.
52) Mention the forms of safe disposal of waste.
Ans: Solid, liquid and gaseous.
53) Mention any three methods of solid waste disposal.
Ans: Segregation, dumping and composting.
54) What is segregation?
Ans: Segregation refers to the separation of waste into different categories of waste. Segregation
depends upon the waste disposal technique. The most popular segregation is biodegradable
and non-biodegradable.
55) Mention any three forms of solid waste.
Ans: Garbage, construction debris and industrial waste like chemicals.
56) What is dumping?
Ans: Dumping is an activity of depositing waste in an area.
57) What is composting?
Ans: Composting is a biological process in which microorganisms, mainly fungi and bacteria, convert
degradable organic waste into humus-like substance.
58) What is incineration?
Ans: Reduction in weight and volume of solid waste disposal is called incineration.
59) What is sewage?
Ans: The wastewater discharged from domestic premises like residences, institutions and commercial
establishments is known as sewage.
60) What is air scrubber? Mention two methods of disposal.
Ans: The air scrubbers are the anti-pollution devices used to trap the particles from the emissions of
the gaseous waste. Air scrubber and Electrostatic precipitators.
61) What is electrostatic precipitator?
Ans: Electrostatic precipitators is an electric device to remove suspended impurities like dust, mist,
fume etc. from the air.
62) Name the 3Rs of waste management.
Ans: Source Reduction, Reuse and Recycle
63) Mention the steps involved in the process prior to recycling.
Ans: i) Collection of waste from doorsteps, commercial places etc.
ii) Collection of waste from community dumps.
iii) Collection/ picking up of waste from final disposal sites.
64) Name any four products which can be recycled and reused.
Ans: Old copies, bottles, plates and clothes.
65) Which is the apex administrative body in India for environmental protection?
Ans: National Council for Environmental Policy and Planning.
66) Mention two examples of preliminary operations.
Ans: i) Screening and communication for the removal of debris and rags.
ii) Grit removal for the elimination of coarse suspended matter that may cause wear or clogging
of equipment.
67) What is the principal function of primary treatment?
Ans: To act as a precursor to secondary treatment
68) Mention any two advantages of waste recycling.
Ans: i) It leads to less utilization of raw materials.
ii) It reduces environmental impacts arising from waste treatment and disposal.

110 Universal Tutorials – X ICSE – Geography Volume 2 of 4


Chapter 18: Reduce-Reuse-Recycle 111

69) What is source reduction?


Ans: Source reduction can be defined as a product that results in a net reduction in the generation of
waste compared to the previous or alternate version.
70) Mention any two devices which help in gaseous waste disposal.
Ans: Air scrubber and electrostatic precipitators.
71) What is pyrolysis?
Ans: Pyrolysis or destructive distillations is a process to decompose solid waste chemically by heat in
oxygen reduced atmosphere.
72) Name any two legal provisions for waste management.
Ans: i) Environment Protection Act (1986).
ii) Biomedical Waste (management and handling) Rules 1998.
73) What is the main objective of primary waste management treatment?
Ans: To remove organic matter.
74) Which is the most common method used for primary waste management treatment?
Ans: Sedimentation.
75) Name any three items which can be recycled.
Ans: Paper, plastic and glass.
76) “The removal/ control of nutrients in wastewater treatment are important for several
reasons.” Explain.
Ans: i) Wastewater discharges to confined bodies of water cause or accelerate the process of
eutrophication.
ii) Wastewater discharges to flowing streams reduce dissolved oxygen.

Section II: [3 Marks]


1) Mention the waste generated by following industry units.
(a) Cement factories (b) Oil refineries (c) Construction units
Ans: a) Dust and harmful gases.
b) Waste water, chemicals and harmful gases.
c) Concrete, plaster, metal, wood etc.
2) How the following pollutes the environment?
(a) Thermal plants (b) Nuclear plants
Ans: a) i) Thermal plants release harmful gases in the air which causes air pollution.
ii) Burning of coal leaves behind ash which causes air water and soil pollution.
b) i) Uranium and Plutonium are the raw material used in the generation of nuclear energy.
These wastes remain radioactive for thousands of years.
ii) These wastes release harmful radiations which can lead to air, water and soil pollution.
3) How the following products add to environmental pollution?
(a) Styrofoam (b) Glass
Ans: a) i) It is not recyclable.
ii) It releases toxic chemicals like styrene, especially when heated.
b) i) Many organic and inorganic material are used to make glass.
ii) Plastic, which is non-biodegradable, is also used to make glass.
4) Distinguish between biodegradable and non- biodegradable waste.
Ans:
Biodegradable Waste Non-biodegradable Waste
These can be decomposed through the These cannot be decomposed through the
action of bacteria fungi and other living action of bacteria, fungi and other living
organism. organisms.
These include fruit and vegetable peels, These include plastic, cans, glass etc.
grass, wood, paper, clothes, dead remains of
plants and animals etc.
Most of these don’t cause much harm to the Most of these are very harmful for the
environment. environment.

Volume 2 of 4 Universal Tutorials – X ICSE – Geography 111


112

5) “Plastic is a non-biodegradable product.” Explain.


Ans: i) Plastic causes serious damage to environment during its production process and during its
disposal process.
ii) Some of the constituents of plastic, such as benzene and Vinyl chloride, are proved to cause
cancer, and other gases and liquid hydrocarbons spoil earth and air.
iii) When plastic is burn, it has its own disadvantages; as when burn, plastic releases a host of
poisonous chemicals including dioxin into the air.
iv) Plastic wastes clog the drains and thus hit especially urban sewage system. The plastic
wastes being dumped into rivers, streams and sea contaminate the water, soil, marine life
and also the air we breathe. Choked drains provide excellent breeding grounds for
mosquitoes besides causing flooding during the rainy seasons.
6) “Threshing of food grains generates a lot of waste.” Explain.
Ans: i) Threshing release large amount of straw and dust in the atmosphere.
ii) Threshers working in the fields releases harmful gases.
7) Describe the waste generated from the mining operations.
Ans: i) A lot of waste is generated in the mining operations. To reach the underlying mineral deposits
in earth, a lot of top soil has to be removed. In some cases the overlying rocks are removed.
A lot of dust is also released into the atmosphere. Tailings are another form of waste, i.e., the
waste material from the ore.
ii) Tailing is a significant threat to the environment because it can result in the generation of
acids and alkaline and alkaline drainage. Mining of uranium is risky because of random gas
that is released during the mining.
8) With reference to tanneries answer the following questions
(i) Define tanneries (ii) How these units pollute the environment?
Ans: i) Tanneries are the factories where leather is made from the skins of dead animals.
ii) In the process of making leather, the tanneries add solid, liquid and gaseous waste. The
liquid waste released by tanneries is very toxic and gives out foul smell. It can even pollute
the groundwater.
9) Mention any two ill effects of sulphur dioxide.
Ans: i) In air, most serious pollutant is the sulphur dioxide. Its concentration in human body causes
cough, breathlessness and spasm of larynx. It also causes irritation in eyes.
ii) It also becomes an allergic agent. When it reacts with some compounds, it makes sulphuric
acid which can damage the lungs.
10) Mention any three ill effects of radioactive substances.
Ans: i) The iodine may affect the White Blood Cells, bone marrow, spleen, lymph, skin cancer,
sterility, eye and damage to the lung.
ii) The strontium has the ability to aggregate in the bones and form a bone cancer and leads to
tissue degeneration.
iii) The radioactive materials are passed through the land to water and cause an adverse effect
on the aquatic animals. They reach to human through the food chain.
iv) The nuclear power generates a lot of energy which is used to run turbines and produces
electricity. The fuel and the coolant produce a large amount of pollution in the environment.
11) Mention any three causes of marine pollution.
Ans: i) Drilling of oil can lead to oil spill which can destroy the marine life. Once oil is spilled it can
neither be removed nor can it be contained because oil and water do not mix.
ii) Accidental and deliberate discharge of crude oil into the ocean by cargo ships is regarded as
one of the prime causes of pollution of the water body.
iii) Dumping of industrial wastes into ocean is another reason for marine pollution. The wastes
often contain toxic materials such as mercury, dioxin and radioactive materials, which
contaminate the water of ocean.
iv) Deposition of sediments from mining leads to ocean pollution.
112 Universal Tutorials – X ICSE – Geography Volume 2 of 4
Chapter 18: Reduce-Reuse-Recycle 113

12) Mention any three effects of marine pollution.


Ans: i) Oil spilling is hazardous for the marine life. It seriously affects the life cycle of coral reefs
thriving in the ocean. The oil spilled in the ocean could clog up the gills of fishes, thereby
preventing respiration. It affects the process of photosynthesis of marine plants, since it
blocks the sunlight.
ii) Toxic waste has direct effect on marine life and affects the human beings indirectly. When the
harmful toxic waste is dumped into the ocean, the fishes could consume the poisonous
chemicals. When the fish is eaten by humans, this could lead to food poisoning.
iii) Carbon dioxide is hazardous for marine life including coral reefs and free-swimming algae.
iv) Plastics dumped into ocean can affect the marine life seriously. Plastic items such as bottles
and bags could choke and suffocate the sea animals, as they eat them thinking that they are
food. Plastics are known to be a major cause for the death of turtles, as they swallow the
floating bags, mistaking them for jelly fish.
v) Dumping of industrial wastes such as pesticides, especially DDT can accumulate in the fatty
tissue of animals. This could lead to the failure in the reproductive system of mammals and
birds.
13) Mention any three effects of solid waste.
Ans: i) Numerous insects, bacteria and virus grow on the routing waste which are capable of
spreading several diseases like diarrhea, cholera etc.
ii) Mosquitoes are known for spreading malaria.
iii) Flies carry the virus and spread it in the adjoining areas. People fall sick and start vomiting.
14) “Terrestrial life is affected with the waste.” Explain.
Ans: Pollutants of waste enter the environment and become a part of food chain which accumulates in
the bodies of organisms at different levels. For example, insecticides and pesticides used by the
farmer mix with the water. Animals drinking that water may get infected. Plants growing in that
area may absorb these chemicals and get infected. Humans consuming these plants may also
get infected.
15) Explain the effects of agricultural waste.
Ans: i) Agricultural activities generate a lot of waste. Although most of agricultural waste is
biodegradable, in most cases it is not handled properly and affects environment adversely.
ii) After harvesting, the crop residue is the major agricultural waste. Farmers often burn this
waste in the fields. It kills the much needed organic life in the soil.
iii) After burning, the ash floats in air and affects the health of people. Activity of threshing also
releases a lot of waste in the form of straw. Its particles remain suspended in air which affects
the health of people, especially the asthmatic patients. Many farmers are involved in dairy
farming which makes the air stinking.
16) Mention any three effects of mining and industrial waste.
Ans: i) Workers are infected with silicosis, a lung disease.
ii) People who work in the deep mines inhale the fine coal dust. It leads to a disease called
black lungs. Many gases are also released in the process of mining like methane and carbon
monoxide.
iii) Workers that work in the matchbox factory are exposed to phosphorus which causes bone
diseases.
17) What is recycling?
Ans: Recycling involves the collection of used and discarded materials, processing these materials
and making them into new products. It reduces the amount of waste that is thrown into the
community dustbins thereby making the environment cleaner and the air fresher to breathe.
18) What is Preliminary wastewater treatment?
Ans: Preliminary wastewater treatment is the removal of such wastewater constituents that may
cause maintenance or operational problems in the treatment operations, processes and ancillary
system. In this treatment a portion of the suspended solids and organic matter is removed from
the wastewater.

Volume 2 of 4 Universal Tutorials – X ICSE – Geography 113


114

19) Mention some benefits of composting.


Ans: i) It supplies essential elements needed by the plants.
ii) It helps reduce the adverse effects of excessive alkalinity, acidity or the excessive use of
chemical fertilizer.
iii) It makes soil easier to cultivate.
iv) It helps keep the soil cool in summer and warm in winter. It aids in preventing soil erosion by
keeping the soil covered.
v) It helps in controlling the growth of weeds in the garden.
20) Why, should sewage be treated before disposal? Explain.
Ans: i) The decomposition of the organic materials contained in waste can lead to the production of
large quantities of harmful gases
ii) Untreated wastewater containing a large amount of organic matter, if discharged into a
river/stream, will consume the dissolved oxygen for satisfying the Biochemical Oxygen
Demand of wastewater and thus deplete the dissolved oxygen of the stream, thereby causing
fish kills and other undesirable effects.
iii) Wastewater may also contain nutrients, which can stimulate the growth of aquatic plants and
legal blooms, thus leading to eutrophication of the lakes and streams.
iv) Untreated wastewater usually contains numerous pathogenic or disease causing
microorganisms and toxic compounds, that dwell in the human intestinal tract or may be
present in certain industrial waste. These may contaminate the land or the water body, where
such sewage is disposed.
21) What is the role of government in saving environment?
Ans: i) Instead of building large multi-purpose projects the government should opt for small dams.
This will help in protecting the forest as lot of forested land is submerged in water because of
dam.
ii) The government should also go for the renewable sources of energy like wind, solar and
biomass energy. These are eco-friendly sources of energy whereas burning of fossil fuels like
coal and petroleum pollutes the environment.
iii) Government should educate the people through publications, films and other materials
aiming at increasing awareness of environmental problems.
iv) Government should frame strict laws and should execute them properly.
22) What is the role of individual in saving environment?
Ans: i) Everybody should follow the techniques of simple living such as the use of solar cooker and
minimum use of cooking gas; closing all taps when not in use; switching off electric
appliances when not in use.
ii) The vehicles should be regularly checked so that minimum smoke emits from them.
23) Give a brief description of various legal provisions of waste management.
Ans: i) Environment Protection Act 1986: This act provides for the protection and improvement of
environment and other connected matters.
ii) Biomedical Waste (management and handling) Rules, 1998: The Central Govt. has made
the Biomedical Waste Rules to safeguard the public and health care workers from the risk
arising from Biomedical Waste.
iii) The Recycled Plastics Manufacture and Usage Rules, 1999: The objective of these rules
is to regulate the manufacture and use of recycled plastics, carry bags and containers. As per
these rules the thickness of the carry bags made of virgin plastics or recycled plastics cannot
be less than 20 microns. Carry bags and containers made of virgin plastic are to be in natural
shade or white.
iv) Motor Vehicles Act, 1989: The Act lays down the responsibilities of the driver of the vehicles
like license, insurance, maintenance of vehicles in proper conditions. The Act also empowers
the centre and state governments to make rules as and when needed.

114 Universal Tutorials – X ICSE – Geography Volume 2 of 4


Chapter 18: Reduce-Reuse-Recycle 115

MISCELLEOUS QUESTIONS / HOME WORK


Sources of Waste:
I) Short Answer Questions
1) What is known as waste? Give two examples of agricultural wastes.
2) Name the solid waste generated by the sugar industry. How is this waste used as a raw material
in another industry?
3) Give two examples of toxic wastes.
4) Give two examples of bio-medical waste.
5) Name the sources of Municipal waste.
6) Distinguish between
a) Toxic and Non-Toxic Waste.
b) Degradable and Non-degradable Waste.
7) What is meant by domestic waste?
8) How does it differ from industrial waste?
9) Why are the effluents from chemical industries harmful?
10) Explain briefly the environmental problems associated with agricultural wastes.
11) a) How do the fertilisers contaminate the groundwater?
b) What is the impact of pesticides and insecticides on human beings and animals?
12) What are bio-medical wastes?
13) a) What is known as Nuclear waste?
b) Give the sources of nuclear waste.
c) Why could the Nuclear waste be injurious?
14) Describe with examples how the following become a health hazard:
a) Industrial Waste.
b) Bio-medical Waste.
c) Nuclear Waste.

Need for Management of Waste


1) Name three diseases which occur because of waster accumulation on land.
2) Name two diseases which are spread through pet animals.
3) Name three water-borne diseases.
4) Name three water-borne diseases.
5) What are the three R’s of waste management?
6) What is Greenhouse Effect? How is it related to Global Warming?
7) State the consequences of Global Warming.
8) What role does the ozone layer play to protect life on earth?
9) What is Acid Rain? How is it caused?
10) Explain the need for management of waste.

Impact of Waste Accumulation


1) What impact does the decomposition of waste in the open areas have on human health?
2) Why are radioactive wastes more hazardous than other wastes?
3) Name the disease caused by mercury contamination in Japan.
4) Name three monuments affected by acid rain.
5) What is known as eutrophication?
6) Explain the impact of waste accumulation on human health.
7) What is acid rain?

Volume 2 of 4 Universal Tutorials – X ICSE – Geography 115


116
8) Why is the handling of solid wastes a major problem?
9) Explain how eutrophication affects aquatic life.
10) What is biomagnification? What can be its effects on human beings?
11) What is known as the Minamata Disease? What caused it?

Safe Disposal of Waste


1) Name two useful products which can be made from biodegradable domestic waste.
2) What service is indirectly done by the rag-pickers for the disposal of waste?
3) Explain the role of segregation of waste in the safe disposal of waste.
4) What is a sanitary landfill?
5) How is waste disposed of in a sanitary landfill?
6) What steps should be taken by the municipal authorities for the safe disposal of solid waste?
7) How is compost made?
8) Give three advantages of using compost.
9) What is meant by the primary treatment of water?
10) Explain the process of sedimentation in the primary treatment of water.
11) Why is the secondary treatment of water known as the biological treatment?
12) What is the need for tertiary treatment of waste water?
13) What is incineration?
14) Explain the process of electrostatic precipitation.
15) Give two advantages of using ESPs

Reduce-Reuse-Recycle
1) What are the three R’s of waste management?
2) Name the processes involved in reducing the waste.
3) Give one example of reusing of waste.
4) What do you mean by recycling of waste?
5) Explain clearly how waste can be reduced by changing the process of production.
6) Give an example of reusing waste effectively.
7) Explain how recycling of waste to produce paper can reduce deforestation.
8) Name the sugarcane waste which can be recycled into useful products.
9) Why should we avoid using polythene carry bags and styrofoam cups?
10) Explain the role of government in waste management.
11) Give three examples to show how social initiatives can help minimise the use of resources.
12) What can an individual do to reduce waste at home as well as in the office?
13) What harm is done to the environment by building big dams?

PREVIOUS YEARS BOARD QUESTIONS:


1) a) i) State the main objective of the treatment of gaseous waste.
ii) Name two common diseases caused as a result of gaseous pollution.
b) What was the cause of the following?
i) The Bhopal Tragedy.
ii) The Minamata Disease.
iii) The Chernobyl Disaster. [2013]
2) a) How does accumulation affect the environment?
b) What is acid rain? Mention two of its effects. [2014]

116 Universal Tutorials – X ICSE – Geography Volume 2 of 4


UT Sample Paper 117

UT Sample Paper: Geography


Recommended Time: 3 hours Max. Marks: 80
Instruction: You will not be allowed to write during the first 15 minutes. This time is to be spent in
reading the question paper. The time given at the top of this paper is the time allowed for
writing the answers.
Attempt all questions from Part I and any five Questions from Part II.
Part I (30 Marks)
(Attempt all question)
Question 1)
Study the Survey of India Map Sheet and answer the following questions: [20]
a) Give the six figure grid reference of:
i) Δ 364 ii) The temple at Rampura
b) Mention any two features seen in the map extract which show that the region has
seasonal rainfall.
c) Calculate the distance in kilometers along the cart–track linking Moti Bhatamal (928758)
and Antroli (953764)
d) Which is the chief form of irrigation shown in the map extract? Give an evidence to justify
your answer.
e) Give one reason to explain why the streams in grid square 9478 do not join a river.
Identify another grid square in the map extract that has similar streams.
f) What do the following represent?
i) Black broken lines in 9575.
ii) Black curved lines in 9879
g) Identify two landforms shown by the contours in grid square 9876.
h) i) What is the general direction of the Balaram Nadi?
ii) Which bank of the main river does the Balaram Nadi join?
i) What is the main occupation of the people living in the area shown in the map extract?
Give one reason to support your answer.
j) What is meant by scale of a map? What is the scale of the map extract provided to you?

Question 2
On the outline map provided, mark and name: [10×1=10]
a) Hyderabad [1]
b) Cotton producing area in south [1]
c) The Satpura Hills [1]
d) The Rann of Kachchh [1]
e) An area of South India with winter rainfall and an area with winter rainfall in North India. [2]
f) Gulf of Manner [1]
g) Mumbai High [1]
h) The capital city of India. [1]
i) An international airport in Eastern India [1]

Volume 2 of 4 Universal Tutorials – X ICSE – Geography 117


118

Part II (50 Marks)


(Attempt five questions from this part)
Question 3)
a) Why does Cherrapunji receive higher rainfall than Shilong? [2]
b) How do the western Ghats affect the rain bearing winds that blow in the region? Name the
winds. [2]
c) Name the parts of India that are affected by cyclones and depressions and explain the
cause of these. [3]
d) Explain giving reasons. [3]
i) Delhi has a high annual range of temperature.
ii) Kolkata gets rain from SE winds.
iii) Mumbai is not hot in summer nor cold in winter.

Question 4)
a) Name two regions where gully erosion is prevalent. What is created during this erosion?
[2]
b) Why is desert soil not suitable for agriculture? [2]
c) What kind of soil do you need for (i) wheat, (ii) cotton, (iii) bajra? [3]
d) Difference between khaddar and bangar soils and say where either of them is found. [3]

Question 5)
a) Explain the difference between ‘deforestation’ and ‘afforestation’ [2]
b) Mention two disadvantages of Tropical Rainforests. [2]
c) i) What are the temperature and rainfall requirements of Tropical Monsoon forests?
ii) Mention any two varieties of trees found in these forests. [3]
d) Mention two important uses for each of the following varieties of trees: teak, sandalwood,
mahogany. [3]

Question 6)
a) i) What is irrigation?
ii) Name two primitive methods of irrigation. [2]
b) Why is well irrigation popular in many parts of India? [2]
c) Mention reasons to explain why water scarcity occurs. [3]
d) i) Mention two dams on the Bhakra Nangal project?
ii) Name the states it helps by irrigating large areas.
iii) Why has the Western Ghats great potential for hydroelectric power? [3]

Question 7)
a) Why are minerals called exhaustible resources? What are mineral ores? [2]
b) What is the significance of coal? [3]
c) How is iron ore exported? Name some of the importing countries of India’s iron ore. [2]
d) i) How is crude oil transported to refineries. [3]
ii) Why are most refineries near the coasts?
iii) Name two refineries which are located away from the coast.

118 Universal Tutorials – X ICSE – Geography Volume 2 of 4


UT Sample Paper 119

Question 8)
a) What are the three kinds of coffee grown in India? Which two states lead in its
production? [2]
b) What is the green revolution? [2]
c) Name three important aspects of the Japanese method of rice cultivation. [3]
d) What is millet? Why are they called tough crops? Name one leading state for each millet.
[3]

Question 9)
a) Mention two reasons to explain the location of the jute textile industry in West Bengal. [2]
b) What are the major problems affecting the cotton textile industry of India? [3]
c) State four geographical factors which should be kept in mind while setting up an agro
based industry. [2]
d) Name any three by-products of the sugar industry. Give one important use of each. [3]

Question 10)
a) What is the difference between heavy electrical and light electrical? [2]
b) Mention two geographical reasons for the growth of IT industries in Bangalore. [2]
c) In reference to petrochemicals: [3]
i) What are its basic requirements?
ii) State three advantages.
iii) Which was the first public sector unit to be set-up?
d) With reference to TISCO. [3]
i) Give its full form and location.
ii) From where does it get power?
iii) Why is its location an advantage in reference to transport?

Question 11)
a) Mention two reasons to explain the need for an efficient transport system? [2]
b) What are some of the problems affecting rail transport? [2]
c) i) Why do so many people still use air transport, inspite of its high costs?
ii) Name two international airports in India. [3]
d) North Eastern India does not have an efficient system of transportation. Explain why. [3]

Volume 2 of 4 Universal Tutorials – X ICSE – Geography 119


120

Attach The Map Along With The Main Answer Sheet

Name: ___________________________________________ UT Roll No.: S– _____________

Test No.: __________________ Batch: ________________ Date: ______________________

120 Universal Tutorials – X ICSE – Geography Volume 2 of 4

You might also like